Sei sulla pagina 1di 227

EACH CHAPTER CONTAIN 3 SETS OF QUESTIONS

CHAPTER 1

Question 1 A
One of the costs of insurance is said to be opportunity costs. This means that if capital and labor were not being used
in the insurance business, they could be used elsewhere and making other productive contributions to
A. Society.
B. The insurance industry.
C. The agency system.
D. Government.

Question 2 C
A small business owner concerned about something happening and not being able to work or earn a living for an ex-
tended period of time due to an accident should purchase
A. Personal liability insurance.
B. Medical insurance.
C. Disability insurance.
D. Universal life insurance.

Question 3 A
Though premiums are an obvious cost of insurance, many insureds believe they are too high because
A. Benefits are intangible until a loss occurs.
B. Premiums are not a regular cost of living.
C. Insurers only use eighty cents from every premium dollar to pay losses.
D. Insureds do not deliberately cause losses.

Question 4 A
A policy that combines property, crime, and liability coverages into one policy is referred to as a(n)
A. Commercial package policy (CPP).
B. Commercial umbrella policy.
C. Workers compensation policy.
D. Employee dishonesty policy.

Question 5 B
Which one of the following statements is correct regarding the benefits provided by insurance?
A. The reduction in losses paid by insurers due to risk control measures benefits individual insureds, but not society as
a whole.
B. Insurance helps reduce the financial burden to society by compensating accident victims.
C. The primary role of insurance is to meet mandatory insurance requirements.
D. Insurance reduces the financial consequences of loss exposures, but not the related uncertainty.

Question 6 C
The process of restoring an individual or organization to a pre-loss financial condition is the process of
A. Subrogation.
B. Premium rebating.
C. Indemnification.
D. Loss mitigation.

1
Question 7 C
Coverage for money, securities, and other property from various causes of loss such as burglary, robbery, theft, and
employee dishonesty typically is provided by
A. Professional liability insurance.
B. Ocean marine insurance.
C. Commercial crime insurance.
D. Inland marine insurance.

Question 8 D
Ideally insurable loss exposures are subject to losses that
A. Occur gradually over long periods of time.
B. Are immeasurable in terms of frequency or severity.
C. Result from unidentifiable causes.
D. Are definite in time, cause, and location.

Question 9 D
Employees are told what to do and do what they are told in a
A. Training environment.
B. Counseling environment.
C. Coaching environment.
D. Compliance environment.

Question 10 A
Which one of the following statements is correct?
A. Contractors must usually provide evidence of liability insurance before a construction contract is granted.
B. Insurers generally do not provide incentives to organizations to implement risk control measures.
C. Insurance policies typically cannot be used to provide evidence of financial resources.
D. One disadvantage of insurance is that it promotes inefficient use of policyholders funds.

Question 11 A
Commercial general liability insurance policies written on an occurrence basis apply to bodily injury and property
damage that occurs during the policy period. This provision supports the principle that insurable loss exposures must
ideally be
A. Definite.
B. Fortuitous.
C. Independent.
D. Pure risks.

Question 12 D
As a single mother on a very tight budget, Ciara is tempted to skimp on her insurance. However, her friend Mehmet
tells her not to skimp on insurance, because it will help manage her cash flows. Which one of the following examples
best illustrates Mehmets point?
A. Ciara finds it difficult to come up with a $787 auto insurance premium every six months.
B. Ciara needs her car to get to her job, and she needs her job to make car payments and pay for her car insurance.
C. When her cars transmission goes out a month after the warranty expires, Ciara is faced with a $1,100 repair bill.
D. When her cars windshield breaks, Ciara has to pay only $100 of the $600 cost of replacing it.

2
Question 13 C
Employees may have increased job satisfaction as a result of perceiving coaching as a form of
A. Counseling.
B. Compliance.
C. Recognition.
D. Discipline.

Question 14 B
Coverage for miscellaneous types of property such as movable property, goods in domestic transit, and property used
in transportation and communication, typically is provided by
A. Auto physical damage insurance.
B. Inland marine insurance.
C. Crime insurance.
D. Business income insurance.

Question 15 C
Which one of the following is an opportunity cost of insurance?
A. The payment of commissions to agents
B. An insurer's loss on invested premiums
C. An insured's funds that could be invested elsewhere if purchasing insurance were not necessary
D. The cost of claims payments that would not have been necessary if insureds' carelessness had not caused losses

Question 1 D
As a single mother on a very tight budget, Ciara is tempted to skimp on her insurance. However, her friend Mehmet
tells her not to skimp on insurance, because it will help manage her cash flows. Which one of the following examples
best illustrates Mehmets point?
A. Ciara finds it difficult to come up with a $787 auto insurance premium every six months.
B. Ciara needs her car to get to her job, and she needs her job to make car payments and pay for her car insurance.
C. When her cars transmission goes out a month after the warranty expires, Ciara is faced with a $1,100 repair bill.
D. When her cars windshield breaks, Ciara has to pay only $100 of the $600 cost of replacing it.

Question 2 C
Coverage for money, securities, and other property from various causes of loss such as burglary, robbery, theft, and
employee dishonesty typically is provided by
A. Professional liability insurance.
B. Ocean marine insurance.
C. Commercial crime insurance.
D. Inland marine insurance.

Question 3 B
The two primary types of health insurance coverage are
A. Sickness and Accident.
B. Medical and Disability Income.
C. Accident and Medical.
D. Disability Income and Sickness.

3
Question 4 B
Liability coverage for loss exposures arising from a business organization's premises and operations, its products, or
its completed work is typically provided by
A. Professional liability insurance.
B. Commercial general liability insurance.
C. Auto liability insurance.
D. Personal liability insurance.

Question 5 C
Oscars custom-built vehicle looks like a sausage sandwich on wheels. He plans to drive it to special events at schools
around the country where it will serve as a mobile billboard to promote his product. Oscar is surprised to learn that
insurers are reluctant to insure his vehicle because it fails to meet one of the ideal characteristics of an insurable risk.
Which characteristic is Oscars vehicle least likely to meet?
A. Definite and measurable
B. Independent and not catastrophic
C. Large number of similar exposure units
D. Pure risk
Question 6 C
Which one of the following is an opportunity cost of insurance?
A. The payment of commissions to agents
B. An insurer's loss on invested premiums
C. An insured's funds that could be invested elsewhere if purchasing insurance were not necessary
D. The cost of claims payments that would not have been necessary if insureds' carelessness had not caused losses

Question 7 A
One of the costs of insurance is said to be opportunity costs. This means that if capital and labor were not being used
in the insurance business, they could be used elsewhere and making other productive contributions to
A. Society.
B. The insurance industry.
C. The agency system.
D. Government.

Question 8 B
An effective coach helps employees develop competencies through
A. Rules, procedures, structure, instructions, and compliance.
B. Questioning, understanding, support, direction, and feedback.
C. Counseling, training, promotions, incentives, and compensation.
D. Multiple tasks, resources, responsibilities, challenges, and processes.

Question 9 A
Though premiums are an obvious cost of insurance, many insureds believe they are too high because
A. Benefits are intangible until a loss occurs.
B. Premiums are not a regular cost of living.
C. Insurers only use eighty cents from every premium dollar to pay losses.
D. Insureds do not deliberately cause losses.

4
Question 10 C
Which one of the following statements is correct with regard to risk and ideally insurable characteristics of loss expo-
sures?
A. An ideally insurable loss exposure should be associated with speculative risk.
B. One purpose of insurance is to enable the insured to profit from a loss.
C. Indemnification is the process of restoring an insured to a pre-loss financial condition.
D. If a loss exposure includes the possibility of gain, it is a more desirable risk to insure.

Question 11 D
Which one of the following statements is correct regarding the benefits that insurance provides?
A. The premiums collected by insurers must be held in cash to be available to pay claims.
B. Insurance provides a source of investment funds for insurers, but not for insureds.
C. Insurers are prohibited from investing in such things as research or technological advancements.
D. Investment income helps keep insurance premiums at a reasonable level.

Question 12 D
Which one of the following statements is correct?
A. Insurers are prohibited from investing in social projects.
B. Insurance provides a source of investment funds for insurers but not for policyholders.
C. Insurers cannot invest premium income because it must be available to pay claims.
D. Insurers investment income helps keep premiums at a reasonable level.

Question 13 A
Liability coverage to individuals and families for bodily injury and property damage arising from the insured's personal
premises or activities is typically provided by
A. Personal liability insurance.
B. Auto liability insurance.
C. Professional liability insurance.
D. Commercial general liability insurance.

Question 14 C
Employees may have increased job satisfaction as a result of perceiving coaching as a form of
A. Counseling.
B. Compliance.
C. Recognition.
D. Discipline.

Question 15 C
The process of restoring an individual or organization to a pre-loss financial condition is the process of
A. Subrogation.
B. Premium rebating.
C. Indemnification.
D. Loss mitigation.

5
Question 1 D
All of the following are types of property insurance, EXCEPT:
A. Fire and allied lines
B. Crime
C. Business income
D. Directors and officers

Question 2 C
The process of restoring an individual or organization to a pre-loss financial condition is the process of
A. Subrogation.
B. Premium rebating.
C. Indemnification.
D. Loss mitigation.

Question 3 D
Which one of the following statements is correct regarding the benefits that insurance provides?
A. The premiums collected by insurers must be held in cash to be available to pay claims.
B. Insurance provides a source of investment funds for insurers, but not for insureds.
C. Insurers are prohibited from investing in such things as research or technological advancements.
D. Investment income helps keep insurance premiums at a reasonable level.

Question 4 B
Which one of the following is an operating cost of insurers?
A. An insured's funds that could be invested elsewhere if purchasing insurance were not necessary
B. Producers' commissions
C. Increased property losses because people have insurance
D. Increased liability loss payments because people have insurance

Question 5 C
Employees may have increased job satisfaction as a result of perceiving coaching as a form of
A. Counseling.
B. Compliance.
C. Recognition.
D. Discipline.

Question 6 B
Which one of the following statements is correct regarding the benefits provided by insurance?
A. The reduction in losses paid by insurers due to risk control measures benefits individual insureds, but not society as
a whole.
B. Insurance helps reduce the financial burden to society by compensating accident victims.
C. The primary role of insurance is to meet mandatory insurance requirements.
D. Insurance reduces the financial consequences of loss exposures, but not the related uncertainty.

6
Question 7 C
Coverage for money, securities, and other property from various causes of loss such as burglary, robbery, theft, and
employee dishonesty typically is provided by
A. Professional liability insurance.
B. Ocean marine insurance.
C. Commercial crime insurance.
D. Inland marine insurance.

Question 8 C
Sometimes the existence of insurance encourages losses. The result of this phenomenon is that it
A. Increases competition in the industry.
B. Reduces agents' commissions.
C. Increases the total cost of insurance.
D. Reduces the term of many policies.

Question 9 C
A homeowners insurance policy typically includes
A. Only property coverage.
B. Only liability coverage.
C. Both property and liability coverage.
D. Umbrella coverage.

Question 10 A
Adhering to the characteristics of an ideally insurable loss exposure in selling insurance help assure that
A. The insurer is able to charge a premium that the insured can afford to pay.
B. The losses associated with it typically involve small amounts.
C. The insurer can charge a high premium for the coverage.
D. The insurer is able to predict the amount and timing of each future loss.

Question 11 B
An effective coach helps employees develop competencies through
A. Rules, procedures, structure, instructions, and compliance.
B. Questioning, understanding, support, direction, and feedback.
C. Counseling, training, promotions, incentives, and compensation.
D. Multiple tasks, resources, responsibilities, challenges, and processes.

Question 12 A
Insurance customers' loss exposures arise out of their activities, as well as
A. Insureds' property.
B. Insurers' requirements.
C. Insureds' goals.
D. Insurers' coverages.

7
Question 13 C
Pravalt Construction Company pays less than its competitors for workers compensation insurance because Pravalt has
had substantially fewer employee injuries than other firms in its class. This illustrates which one of the following bene-
fits of insurance?
A. Meet legal requirements
B. Enable efficient use of resources
C. Promote risk control
D. Reduce social burden

Question 14 C
Which one of the following is an example of a fortuitous loss?
A. Insureds intentionally set a fire and burn their house down.
B. An insured demolishes a deck that has been allowed to rot.
C. An unknown vandal spray-paints graffiti on the insureds garage.
D. The value of an insureds home decreases due to a lack of maintenance.

Question 15 D
Claim buildup is considered to be among which one of the following costs of insurance?
A. Premiums paid
B. Operating costs
C. Opportunity costs
D. Increased losses

8
CHAPTER 2

Question 1 A
What is solvency?
A. The ability of an insurer to meet its obligations as they become due
B. Transparency of policy language
C. The process of a state taking over the assets and obligations of a failing insurer
D. Destructive competition

Question 2 C
The capital of a stock insurance company comes primarily from
A. Sale of insurance policies.
B. Return on invested premium reserves.
C. Sale of company stock.
D. Leveraging the difference from when a premium is paid in and when a claim is paid out.

Question 3 C
Destructive competition in the insurance industry could result in
A. Oversupply of insurance.
B. Excess regulation.
C. Insurance shortages.
D. Inadequate regulation.

Question 4 A
Some communities in the United States celebrate holidays with firecrackers. Firecracker vendors erect stands in the
parking lots of shopping centers before the holidays. One shopping center owner sponsoring a firecracker vendor's
booth found that his insurance did not cover the exposure. He found that none of the insurers licensed to do business
in the state sold insurance coverage for the exposures.
How can the shopping center owner obtain appropriate coverage for the exposure from the firecracker vendor's
booth?
A. By obtaining coverage through the excess and surplus lines market
B. By pooling the risk with other shopping center owners with similar exposures
C. Through a proportionate sharing arrangement with multiple insurers
D. From alien insurers in countries that also celebrate holidays with firecrackers

Question 5 D
A reinsurance company
A. Is formed to write all or part of the insurance for a parent company.
B. Provides primary insurance for loss exposures that private insurers are unwilling to provide.
C. Transfers losses to a primary insurer.
D. Assumes loss exposures from a primary insurer.

9
Question 6 B
Which one of the following is true regarding the administration of the Insurance Regulatory Information System
(IRIS)?
A. If the insurer has financial ratios that are inside predetermined norms, IRIS identifies the company for regulatory
attention.
B. If regulators determine that an insurer is insolvent, the state insurance department places it in receivership.
C. If an insurer cannot be rehabilitated, the state's guaranty fund may be available to increase the effects of the in-
surer insolvency.
D. Under a special provision in state licensing laws, state regulators are empowered to completely take over an in-
surer at any time.

Question 7 A
Which one of the following is a key focus of states' insurance regulation?
A. Insurer licensing.
B. Product branding.
C. Premium taxation.
D. Producer concentration.

Question 8 D
Gen Company is a start-up corporation specializing in providing genetic material for use in new, experimental and of-
ten controversial medical treatments. Reasons why its liability insurance needs may have to be met in the surplus lines
market include which one of the following?
A. Its activities raise moral issues.
B. Gen Company is a mainstream business.
C. Surplus lines insurance company personnel enjoy controversy.
D. Its loss exposures require new forms of coverage.

Question 9 A
Which one of the following best describes what is determined by the insurers staff review of applications from pro-
spective insureds?
A. Whether the characteristics of the customer match the insurers eligibility and selection guidelines
B. Whether the account should be written as a personal insurance policy or a commercial insurance policy
C. Whether any loss control recommendations will be made
D. Whether claims will be paid or denied

Question 10 B
Which one of the following is a federal insurance plan in which the government acts as a partner with a private in-
surer that sells insurance and pays the claims, and then reimburses the insurer for the portion of losses that exceeds
premiums and investment income?
A. Beach and Windstorm Plan
B. National Flood Insurance Program (NFIP)
C. Terrorism Risk Insurance Program (TRIP)
D. Residual Auto Plan

10
Question 11 D
What is the most common reason premium audits are conducted?
A. The amount of actual losses are not known at the start of the policy period.
B. Loss control activities during the policy period may reduce the risk of loss.
C. Claim examiners have discovered unacceptable operations during the policy period.
D. The amount of the loss exposure is unknown at the start of the policy period.

Question 12 B
Which one of the following correctly describes a reason for government involvement in property-casualty insurance?

A. Selling insurance provides the government with a non-tax source of revenues and profits.

B. Government programs can meet legitimate public demands unmet by private insurers.

C. Competition from government plans keeps private insurers premiums competitive.

D. Preventing high-risk individuals or activities from being insured is in the public interest.

Question 13 A
A company interested in improving cash flow should consider meeting its insurance needs through which one of the
following types of insurance organizations?

A. Captive insurers

B. Stock insurers

C. Mutual insurers

D. Reciprocal insurance exchanges

Question 14 C
When deciding to approve or disapprove an insurer's request for a rate, a state insurance commissioner must deter-
mine if the rates are adequate. This means that the rates should be

A. Similar to the rates charged by other insurers operating in the state.

B. Similar for insureds with similar loss exposures.

C. Sufficient to pay all claims and the expenses related to those claims.

D. Able to generate a fair return for the insurer but no to excessive or unrealistic profit.

11
Question 15 A
Riko's insurance agent was caught embezzling several insureds' premiums, which involves what area of insurance
company operations and what punishment by state regulators?

A. Sales; agent license revoked

B. Underwriting; agent license revoked

C. Claim handling; operating license suspended

D. Claim handling; operating license revoked

Question 16 D
Generally, how often must a foreign insurer's license be renewed?

A. Every five years

B. Every two years

C. Semi-annually

D. Annually

Question 17 A
In developing insurance rates, a state mandatory rate law requires the insurer to

A. Use state developed rates.

B. Establish and file rates for all admitted lines of insurance.

C. Use rates developed by the National Association of Insurance Commissioners (NAIC).

D. Maintain rates equal to policyholder surplus.

Question 18 B
Why are insurance regulators concerned about the effects of large catastrophes?

A. They could lead to destructive competition.

B. Insurers may become insolvent.

C. Insurance rates will rise.

D. Licensed insurers will be unable to handle demand.

12
Question 19 A
Which one of the following statements is true?

A. Mutual insurance companies include some large national insurers.

B. Mutual insurers are usually large national insurers.

C. Mutual insurers are exclusively regional or local insurers.

D. Mutual insurers include few regional insurers.

Question 20 A
Which one of the following statements is correct regarding government involvement in insurance?

A. Federal and state government are involved in insurance to facilitate compulsory insurance purchases.

B. Most organizations obtain workers' compensation insurance through federal or state insurance programs.

C. Government insurance plans typically incur significant costs in marketing and sales commissions.

D. Legislators find it more straightforward to invite and analyze bids from private insurers than to establish govern-
ment plans.

Question 21 C
Insurance Company wrote a commercial liability policy for a manufacturer of off-road motorcycles. The potential costs
of the insured's loss exposure exceed Insurance Company's capacity. Insurance Company could consider which type of
contractual transferring agreement to meet its needs?

A. Mutual insurance

B. Reciprocal insurance

C. Reinsurance

D. Interinsurance

Question 22 A
Which one of the following is generally exempt from state insurance regulations pertaining to policy forms and rates?

A. Surplus line insurers

B. Workers compensation insurers

C. Commercial property insurers

D. Health insurers

13
Question 23 D
The two objectives of insurance policy form regulation are to ensure that policies are clear and readable and to

A. Ensure that the insurers rights are protected.

B. Ensure policies are negotiable between an insurer and insured.

C. Limit policies' length and complexity.

D. Detect and address any policy provisions that are unfair.

Question 24 A
Prompt and professional loss adjustment services are a responsibility of what part of an insurer's organization?

A. The claims function

B. The loss control function

C. The premium audit function

D. The underwriting function

Question 25 A
Insurers are required to submit annual financial statements to

A. State insurance departments.

B. The National Association of Insurance Commissioners (NAIC).

C. The Insurance Regulatory Information System (IRIS).

D. The state guaranty fund.

Question 26 C
Unfair trade practices acts involve which one of the following insurance company operations?

A. Rate filings

B. Coverage form design

C. Underwriting

D. Financial reporting

14
Question 27 C
Which one of the following statements concerning government insurance programs is true?

A. Businesses seeking flood insurance under the National Flood Insurance Program (NFIP) must purchase it at local
federal government offices.

B. Various state insurance programs provide crop insurance for perils such as drought, disease, excessive rain and hail.

C. Fair Access to Insurance Requirements (FAIR) plans make basic property insurance available to property owners
who cant get it otherwise.

D. The federal government provides workers compensation insurance to employers who cannot get it from private
insurers.

Question 28 C
Kyle is a professional athlete who has had several successful seasons. He earns $10 million a year, and is hopeful that
his salary will double when he renegotiates his contract next year. His financial adviser recommends that he purchase
an umbrella policy with multi-million dollar limits to provide an excess layer of protection above his primary personal
insurance policies. Kyle's insurance broker has approached many of the large, well-known insurance companies, and
they have balked at writing limits as high as those needed by Kyle. The broker is likely to find the coverage that Kyle
needs in the

A. Standard insurance market.

B. Commercial property and casualty market.

C. Surplus lines insurance market.

D. Admitted property and casualty market.

Question 29 C
Most states regulate the excess and surplus lines market by

A. Establishing a monitoring board comprised of all insurers licensed to do business in the state.

B. Examining the market conduct practices of the unlicensed insurers.

C. Requiring that licensed surplus lines brokers transact business with the unlicensed insurers.

D. Approving the policy forms and rates used by the unlicensed insurers.

15
Question 30 D
An Ohio insurer that is licensed to sell insurance in Michigan is known as what in Michigan?

A. An alien insurer

B. A domestic insurer

C. A captive insurer

D. A foreign insurer

Question 1 C
For an insurer to be considered solvent, states require it to have financial reserves

A. As a fraction of its ordinary expenses.

B. Equal to its ordinary expenses.

C. Well in excess of its ordinary expenses.

D. Double its ordinary expenses.

Question 2 D
An Ohio insurer that is licensed to sell insurance in Michigan is known as what in Michigan?

A. An alien insurer

B. A domestic insurer

C. A captive insurer

D. A foreign insurer

Question 3 A
How does a flex rating law work?

A. Insurers may adjust rates within a range without prior approval.

B. Insurers may change rates without prior approval, but they are subject to regulatory review.

C. Insurers may adjust rates within a range with prior approval.

D. Insurers may adjust rates at will, but are required to file a rate schedule.

16
Question 4 B
Which one of the following statements is true?

A. Insurers must be free to create policies that are in their best interest.

B. Insurance regulators review policies to determine if they benefit consumers.

C. Insurance policies are private contracts, the language of which is largely unregulated.

D. Regulators set coverage standards, but allow insurers to determine policy language.

Question 5 A
Prompt and professional loss adjustment services are a responsibility of what part of an insurer's organization?

A. The claims function

B. The loss control function

C. The premium audit function

D. The underwriting function

Question 6 A
In developing insurance rates, a state mandatory rate law requires the insurer to

A. Use state developed rates.

B. Establish and file rates for all admitted lines of insurance.

C. Use rates developed by the National Association of Insurance Commissioners (NAIC).

D. Maintain rates equal to policyholder surplus.

Question 7 A
Insurers are required to submit annual financial statements to

A. State insurance departments.

B. The National Association of Insurance Commissioners (NAIC).

C. The Insurance Regulatory Information System (IRIS).

D. The state guaranty fund.

17
Question 8 D
What is the most common reason premium audits are conducted?

A. The amount of actual losses are not known at the start of the policy period.

B. Loss control activities during the policy period may reduce the risk of loss.

C. Claim examiners have discovered unacceptable operations during the policy period.

D. The amount of the loss exposure is unknown at the start of the policy period.

Question 9 A
Which one of the following is a key focus of states' insurance regulation?

A. Insurer licensing.

B. Product branding.

C. Premium taxation.

D. Producer concentration.

Question 10 B
The state of Maryland operates a residual auto plan (the Maryland Auto Insurance FundMAIF) that provides cover-
age for drivers who are unable to obtain coverage from private insurers. Which one of the following is the best ra-
tionale for the MAIF program?

A. Auto insurance for high-risk drivers is profitable, and the program enables the state to share in the profits.

B. Auto insurance is compulsory, and the program makes it possible for all drivers to have reasonably priced insur-
ance.

C. Private insurers face limited competition, and the state increases competitive pressures by operating this type of
plan.

D. Private insurers overcharge for auto insurance, and the state provides a low-cost alternative.

Question 11 B
Why are insurance regulators concerned about the effects of large catastrophes?

A. They could lead to destructive competition.

B. Insurers may become insolvent.

C. Insurance rates will rise.

D. Licensed insurers will be unable to handle demand.

18
Question 12 A
What is solvency?

A. The ability of an insurer to meet its obligations as they become due

B. Transparency of policy language

C. The process of a state taking over the assets and obligations of a failing insurer

D. Destructive competition

Question 13 B
Which one of the following statements is correct regarding government insurance programs?

A. Government insurers cannot function as primary insurers for duties such as collecting premiums, providing cover-
age, or paying claims.

B. Government programs can operate as reinsurers, reinsuring 100 percent of the risk or that part in excess of the pri-
vate insurer's retention.

C. Government partnerships with private insurers usually develop in especially desirable lines of business.

D. Government insurance programs cannot operate in direct competition with private insurers.

Question 14 D
Generally, how often must a foreign insurer's license be renewed?

A. Every five years

B. Every two years

C. Semi-annually

D. Annually

Question 15 B
Market conduct regulation focuses on insurers' treatment of applicants for insurance, insureds, and others who pre-
sent claims for coverage. Market conduct regulation affects which one of the following areas of operation?

A. Financial requirements

B. Sales

C. Field examinations

D. Annual statements

19
Question 16 C
Most states regulate the excess and surplus lines market by

A. Establishing a monitoring board comprised of all insurers licensed to do business in the state.

B. Examining the market conduct practices of the unlicensed insurers.

C. Requiring that licensed surplus lines brokers transact business with the unlicensed insurers.

D. Approving the policy forms and rates used by the unlicensed insurers.

Question 17 A
An insurer that was formed for the purpose of earning a profit for its stockholders is a

A. Stock insurer.

B. Reciprocal insurance exchange.

C. Mutual insurer.

D. Captive insurer.

Question 18 C
Which one of the following describes the characteristics of a mutual insurance company?

A. A corporation owned by stockholders that earns profits for the stockholders.

B. An unincorporated association that provides reciprocal coverage to subscribers.

C. A corporation owned by policyholders that provides insurance to its policyholders.

D. An unincorporated association that earns profits for its individual investors.

Question 19 B
Risk control is intended to prevent or reduce losses. When practicing sound risk control, an organization

A. Will not be required to expend economic resources to insure those loss exposures.

B. Is likely to use insurance to treat those loss exposures.

C. Will often be able to eliminate losses from those loss exposures.

D. Is likely to see the same economic benefit as compared to insurance.

20
Question 20 B
Which one of the following insurance customers might be compelled to meet its liability insurance needs through the
excess and surplus lines market?

A. Hobby shop in a proprietor-owned building

B. Major airline requiring multi-billion dollar limits

C. Plumbing outfit with one full-time and two part-time employees

D. Landscaping company with two full-time and five seasonal workers

Question 21 C
All of the following belong to classes of business likely to obtain liability coverage in the excess and surplus lines mar-
ket, EXCEPT:

A. Security services

B. Sports facilities

C. Dry cleaners

D. Amusement parks

Question 22 C
When deciding to approve or disapprove an insurer's request for a rate, a state insurance commissioner must deter-
mine if the rates are adequate. This means that the rates should be

A. Similar to the rates charged by other insurers operating in the state.

B. Similar for insureds with similar loss exposures.

C. Sufficient to pay all claims and the expenses related to those claims.

D. Able to generate a fair return for the insurer but no to excessive or unrealistic profit.

Question 23 A
Which one of the following statements is correct regarding government involvement in insurance?

A. Federal and state government are involved in insurance to facilitate compulsory insurance purchases.

B. Most organizations obtain workers' compensation insurance through federal or state insurance programs.

C. Government insurance plans typically incur significant costs in marketing and sales commissions.

D. Legislators find it more straightforward to invite and analyze bids from private insurers than to establish govern-
ment plans.

21
Question 24 B
Some insurance rating laws allow rates to be put into use immediately but require insurers to files the rates with the
state within a specific period of time. These types of laws are known as

A. File-and-use laws.

B. Use-and-file laws.

C. Prior-approval laws.

D. Flex rating laws.

Question 25 A
A stock insurer is distinguished from a mutual insurer by the fact that

A. Owners are not necessarily insureds.

B. It seeks to generate a profit.

C. It is governed by a board of directors.

D. Owners have voting rights.

Question 26 A
Bonnie operates a small produce store that sells a lot of grapes and bananas. The store has a long history of slip-and-
fall claims against it. Bonnie may look to the E&S market for liability coverage because

A. Of the store's poor loss experience..

B. She needs high limits of coverage.

C. Her exposure is unique.

D. She needs unusually broad coverage.

Question 27 C
Unfair trade practices acts involve which one of the following insurance company operations?

A. Rate filings

B. Coverage form design

C. Underwriting

D. Financial reporting

22
Question 28 B
Which one of the following is a federal insurance plan in which the government acts as a partner with a private in-
surer that sells insurance and pays the claims, and then reimburses the insurer for the portion of losses that exceeds
premiums and investment income?

A. Beach and Windstorm Plan

B. National Flood Insurance Program (NFIP)

C. Terrorism Risk Insurance Program (TRIP)

D. Residual Auto Plan

Question 29 C
The capital of a stock insurance company comes primarily from
A. Sale of insurance policies.

B. Return on invested premium reserves.

C. Sale of company stock.

D. Leveraging the difference from when a premium is paid in and when a claim is paid out.

Question 30 C
Insurance Company wrote a commercial liability policy for a manufacturer of off-road motorcycles. The potential costs
of the insured's loss exposure exceed Insurance Company's capacity. Insurance Company could consider which type of
contractual transferring agreement to meet its needs?

A. Mutual insurance

B. Reciprocal insurance

C. Reinsurance

D. Interinsurance

Question 1 B
Which one of the following is true regarding the administration of the Insurance Regulatory Information System
(IRIS)?
A. If the insurer has financial ratios that are inside predetermined norms, IRIS identifies the company for regulatory
attention.

B. If regulators determine that an insurer is insolvent, the state insurance department places it in receivership.

C. If an insurer cannot be rehabilitated, the state's guaranty fund may be available to increase the effects of the in-
surer insolvency.

D. Under a special provision in state licensing laws, state regulators are empowered to completely take over an in-
surer at any time.

23
Question 2 B
Which one of the following statements is true?

A. Insurers must be free to create policies that are in their best interest.

B. Insurance regulators review policies to determine if they benefit consumers.

C. Insurance policies are private contracts, the language of which is largely unregulated.

D. Regulators set coverage standards, but allow insurers to determine policy language.

Question 3 D
What term refers to the ability of an insurer to meet its obligations as they become due?

A. Capacity

B. Reserves

C. Capital

D. Solvency

Question 4 C
Unfair trade practices acts involve which one of the following insurance company operations?

A. Rate filings

B. Coverage form design

C. Underwriting

D. Financial reporting

Question 5 C
Following the terrorist attacks on the United States, insurers became reluctant to provide property insurance on tar-
get properties until the federal Terrorism Risk Insurance Program (TRIP) was introduced. Which one of the following
social purposes did this government insurance program serve?

A. Incentive to purchase insurance

B. Pooling of loss exposures

C. Prevent economic disruption

D. Reduced risk to society

24
Question 6 A
How does a stock insurer differ from a reciprocal insurance exchange?

A. Stockholders own a stock insurer. Subscribers own a reciprocal insurance exchange.

B. A stock insurer provides insurance to its policyholder-owners. A reciprocal insurance exchange provides insurance to
investors.

C. Both are owned by stockholders. However, the reciprocal insurance exchange provides coverage to investors.

D. Both are formed to provide profit to investors. However, the stock insurer is managed through a board of directors.

Question 7 B
Owners of reciprocal insurance exchanges are also known as

A. Policyholders.

B. Subscribers.

C. Correspondents.

D. Names.

Question 8 A
Bonnie operates a small produce store that sells a lot of grapes and bananas. The store has a long history of slip-and-
fall claims against it. Bonnie may look to the E&S market for liability coverage because

A. Of the store's poor loss experience..

B. She needs high limits of coverage.

C. Her exposure is unique.

D. She needs unusually broad coverage.

Question 9 B
The state of Maryland operates a residual auto plan (the Maryland Auto Insurance FundMAIF) that provides cover-
age for drivers who are unable to obtain coverage from private insurers. Which one of the following is the best ra-
tionale for the MAIF program?

A. Auto insurance for high-risk drivers is profitable, and the program enables the state to share in the profits.

B. Auto insurance is compulsory, and the program makes it possible for all drivers to have reasonably priced insur-
ance.

C. Private insurers face limited competition, and the state increases competitive pressures by operating this type of
plan.

D. Private insurers overcharge for auto insurance, and the state provides a low-cost alternative.

25
Question 10 D
Small United States unincorporated insurance associations, domiciled mainly in Texas, are known as

A. American reciprocals.

B. American mutuals.

C. American captives.

D. American Lloyds.

Question 11 A
Which one of the following statements is correct regarding government involvement in insurance?

A. Federal and state government are involved in insurance to facilitate compulsory insurance purchases.

B. Most organizations obtain workers' compensation insurance through federal or state insurance programs.

C. Government insurance plans typically incur significant costs in marketing and sales commissions.

D. Legislators find it more straightforward to invite and analyze bids from private insurers than to establish govern-
ment plans.

Question 12 A
Which one of the following is true about the functions within an insurance organization?

A. Each function contributes or detracts from the overall effectiveness of the insurer.

B. Insurers use common terminology to identify these functions.

C. These well defined functions operate with a high degree of independence.

D. The essential functions are always performed by insurer personnel.

Question 13 B
Which one of the following is a federal insurance plan in which the government acts as a partner with a private in-
surer that sells insurance and pays the claims, and then reimburses the insurer for the portion of losses that exceeds
premiums and investment income?

A. Beach and Windstorm Plan

B. National Flood Insurance Program (NFIP)

C. Terrorism Risk Insurance Program (TRIP)

D. Residual Auto Plan

26
Question 14 B
Some insurance rating laws allow rates to be put into use immediately but require insurers to files the rates with the
state within a specific period of time. These types of laws are known as

A. File-and-use laws.

B. Use-and-file laws.

C. Prior-approval laws.

D. Flex rating laws.

Question 15 A
In developing insurance rates, a state mandatory rate law requires the insurer to

A. Use state developed rates.

B. Establish and file rates for all admitted lines of insurance.

C. Use rates developed by the National Association of Insurance Commissioners (NAIC).

D. Maintain rates equal to policyholder surplus.

Question 16 A
Prompt and professional loss adjustment services are a responsibility of what part of an insurer's organization?

A. The claims function

B. The loss control function

C. The premium audit function

D. The underwriting function

Question 17 C
All of the following belong to classes of business likely to obtain liability coverage in the excess and surplus lines mar-
ket, EXCEPT:

A. Security services

B. Sports facilities

C. Dry cleaners

D. Amusement parks

27
Question 18 D
Generally, how often must a foreign insurer's license be renewed?

A. Every five years

B. Every two years

C. Semi-annually

D. Annually

Question 19 C
Destructive competition in the insurance industry could result in

A. Oversupply of insurance.

B. Excess regulation.

C. Insurance shortages.

D. Inadequate regulation.

Question 20 A
What is solvency?

A. The ability of an insurer to meet its obligations as they become due

B. Transparency of policy language

C. The process of a state taking over the assets and obligations of a failing insurer

D. Destructive competition

Question 21 A
The premium charged for an insurance policy should be

A. Commensurate with the exposure.

B. Equal to the exposure.

C. Concurrent with the exposure.

D. Equivalent to the exposure.

28
Question 22 D
The two objectives of insurance policy form regulation are to ensure that policies are clear and readable and to

A. Ensure that the insurers rights are protected.

B. Ensure policies are negotiable between an insurer and insured.

C. Limit policies' length and complexity.

D. Detect and address any policy provisions that are unfair.

Question 23 C
All of the following are types of private insurers, EXCEPT:

A. Stock insurers

B. Mutual insurers

C. State workers compensation funds

D. Reciprocal insurance exchanges

Question 24 D
Gen Company is a start-up corporation specializing in providing genetic material for use in new, experimental and of-
ten controversial medical treatments. Reasons why its liability insurance needs may have to be met in the surplus lines
market include which one of the following?

A. Its activities raise moral issues.

B. Gen Company is a mainstream business.

C. Surplus lines insurance company personnel enjoy controversy.

D. Its loss exposures require new forms of coverage.

Question 25 A
Which one of the following statements is true?

A. Mutual insurance companies include some large national insurers.

B. Mutual insurers are usually large national insurers.

C. Mutual insurers are exclusively regional or local insurers.

D. Mutual insurers include few regional insurers.

29
Question 26 B
Market conduct regulation focuses on insurers' treatment of applicants for insurance, insureds, and others who pre-
sent claims for coverage. Market conduct regulation affects which one of the following areas of operation?

A. Financial requirements

B. Sales

C. Field examinations

D. Annual statements

Question 27 C
Kyle is a professional athlete who has had several successful seasons. He earns $10 million a year, and is hopeful that
his salary will double when he renegotiates his contract next year. His financial adviser recommends that he purchase
an umbrella policy with multi-million dollar limits to provide an excess layer of protection above his primary personal
insurance policies. Kyle's insurance broker has approached many of the large, well-known insurance companies, and
they have balked at writing limits as high as those needed by Kyle. The broker is likely to find the coverage that Kyle
needs in the

A. Standard insurance market.

B. Commercial property and casualty market.

C. Surplus lines insurance market.

D. Admitted property and casualty market.

Question 28 C
The capital of a stock insurance company comes primarily from

A. Sale of insurance policies.

B. Return on invested premium reserves.

C. Sale of company stock.

D. Leveraging the difference from when a premium is paid in and when a claim is paid out.

30
Question 29 A
Riko's insurance agent was caught embezzling several insureds' premiums, which involves what area of insurance
company operations and what punishment by state regulators?
A. Sales; agent license revoked

B. Underwriting; agent license revoked

C. Claim handling; operating license suspended

D. Claim handling; operating license revoked

Question 30 C
When deciding to approve or disapprove an insurer's request for a rate, a state insurance commissioner must deter-
mine if the rates are adequate. This means that the rates should be

A. Similar to the rates charged by other insurers operating in the state.

B. Similar for insureds with similar loss exposures.

C. Sufficient to pay all claims and the expenses related to those claims.

D. Able to generate a fair return for the insurer but no to excessive or unrealistic profit.

31
CHAPTER 3

Question 1 C
All of the following are shown on the balance sheet of an insurer, EXCEPT:

A. Admitted assets

B. Unearned premium reserve

C. Premium taxes, licenses, and fees

D. Loss reserves

Question 2 A
Insurers carefully evaluate the interaction between premiums and loss ratio and how changes in each affect profit.
Which one of the following statements concerning the relationship between premium and loss ratio is true?

A. If premiums increase by a greater percentage than incurred losses increase, the loss ratio will decrease.

B. If incurred losses increase at a faster rate than premiums, the loss ratio will improve.

C. If premiums decrease at a faster rate than losses increase, the loss ratio will decrease.

D. If losses decrease at a faster rate than premiums decrease, the loss ratio will increase.

Question 3 A
Which one of the following is shown on an insurer's income statement?

A. Acquisition expenses

B. Cash and short-term investments

C. Loss reserves

D. Loss expense reserves

Question 4 C
Underwriting income (or loss) is calculated by subtracting losses and expenses from

A. Paid premium.

B. Written premium.

C. Earned premium.

D. Unearned premium.

32
Question 5 A
Which one of the following is the best measure of the amount of insurance provided for a given period?

A. Earned premium

B. Paid premium

C. Written premium

D. Unearned premium

Question 6 B
Which one of the following is considered to be an acquisition expense for an insurance company?

A. Dividends

B. Sales commissions

C. Losses

D. Guarantee fund expenses

Question 7 A
Unity Insurance Company (UIC) has an operating ratio of 0.98, a combined ratio of 1.02, and a loss ratio of 0.77. Con-
sidering these ratios, which one of the following statements is true for UIC?

A. UIC's investment income is offsetting losses and expenses from its underwriting operation, allowing an operating
profit.

B. UIC is collecting enough premium to allow a profit from its underwriting operations.

C. UIC is collecting enough premium exclusive of investment income to allow an operating profit.

D. UIC's favorable financial basis expense ratio is offsetting losses, allowing an underwriting profit.

Question 8 C
Which one of the following best explains why underwriting expenses are divided by written premiums rather than
earned premiums in order to calculate an insurer's expense ratio?

A. Loss adjustment expenses are incurred evenly throughout the policy period.

B. The expense ratio does not take into account an insurer's investment income.

C. Many underwriting expenses are incurred at the beginning of the policy period.

D. Insurance coverage is provided evenly throughout the policy period.

33
Question 9 C
The financial report for Hometown Insurer contains the following information:

Earned premiums
$4,000,000

Written premiums
$5,000,000

Net investment income


$1,000,000

Incurred losses
$3,000,000

Incurred underwriting expense


$2,000,000

What is Hometown Insurer's investment income ratio?

A. 10%

B. 20%

C. 25%

D. 50%

Question 10 B
A policy provides coverage starting on April 1 with an annual premium of $800. How much premium is earned as of
June 30?

A. $100

B. $200

C. $600

D. $800

Question 11 C
Rachel is considering investing in the stock of an insurance company. Which one of the following is likely to provide
the best indication of the insurer's financial position at this point in time?
A. Income statement

B. Cash flow report

C. Balance sheet

34
D. Sales report

Question 12 C
ABC Insurance Company's combined ratio is 102. Its operating ratio is 98. These ratios indicate that

A. ABC's underwriting performance is offsetting the lack of investment earnings.

B. ABC's earned premium is greater than the sum of losses and expenses from its underwriting operations.

C. ABC's return on investments is offsetting the unprofitable underwriting performance.

D. ABC is not generating a profit from its core operations.

Question 13 B
The financial report for Apex Insurance contains the following information (all figures in '000s):

Admitted assets $500,000


Nonadmitted assets $400,000
Liabilities $300,000
What is the amount of Apex Insurance's policyholders' surplus?

A. $100,000

B. $200,000

C. $400,000

D. $600,000

Question 14 D
Which one of the following is an admitted asset?

A. Premium balances due in more than ninety days

B. Furniture

C. Supplies

D. Real estate

Question 15 C
The two major sources of an insurer's investment funds are policyholders' surplus and
A. Reserves for loss expenses.

B. Acquisition expenses.

C. Premiums received but not yet used to pay claims.

35
D. Reserves for unearned premiums.

Question 16 C
Loss reserves are often the largest liability on an insurer's balance sheet. Loss reserves include

A. All loss amounts paid for the year.

B. Claim expenses paid for the year.

C. Unpaid loss amounts for claims not yet settled.

D. All loss amounts paid for the year, excluding expenses not yet settled.

Question 17 C
Which one of the following is the correct formula for calculating an insurer's combined ratio?

A. Investment income ratio + overall operating ratio

B. Investment income ratio + premium income ratio

C. Loss ratio + expense ratio

D. Loss ratio + underwriting ratio

Question 18 B
The financial report for Hometown Insurer contains the following information:

Earned premiums $4,000,000


Written premiums $5,000,000
Net investment income $1,000,000
Incurred Losses $3,000,000
Incurred underwriting expense $2,000,000
What is Hometown Insurer's expense ratio?

A. 20%

B. 40%

C. 50%

D. 75%

Question 19 C
An insurer's net underwriting gain or loss provides a better measure of the insurer's marketing, underwriting, and
claims skills than its overall gain or loss from operations. This is true because the overall gain or loss from operations
includes investment gains or losses that
A. Vary over time.
B. Are inversely related to how well the insurer conducts its marketing, underwriting, and claims activities.
C. Are not directly related to how well the insurer conducts its marketing, underwriting, and claims activities.
D. Are generally not subject to income tax.

36
Question 20 B
Which one of the following types of financial statements shows the financial position of an insurance company at any
particular point in time?

A. Income statement

B. Balance sheet

C. Sales report

D. Cash flow report

Question 21 B
If an insurer's admitted assets stayed the same but its liabilities decreased significantly, its policyholders' surplus
would

A. Either stay the same or decrease.

B. Increase.

C. Decrease.

D. Either stay the same or increase.

Question 22 D
An analyst in the financial department for an insurer has calculated the loss ratio to be 1.03. This result indicates that
the insurer is

A. Collecting approximately $1.03 in premium for every $1.00 it pays in claim-related expenses.

B. Collecting approximately $1.03 in premium for every $1.00 it pays in claim-related and underwriting expenses.

C. Paying out approximately $1.03 in claim-related and underwriting expenses for every $1.00 it collects in premium.

D. Paying out approximately $1.03 in claim-related expenses for every $1.00 it collects in premium.

Question 23 C
Compared to its net underwriting gain or loss, an insurer's overall gain or loss from operations gives a

A. Distorted picture of an insurer's profitability.

B. Less complete picture of an insurer's profitability.

C. More complete picture of an insurer's profitability.

D. More conservative picture of an insurer's profitability.

37
Question 24 A
Atwell Insurance Company has the following expenses:

Dividends
$ 50,000
Claim staff salaries $ 90,000
Agents' commissions $ 180,000
Advertising costs $ 30,000
Rent and utilities $ 40,000

What is the amount of Atwell Insurance Company's acquisition expenses?


A. $210,000

B. $260,000

C. $310,000

D. $410,000

Question 25 D
Which one of the following categories of loss expenses can an insurer use to compare its revenue and expenses?

A. Loss reserves at end of period

B. Change in loss reserves

C. Loss reserves at beginning of period

D. Incurred losses

Question 26 B
The salaries of investment professionals who are employed by an insurance company are categorized as

A. General expenses.

B. Investment expenses.

C. Acquisition expenses.

D. Other underwriting expenses.

Question 27 C
On January 1, 2005, the XYZ Insurance Company issued a policy with a one-year policy period. The premium for this
policy was $1,200. What was the unearned premium for this policy as of May 1, 2005?
A. $ 400
B. $ 600
C. $ 800
D. $1,200

38
Question 28 D
On November 1, Peafowl Insurance Company issued a one-year business auto policy to Tri-State Painting Company
with a premium of $24,000. What was the written premium for this policy as of December 31?

A. $ 4,000

B. $ 6,000

C. $20,000

D. $24,000

Question 29 D
Which one of the following is the correct formula for an insurer's overall operating ratio?

A. Loss ratio + Expense ratio

B. Combined ratio - Dividend ratio

C. Loss ratio + Dividend ratio

D. Combined ratio - Investment income ratio

Question 30 B
A decrease in incurred losses will generally cause a decrease in all of the following, EXCEPT:

A. Combined ratio

B. Expense ratio

C. Overall ratio

D. Loss ratio

Question 1 D
An analyst in the financial department for an insurer has calculated the loss ratio to be 1.03. This result indicates that
the insurer is

A. Collecting approximately $1.03 in premium for every $1.00 it pays in claim-related expenses.

B. Collecting approximately $1.03 in premium for every $1.00 it pays in claim-related and underwriting expenses.

C. Paying out approximately $1.03 in claim-related and underwriting expenses for every $1.00 it collects in premium.

D. Paying out approximately $1.03 in claim-related expenses for every $1.00 it collects in premium.

39
Question 2 C
A financial analyst determines that the combined ratio for the DEF Insurance Company was 112 percent for the prior
year. Which one of the following statements best describes how the analyst can interpret this information for the
year?

A. The insurers net profit had to be negative.

B. The insurer had a loss ratio that was over 100 percent.

C. The insurer had an underwriting loss.

D. The insurers expense ratio exceeded its loss ratio.

Question 3 A
Expenses associated with an insurer's underwriting activity include all of the following, EXCEPT:

A. Investment expenses

B. Payment for losses

C. Loss adjustment expenses

D. Premium taxes, licenses, and fees

Question 4 D
The financial report for Hometown Insurer contains the following information:

Earned premiums
$4,000,000

Written premiums $5,000,000

Net investment income


$1,000,000

Incurred losses
$3,000,000

Incurred underwriting expenses


$2,000,000

What is Hometown Insurer's loss ratio?

A. 20%

B. 30%

C. 50%

40
D. 75%

Question 5 B
A policy provides coverage starting on April 1 with an annual premium of $800. How much premium is earned as of
June 30?

A. $100

B. $200

C. $600

D. $800

Question 6 A
Atwell Insurance Company has the following expenses:

Dividends
$ 50,000
Claim staff salaries $ 90,000
Agents' commissions $ 180,000
Advertising costs $ 30,000
Rent and utilities $ 40,000

What is the amount of Atwell Insurance Company's acquisition expenses?


A. $210,000

B. $260,000

C. $310,000

D. $410,000

Question 7 D
Which one of the following is considered to be an acquisition expense for an insurance company?

A. Losses

B. Guarantee fund expenses

C. Dividends

D. Advertising expenses

41
Question 8 A
Insurers carefully evaluate the interaction between premiums and loss ratio and how changes in each affect profit.
Which one of the following statements concerning the relationship between premium and loss ratio is true?

A. If premiums increase by a greater percentage than incurred losses increase, the loss ratio will decrease.

B. If incurred losses increase at a faster rate than premiums, the loss ratio will improve.

C. If premiums decrease at a faster rate than losses increase, the loss ratio will decrease.

D. If losses decrease at a faster rate than premiums decrease, the loss ratio will increase.

Question 9 C
Which one of the following best explains why underwriting expenses are divided by written premiums rather than
earned premiums in order to calculate an insurer's expense ratio?

A. Loss adjustment expenses are incurred evenly throughout the policy period.

B. The expense ratio does not take into account an insurer's investment income.

C. Many underwriting expenses are incurred at the beginning of the policy period.

D. Insurance coverage is provided evenly throughout the policy period.

Question 10 A
The balance sheet shows an insurer's financial position

A. At a particular point in time.

B. For a particular period of time.

C. Based upon revenues versus expenses.

D. Based upon assets minus expenses.

Question 11 C
The financial report for Hometown Insurer contains the following information:

Earned premiums
$4,000,000

Written premiums
$5,000,000

Net investment income


$1,000,000

Incurred losses
$3,000,000

42
Incurred underwriting expense
$2,000,000

What is Hometown Insurer's investment income ratio?

A. 10%

B. 20%

C. 25%

D. 50%

Question 12 D
The investment income ratio indicates the

A. Cost of running an insurer's investment department.

B. Percentage of written premium represented by net investment income.

C. Degree of success achieved by the insurer in controlling losses.

D. Degree of success achieved by the insurer in its investment activities.

Question 13 A
Which one of the following is the best measure of the amount of insurance provided for a given period?

A. Earned premium

B. Paid premium

C. Written premium

D. Unearned premium

Question 14 C
All of the following appear on an insurer's balance sheet, EXCEPT:

A. Assets

B. Liabilities

C. Net income

D. Policyholders' surplus

43
Question 15 D
An insurer's financial statement shows a loss reserve and an unearned premium reserve. These reserves are part of
the insurer's

A. Policyholders' surplus.

B. Nonadmitted assets.

C. Admitted assets.

D. Total liabilities.

Question 16 C
All of the following are true with regard to policyholders' surplus, EXCEPT:

A. It measures the difference between what an insurer owns and what it owes.

B. It provides a financial cushion for the insurer if losses turn out to be higher than expected.

C. It equals nonadmitted assets.

D. It provides resources for expansion.

Question 17 C
On January 1, 2005, the XYZ Insurance Company issued a policy with a one-year policy period. The premium for this
policy was $1,200. What was the unearned premium for this policy as of May 1, 2005?

A. $ 400

B. $ 600

C. $ 800

D. $1,200

Question 18 B
An insurer's income statement shows amounts for all of the following, EXCEPT:

A. Earned premiums

B. Policyholders' surplus

C. Incurred losses

D. Underwriting expenses

44
Question 19 A
The formula for determining an insurer's net underwriting gain or loss is

A. Earned premiums - (losses + underwriting expenses).

B. Written premiums - (losses + underwriting expenses).

C. Earned premiums (losses + underwriting expenses).

D. Written premiums (losses + underwriting expenses).

Question 20 B
An insurer's loss reserve is a(n)

A. Asset designated for paying claims for losses that have already occurred.

B. Liability designated for paying claims for losses that have already occurred.

C. Asset designated for paying claims for losses that might happen in the future.

D. Liability designated for paying claims for losses that might happen in the future.

Question 21 B
Which one of the following is the correct formula for net investment income?

A. Investment income + change in investment expenses

B. Investment income - investment expenses

C. Change in investment expenses - investment income

D. Investment expenses + investment income

Question 22 D
Which one of the following measures the amount an insurer would owe its policyholders in premium refunds if it were
to immediately cease operations?

A. Policyholders' surplus

B. Nonadmitted assets

C. "All other" liabilities

D. Unearned premium reserve

45
Question 23 C
ABC Insurance Company's combined ratio is 102. Its operating ratio is 98. These ratios indicate that

A. ABC's underwriting performance is offsetting the lack of investment earnings.

B. ABC's earned premium is greater than the sum of losses and expenses from its underwriting operations.

C. ABC's return on investments is offsetting the unprofitable underwriting performance.

D. ABC is not generating a profit from its core operations.

Question 24 C
Which one of the following is the correct formula for calculating an insurer's combined ratio?

A. Investment income ratio + overall operating ratio

B. Investment income ratio + premium income ratio

C. Loss ratio + expense ratio

D. Loss ratio + underwriting ratio

Question 25 A
Unity Insurance Company (UIC) has an operating ratio of 0.98, a combined ratio of 1.02, and a loss ratio of 0.77. Con-
sidering these ratios, which one of the following statements is true for UIC?

A. UIC's investment income is offsetting losses and expenses from its underwriting operation, allowing an operating
profit.

B. UIC is collecting enough premium to allow a profit from its underwriting operations.

C. UIC is collecting enough premium exclusive of investment income to allow an operating profit.

D. UIC's favorable financial basis expense ratio is offsetting losses, allowing an underwriting profit.

Question 26 B
Which one of the following is considered to be an acquisition expense for an insurance company?

A. Dividends

B. Sales commissions

C. Losses

D. Guarantee fund expenses

46
Question 27 B
The financial report for Hometown Insurer contains the following information:

Earned premiums $4,000,000


Written premiums $5,000,000
Net investment income $1,000,000
Incurred Losses $3,000,000
Incurred underwriting expense $2,000,000
What is Hometown Insurer's expense ratio?

A. 20%

B. 40%

C. 50%

D. 75%

Question 28 A
The financial report for LMN Insurance contains the following information:

Earned premiums $ 8,000,000


Written premiums
$ 10,000,000
Incurred losses $ 6,000,000
Incurred underwriting expenses $ 4,000,000

What is LMN Insurance's expense ratio?

A. 40%

B. 50%

C. 60%

D. 100%

Question 29 B
The salaries of investment professionals who are employed by an insurance company are categorized as

A. General expenses.

B. Investment expenses.

C. Acquisition expenses.

D. Other underwriting expenses.

47
Question 30 D
Most of the income an insurer receives is either from underwriting or

A. Nonadmitted assets.

B. Losses.

C. Policyholders' surplus.

D. Investments.

Question 1 C
All of the following are shown on an insurer's income statement, EXCEPT:

A. Net underwriting gain (loss)

B. Net investment income

C. Unearned premium reserve

D. Net income

Question 2 B
An insurer's income statement shows its revenues, expenses, and net income

A. At a particular point in time.

B. Over a particular period of time.

C. Focusing primarily on revenues.

D. Based upon assets minus expenses.

Question 3 A
Earned premium for XYZ Insurance (XYZ) for the year was $500 million and incurred losses were $400 million. Incurred
underwriting expenses were $60 million. XYZ had a net investment gain of $40 million for the year. Which one of the
following is XYZ's overall gain or loss from operations?

A. $80 million gain

B. $80 million loss

C. $200 million gain

D. $200 million loss

48
Question 4 A
All of the following are true with regard to the unearned premium reserve, EXCEPT:

A. It equals the admitted portion of unearned premium minus the nonadmitted portion of unearned premium.

B. It represents premiums prepaid by insureds for services that the insurer has not yet rendered.

C. It represents the total premium refunds the insurer would owe its policyholders if it were to immediately cancel all
policies.

D. It is the total of an insurer's unearned premiums on all policies at a particular time.

Question 5 A
Earned premiums are the portion of written premiums that

A. Apply to the part of the policy period that has already occurred.

B. Are available to generate investment income.

C. Are billed at the beginning of the policy period.

D. Apply to the part of the policy period that follows a loss.

Question 6 D
On November 1, Peafowl Insurance Company issued a one-year business auto policy to Tri-State Painting Company
with a premium of $24,000. What was the written premium for this policy as of December 31?

A. $ 4,000

B. $ 6,000

C. $20,000

D. $24,000

Question 7 D
An analyst in the financial department for an insurer has calculated the loss ratio to be 1.03. This result indicates that
the insurer is

A. Collecting approximately $1.03 in premium for every $1.00 it pays in claim-related expenses.

B. Collecting approximately $1.03 in premium for every $1.00 it pays in claim-related and underwriting expenses.

C. Paying out approximately $1.03 in claim-related and underwriting expenses for every $1.00 it collects in premium.

D. Paying out approximately $1.03 in claim-related expenses for every $1.00 it collects in premium.

49
Question 8 B
A decrease in incurred losses will generally cause a decrease in all of the following, EXCEPT:

A. Combined ratio

B. Expense ratio

C. Overall ratio

D. Loss ratio

Question 9 D
Most of the income an insurer receives is either from underwriting or

A. Nonadmitted assets.

B. Losses.

C. Policyholders' surplus.

D. Investments.

Question 10 C
An insurer's loss expense reserve is

A. An asset designated to pay the cost of settling the claims included in the loss reserve.

B. An asset designated to pay claims for losses that have occurred.

C. A liability designated to pay the cost of settling the claims included in the loss reserve.

D. A liability designated to pay claims for losses that have occurred.

Question 11 C
An insurer's net underwriting gain or loss provides a better measure of the insurer's marketing, underwriting, and
claims skills than its overall gain or loss from operations. This is true because the overall gain or loss from operations
includes investment gains or losses that

A. Vary over time.

B. Are inversely related to how well the insurer conducts its marketing, underwriting, and claims activities.

C. Are not directly related to how well the insurer conducts its marketing, underwriting, and claims activities.

D. Are generally not subject to income tax.

50
Question 12 B
The financial report for Hometown Insurer contains the following information:

Earned premiums $4,000,000


Written premiums $5,000,000
Net investment income $1,000,000
Incurred Losses $3,000,000
Incurred underwriting expense $2,000,000
What is Hometown Insurer's expense ratio?

A. 20%

B. 40%

C. 50%

D. 75%

Question 13 C
Which one of the following is true with regard to nonadmitted assets?

A. Regulators allow them to be shown on insurers' financial statements.

B. They can be readily converted to cash at or near their market value.

C. They help prevent an insurer from overstating its true financial condition.

D. Regulators assume that they are readily available for paying claims.

Question 14 C
Which one of the following is the correct formula for calculating an insurer's combined ratio?

A. Investment income ratio + overall operating ratio

B. Investment income ratio + premium income ratio

C. Loss ratio + expense ratio

D. Loss ratio + underwriting ratio

51
Question 15 A
Unity Insurance Company (UIC) has an operating ratio of 0.98, a combined ratio of 1.02, and a loss ratio of 0.77. Con-
sidering these ratios, which one of the following statements is true for UIC?

A. UIC's investment income is offsetting losses and expenses from its underwriting operation, allowing an operating
profit.

B. UIC is collecting enough premium to allow a profit from its underwriting operations.

C. UIC is collecting enough premium exclusive of investment income to allow an operating profit.

D. UIC's favorable financial basis expense ratio is offsetting losses, allowing an underwriting profit.

Question 16 C
A financial analyst determines that the combined ratio for the DEF Insurance Company was 112 percent for the prior
year. Which one of the following statements best describes how the analyst can interpret this information for the
year?

A. The insurers net profit had to be negative.

B. The insurer had a loss ratio that was over 100 percent.

C. The insurer had an underwriting loss.

D. The insurers expense ratio exceeded its loss ratio.

Question 17 C
The largest expense category for most insurers is payment for

A. Additions to loss reserves.

B. Acquisition expenses.

C. Losses arising from claims.

D. Loss adjustment expenses.

Question 18 B
A combined ratio of 1.05 indicates that an insurer is

A. Profitable overall.

B. Not profitable on an underwriting basis.

C. Not profitable overall.

D. Profitable on an underwriting basis.

52
Question 19 B
An insurer's income statement shows amounts for all of the following, EXCEPT:

A. Earned premiums

B. Policyholders' surplus

C. Incurred losses

D. Underwriting expenses

Question 20 A
For a one-year policy covering losses that take several years to develop, all of the following ratios are likely to be re-
vised for several years following the policy period, EXCEPT:
A. Expense ratio
B. Combined ratio
C. Loss ratio
D. Overall operating ratio

Question 21 D
The financial report for Hometown Insurer contains the following information:

Earned premiums $4,000,000


Written premiums $5,000,000
Net investment income $1,000,000
Incurred losses $3,000,000
Incurred underwriting expenses $2,000,000

What was the Hometown Insurer's combined ratio?

A. 75%

B. 95%

C. 105%

D. 115%

Question 22 C
ABC Insurance Company's combined ratio is 102. Its operating ratio is 98. These ratios indicate that

A. ABC's underwriting performance is offsetting the lack of investment earnings.

B. ABC's earned premium is greater than the sum of losses and expenses from its underwriting operations.

C. ABC's return on investments is offsetting the unprofitable underwriting performance.

D. ABC is not generating a profit from its core operations.

53
Question 23 A
Insurers carefully evaluate the interaction between premiums and loss ratio and how changes in each affect profit.
Which one of the following statements concerning the relationship between premium and loss ratio is true?

A. If premiums increase by a greater percentage than incurred losses increase, the loss ratio will decrease.

B. If incurred losses increase at a faster rate than premiums, the loss ratio will improve.

C. If premiums decrease at a faster rate than losses increase, the loss ratio will decrease.

D. If losses decrease at a faster rate than premiums decrease, the loss ratio will increase.

Question 24 B
The balance sheet for ABC Insurance (ABC), dated June 30, shows total admitted assets of $50 million. ABC's loss and
loss adjustment expense reserve is $30 million, its unearned premium reserve is $10 million, and its "all other" liabili-
ties are $5 million. ABC's policyholders' surplus as of June 30 was

A. $0 million.

B. $5 million.

C. $45 million.

D. $50 million.

Question 25 D
Which one of the following is considered to be an acquisition expense for an insurance company?

A. Losses

B. Guarantee fund expenses

C. Dividends

D. Advertising expenses

Question 26 C
Underwriting income (or loss) is calculated by subtracting losses and expenses from

A. Paid premium.

B. Written premium.

C. Earned premium.

D. Unearned premium.

54
Question 27 C
All of the following are true with regard to policyholders' surplus, EXCEPT:

A. It measures the difference between what an insurer owns and what it owes.

B. It provides a financial cushion for the insurer if losses turn out to be higher than expected.

C. It equals nonadmitted assets.

D. It provides resources for expansion.

Question 28 B
Which one of the following is considered to be an acquisition expense for an insurance company?

A. Dividends

B. Sales commissions

C. Losses

D. Guarantee fund expenses

Question 29 B
A policy provides coverage starting on April 1 with an annual premium of $800. How much premium is earned as of
June 30?

A. $100

B. $200

C. $600

D. $800

Question 30 D
Which one of the following categories of loss expenses can an insurer use to compare its revenue and expenses?

A. Loss reserves at end of period

B. Change in loss reserves

C. Loss reserves at beginning of period

D. Incurred losses

55
CHAPTER 4

Question 1 D
When an insurance company appoints an insurance agent to serve as its representative, what specifies the scope of
authority given to the agent in this relationship?

A. Implied authority

B. Binding authority

C. A unilateral contract

D. An agency contract

Question 2 B
In terms of their legal relationships, which one of the following statements accurately describes insurance agents and
insurance brokers?

A. An insurance agent represents the customer while a broker represents the insurer.

B. An insurance agent represents the insurer while a broker represents the customer.

C. Both insurance agents and insurance brokers represent the customer.

D. Both insurance agents and insurance brokers represent the insurer.

Question 3 C
When an insurer appoints an agent to act its behalf, the agent's scope of authority is spelled out in the

A. Broker of record letter.

B. Insurance contract.

C. Agency contract.

D. Letter of intent.

Question 4 C
Insurers using direct response marketing

A. Use personal contact rather than the Internet for sales.

B. Use a select group of agents.

C. Often advertise heavily, which can be costly.

D. Do not target market.

56
Question 5 B
After investing significant resources to develop a Web-based system, a personal lines insurer has decided to expand its
territory by assuming the role of a direct writer and using the Internet as a distribution channel. What is the insurer's
greatest concern with the existing book which used independent agents as the distribution system?

A. The quality of the existing book compared to the new book

B. The agents own the expiriations for the existing book

C. Clear communication with existing customers

D. The service needs of the existing book

Question 6 D
When selecting a distribution channel, which one of the following is an important consideration with regard to cus-
tomers' needs and characteristics?

A. The target market

B. The compatibility of the channel with established channels

C. The expertise of current staff

D. The ease of accessibility within the channel

Question 7 C
Which one of the following statements is correct with respect to insurance distribution system?

A. Insurers specializing in large commercial accounts should select a direct response distribution system.

B. Multiple distribution systems cannot be used simultaneously because they do not generally complement each
other.
C. The expertise of current staff and producers must be sufficient to manage the selected system.

D. All insurance distribution systems are equally effective in accomplishing both sales and service activities.

Question 8 C
All of the following are true about the insurance agency relationship, EXCEPT:

A. The agency agreement is a written agreement between the insurer and an agent.

B. The agency agreement gives the agent the right to represent the insurer and sell insurance on the insurers behalf.

C. The principal is the party the agent authorizes to bind coverage.

D. The agent is the party authorized by the principal to act on the principals behalf.

57
Question 9 D
A bank recently began selling insurance and advertises a discounted auto loan rate to any new- or used-car buyer who
also purchases auto insurance through the bank. This discount is an example of

A. Binding.

B. Contingent commissions.

C. Mixed marketing.

D. Rebating.

Question 10 A
Which one of the following is the primary duty that a principal legally owes to an agent?

A. Paying the agent for services performed

B. Loyalty to the agent

C. Reasonable care in action on behalf of the agent

D. Accounting of the agents money

Question 11 A
An independent business owner or firm that sells insurance by representing customers rather than insurers is

A. An insurance broker.

B. A managing general agency.

C. An exclusive agent.

D. A direct writer.

Question 12 B
Which one of the following statements is correct with respect to selecting a distribution channel?

A. Customers are prepared to spend a reasonable amount of time and effort interacting with a distribution channel if
that channel saves them money.

B. The types of insurance products and services to be delivered by the insurer must fit the marketing channel selected.

C. The majority of customers prefer to speak with a producer or service representative rather than access their ac-
count information online.

D. The primary consideration is what approach will allow the insurer to operate efficiently.

58
Question 13 A
Which one of the following is true concerning the direct response distribution channel to market insurance?

A. It relies heavily on advertising.

B. It relies heavily on producer interaction.

C. Commision costs are generally increased.

D. There is less contact between the insurer and the customer with its use.

Question 14 A
The laws of agency impose all of the following specific duties on all agents, EXCEPT:

A. Knowledge

B. Accounting

C. Loyalty

D. Relaying information

Question 15 A
At the end of the year, some agencies that have carefully selected profitable business throughout the year may earn
a(n)

A. Contingent commission.

B. Sales commission.

C. Indirect commission.

D. Annual commission.

Question 16 B
If a third party sues the agent in connection with activities performed on behalf of the principal, the principal must

A. Reimburse the agent for any liability incurred, whether or not the agent was at fault.

B. Reimburse the agent for any liability incurred, if the agent was not at fault.

C. Bring a countersuit against the third party.

D. Provide legal defense for the agent.

59
Question 17 D
Rhonda is a personal lines insurance producer. Under the terms of her agency contracts, the insurers forward policies
promptly to Rhondas customers and send a premium invoice with the policy. Insureds are instructed to remit pre-
mium payments to their insurer. The insurers calculate Rhondas commissions and forward those commission pay-
ments to her monthly. Rhonda is operating under which one of the following payment procedures?

A. Item basis

B. Statement basis

C. Account current basis

D. Direct bill basis

Question 18 A
Which one of the following statements is true concerning the use of the Internet to market insurance products?

A. Consumers assume that insurance purchased over the Internet will be less expensive than the same product pur-
chased from a producer.

B. Consumers who use the Internet for insurance focus chiefly on service rather than price.

C. Consumers' ability to access competitor information is reduced when insurers market products over the Internet.

D. Insurers' costs for underwriting and claim processing services are increased by having an Internet presence.

Question 19 B
The licensing process for claim representatives involves a background check as well as

A. An undergraduate degree.

B. Ethics requirements.

C. Interning with an experienced adjuster.

D. Customer service training.

Question 20 A
Which one of the following statements is correct?

A. Many producers offer consulting services for which they are paid on a fee basis.

B. Producers are restricted to only providing services to their current insureds.

C. It is illegal for producers to reduce their consulting fees in recognition of commissions received.

60
D. When consulting services are provided, insurance premiums, commissions, and fees are billed together.

Question 21 A
All of the following are considered misrepresentation or false advertising for insurance agents or other insurance per-
sonnel, EXCEPT:

A. Paying a portion of the premium or giving a commission to a policyholder

B. Misrepresenting the advantages of an insurance policy

C. Making a false statement about dividends paid on an insurance policy

D. Using an insurance policy title that misrepresents the true nature of the policy

Question 22 C
A producer will generally review a company's loss run report while performing which one of the following functions?

A. Claims handling

B. Sales

C. Risk management review

D. Customer service

Question 23 D
Which one of the following classifications of insurance personnel must be licensed in each state in which they do busi-
ness?

A. Underwriters of property and casualty insurance

B. Consultants who give insurance advice or counsel about policies

C. Claim adjusters who represent insurers

D. Producers of insurance products

Question 24 D
Which one of the following statements is correct?

A. All insurance producers perform the same functions, which are specified in state insurance regulations.

B. Cold calling, or cold canvas, is generally not used by insurance producers because it is an ineffective method of pro-
specting.

C. Insurance producers are prohibited from accepting referrals from strategic partners, such as banks or real estate
brokers.

D. In the exclusive agent and direct writer marketing systems, insurers might assist producers with prospecting.

61
Question 25 D
When a firm is selecting a distribution channel, which one of the following is an important consideration with regard
to its operations?

A. How quickly can inquiries and transactions be processed in the channel?

B. Are customers willing to pay a premium for personalized products and services through the channel?

C. What are customers' expectations regarding accessibility to the channel?

D. Does the selected channel capitalize on its core capabilities?

Question 26 D
Given the complexity of commercial insurance, which one of the following is best suited for its distribution?

A. Internet

B. Direct response

C. Group marketing

D. Independent agencies

Question 27 D
A producer is usually paid a fee for performing which one of the following functions?

A. Premium collection

B. Prospecting

C. Policy issuance

D. Consulting

Question 28 D
Donna is an insurance producer. She makes an appointment with a potential policyholder to review the prospect's loss
exposures. She asks the prospect to complete a questionnaire to assist her in this effort. Donna is conducting a

A. Customer service analysis.

B. Sales closing call.

C. Cold canvas review.

D. Risk management review.

62
Question 29 D
A business manufactures a product that is harmful to the public if used improperly. More than three licensed insur-
ance companies have refused to provide commercial general liability coverage for the business. The producer should
contact which one of the following to attempt to arrange coverage?

A. A direct writer

B. An independent agent

C. A managing general agent

D. A surplus lines broker

Question 30 B
To function legally as an insurance agent, a producer must

A. Be a college graduate.

B. Be licensed by the state.

C. Work for an insurance company.

D. Have a valid drivers' license.

Question 1 C
Which one of the following statements is correct with respect to insurance distribution system?

A. Insurers specializing in large commercial accounts should select a direct response distribution system.

B. Multiple distribution systems cannot be used simultaneously because they do not generally complement each
other.

C. The expertise of current staff and producers must be sufficient to manage the selected system.

D. All insurance distribution systems are equally effective in accomplishing both sales and service activities.

Question 2 A
Once a state agent's license has been issued, the agent must seek to be appointed by one or more
A. Insurers.

B. Brokers.

C. Trade associations.

D. Insurance departments.

63
Question 3 D
When a firm is selecting a distribution channel, which one of the following is an important consideration with regard
to its operations?

A. How quickly can inquiries and transactions be processed in the channel?

B. Are customers willing to pay a premium for personalized products and services through the channel?

C. What are customers' expectations regarding accessibility to the channel?

D. Does the selected channel capitalize on its core capabilities?

Question 4 A
Which one of the following statements is true concerning the use of the Internet to market insurance products?

A. Consumers assume that insurance purchased over the Internet will be less expensive than the same product pur-
chased from a producer.

B. Consumers who use the Internet for insurance focus chiefly on service rather than price.

C. Consumers' ability to access competitor information is reduced when insurers market products over the Internet.

D. Insurers' costs for underwriting and claim processing services are increased by having an Internet presence.

Question 5 B
After investing significant resources to develop a Web-based system, a personal lines insurer has decided to expand its
territory by assuming the role of a direct writer and using the Internet as a distribution channel. What is the insurer's
greatest concern with the existing book which used independent agents as the distribution system?

A. The quality of the existing book compared to the new book

B. The agents own the expiriations for the existing book

C. Clear communication with existing customers

D. The service needs of the existing book

Question 6 A
Which one of the following would be a critical strength required for a personal lines insurer planning to expand geo-
graphically through Web-based distribution?
A. Financial resources

B. Producer force

C. Coverage expertise

64
D. Service capabilities

Question 7 D
Producers are prohibited from paying a portion of the premiums for a policy or giving any commission to a policy-
holder. Actions of this type are called

A. Tie-in sales.

B. Contingent commissions.

C. Misrepresentation of dividends.

D. Rebating.

Question 8 D
Many insurers marketing through the independent agency system use

A. Word-of-mouth advertising.

B. Direct mail advertising listing every agent with a photo of each.

C. Local advertising focusing on specific agencies.

D. National advertising programs intended to enhance the company image.

Question 9 A
Which one of the following distribution channels is also an efficient way to handle claim reporting and billing inquir-
ies?

A. Call centers

B. Financial institutions

C. Direct response

D. Affinity marketing

Question 10 D
Erin has recently graduated from college and relocated to Chicago. She has purchased a car and rented an apartment,
and now needs to purchase insurance. Erin is looking for minimum coverages and the lowest price available. Which
one of the following is likely the best distribution system to meet Erin's needs?

A. Financial institution

B. Regional broker

C. Independent agent

D. Direct writer Internet search

65
Question 11 D
Which one of the following types of agent is usually paid the same commission rate for new business and renewals?

A. Exclusive agent

B. Direct writer sales agent

C. Managing agent

D. Independent agent

Question 12 C
All of the following are true about the insurance distribution system, EXCEPT:

A. Managing general agencies appoint and supervise independent agents for insurers that operate in the system.

B. Insurance brokers are independent business owners or firms that place insurance by representing the customers,
not insurers.

C. An agency expiration list is the record of policies that have lapsed or been canceled due to overdue premiums.

D. The direct response system advertises to encourage potential customers to contact the insurer to purchase insur-
ance.

Question 13 D
Which one of the following methods of prospecting is more likely to be used by an experienced producer than a new
insurance producer?

A. Interactive Web sites

B. Direct mail

C. Cold canvass

D. Referrals from present clients

Question 14 A
When an independent agency is bought or sold, the agency's most valuable asset is its

A. Expiration lists.

B. Building and personal property.

C. Accounts receivables.

D. Written premiums.

66
Question 15 A
Which one of the following methods of premium collection is generally used for personal insurance policies and small
commercial accounts?

A. Direct bill

B. Account current basis bill

C. Agency bill

D. Item basis bill

Question 16 A
Among other things, a binder provides

A. Applicable coverages and limits.

B. Operations of the insured.

C. Policy conditions and exclusions.

D. Insureds financial profile.

Question 17 D
In the legal relationship known as agency, which one of the following is the principal?

A. Third party

B. Agent

C. Insured

D. Insurer

Question 18 D
A producer is usually paid a fee for performing which one of the following functions?

A. Premium collection

B. Prospecting

C. Policy issuance

D. Consulting

67
Question 19 B
The authority that the principal specifically grants the agent is called

A. Implied authority.

B. Express authority.

C. Apparent authority.

D. Conditional authority

Question 20 D
Which one of the following premium collection methods is used in the majority of personal insurance sales?

A. Item basis

B. Statement basis

C. Account current basis

D. Direct bill process

Question 21 D
Which one of the following statements is true regarding the agency contract?

A. The agent is the principal in the agency contract relationship.

B. An agency contract is also known as an agency endowment.

C. Insurance agency contracts usually have a fixed expiration date, at which time the parties negotiate the renewal.

D. The agency contract states the scope of the agency's authority to conduct business for the insurer.

Question 22 A
The functions that insurance producers perform vary widely from one marketing system to another. Which one of the
following functions do all producers perform?

A. Prospecting

B. Premium collection

C. Risk management review

D. Consulting

68
Question 23 D
For an independent agency or an exclusive agency, the percentage of the premium that goes to the agency or to the
producer for new policies sold or existing policies renewed is the

A. Contingent commission.

B. Profit sharing.

C. Salary.

D. Sales commission.

Question 24 C
Principals often authorize another party to act on their behalf. This relationship is called

A. Broker.

B. Fiduciary.

C. Agency.

D. Attorney in fact.

Question 25 A
The laws of agency impose all of the following specific duties on all agents, EXCEPT:

A. Apparent authority

B. Accounting

C. Loyalty

D. Relaying information

Question 26 B
A risk retention group can write insurance only for its members, and it may write

A. Only personal liability insurance.

B. Only commercial liability insurance.

C. Any line of business approved by the state department of insurance.

D. Only workers compensation insurance.

69
Question 27 D
T. Roy Insurance Company is selecting a distribution system and channel for its products. Which one of the following
best describes the type of consideration that T. Roy should give to its operations?

A. What are customer's expectations regarding accessibility?

B. Will customers be willing to pay a premium for services?

C. Can customers navigate easily within the channel?

D. Does it have the necessary financial resources?

Question 28 C
Which one of the following statements is correct?

A. Insurance agents are not subject to licensing requirements in most states.

B. Licensing requirements for insurance brokers are standardized among all the states.

C. Some states require insurance consultants, who give advice or opinions about polices, to be licensed.

D. Public adjusters, who represent insureds for a fee, are not subject to licensing requirements.

Question 29 C
The principal method of determining a prospect's insurance needs is through

A. Reviewing the prospect's current insurance policies.

B. Determining how much the prospect is willing to pay for insurance coverage.

C. Completing a risk management review for the prospect.

D. Asking the prospect what coverage they would like to purchase.

Question 30 A
Offering a cash payment or something of value to an applicant as an inducement to buy or maintain insurance is
called

A. Rebating.

B. Misrepresentation.

C. Tie-in.

D. Twisting.

70
Question 1 A
All of the following are considered misrepresentation or false advertising for insurance agents or other insurance per-
sonnel, EXCEPT:

A. Paying a portion of the premium or giving a commission to a policyholder

B. Misrepresenting the advantages of an insurance policy

C. Making a false statement about dividends paid on an insurance policy

D. Using an insurance policy title that misrepresents the true nature of the policy

Question 2 D
Which one of the following premium collection methods is used in the majority of personal insurance sales?

A. Item basis

B. Statement basis

C. Account current basis

D. Direct bill process

Question 3 D
Rhonda is a personal lines insurance producer. Under the terms of her agency contracts, the insurers forward policies
promptly to Rhondas customers and send a premium invoice with the policy. Insureds are instructed to remit pre-
mium payments to their insurer. The insurers calculate Rhondas commissions and forward those commission pay-
ments to her monthly. Rhonda is operating under which one of the following payment procedures?

A. Item basis

B. Statement basis

C. Account current basis

D. Direct bill basis

Question 4 D
Given the complexity of commercial insurance, which one of the following is best suited for its distribution?

A. Internet

B. Direct response

C. Group marketing

D. Independent agencies

71
Question 5 B
In an insurance agency relationship, the agent's fundamental responsibility is to act for the benefit of the

A. Insured.

B. Insurer.

C. Broker.

D. Policyholder.

Question 6 A
When an independent agency is bought or sold, the agency's most valuable asset is its

A. Expiration lists.

B. Building and personal property.

C. Accounts receivables.

D. Written premiums.

Question 7 B
Which one of the following statements is correct with respect to selecting a distribution channel?

A. Customers are prepared to spend a reasonable amount of time and effort interacting with a distribution channel if
that channel saves them money.

B. The types of insurance products and services to be delivered by the insurer must fit the marketing channel selected.

C. The majority of customers prefer to speak with a producer or service representative rather than access their ac-
count information online.

D. The primary consideration is what approach will allow the insurer to operate efficiently.

Question 8 D
When a firm is selecting a distribution channel, which one of the following is an important consideration with regard
to its operations?

A. How quickly can inquiries and transactions be processed in the channel?

B. Are customers willing to pay a premium for personalized products and services through the channel?

C. What are customers' expectations regarding accessibility to the channel?

D. Does the selected channel capitalize on its core capabilities?

72
Question 9 C
A producer will generally review a company's loss run report while performing which one of the following functions?

A. Claims handling

B. Sales

C. Risk management review

D. Customer service

Question 10 C
The Wellborn agency represents only Coastal Insurance. Wellborn is an independent contractor but its contract with
Coastal specifies that all insurance it sells will be placed with Coastal. Wellborn is operating under which one of the
following marketing systems?

A. The independent agency system

B. The direct writer system

C. The exclusive agency system

D. The managing general agency system

Question 11 C
It is an unfair trade practice for a lender to require that a borrower purchase insurance from the lender or from any
insurance producer recommended by the lender. This unfair trade practice is called

A. Misrepresentation.

B. Twisting.

C. Tie-in sale.

D. Rebating.

Question 12 A
An independent business owner or firm that sells insurance by representing customers rather than insurers is

A. An insurance broker.

B. A managing general agency.

C. An exclusive agent.

D. A direct writer.

73
Question 13 B
The licensing process for claim representatives involves a background check as well as

A. An undergraduate degree.

B. Ethics requirements.

C. Interning with an experienced adjuster.

D. Customer service training.

Question 14 D
Which one of the following statements is true regarding the agency contract?

A. The agent is the principal in the agency contract relationship.

B. An agency contract is also known as an agency endowment.

C. Insurance agency contracts usually have a fixed expiration date, at which time the parties negotiate the renewal.

D. The agency contract states the scope of the agency's authority to conduct business for the insurer.

Question 15 B
Martin Insurance Company targets manufacturers of specialty products. Its insureds place a high value on producer
expertise, responsiveness, and confidentiality. Martin wants to expand into similar markets. Which one of the follow-
ing insurance marketing distribution systems is most appropriate for Martin Insurance Company?

A. Independent agent network

B. Direct writer

C. Financial institution

D. Surplus lines broker

Question 16 D
When an insurance company appoints an insurance agent to serve as its representative, what specifies the scope of
authority given to the agent in this relationship?

A. Implied authority

B. Binding authority

C. A unilateral contract

D. An agency contract

74
Question 17 D
Many insurers marketing through the independent agency system use

A. Word-of-mouth advertising.

B. Direct mail advertising listing every agent with a photo of each.

C. Local advertising focusing on specific agencies.

D. National advertising programs intended to enhance the company image.

Question 18 B
After investing significant resources to develop a Web-based system, a personal lines insurer has decided to expand its
territory by assuming the role of a direct writer and using the Internet as a distribution channel. What is the insurer's
greatest concern with the existing book which used independent agents as the distribution system?

A. The quality of the existing book compared to the new book

B. The agents own the expiriations for the existing book

C. Clear communication with existing customers

D. The service needs of the existing book

Question 19 A
Which one of the following is the primary duty that a principal legally owes to an agent?

A. Paying the agent for services performed

B. Loyalty to the agent

C. Reasonable care in action on behalf of the agent

D. Accounting of the agents money

Question 20 D
An important question for an insurer to ask when examining customers' needs and characteristics to select a distribu-
tion channel is

A. Who is the target market?

B. Is the expertise of the current staff adequate?

C. Does cost outweigh the benefit?

D. How quickly can inquiries and transactions be processed?

75
Question 21 D
A producer is usually paid a fee for performing which one of the following functions?

A. Premium collection

B. Prospecting

C. Policy issuance

D. Consulting

Question 22 B
A risk retention group can write insurance only for its members, and it may write

A. Only personal liability insurance.

B. Only commercial liability insurance.

C. Any line of business approved by the state department of insurance.

D. Only workers compensation insurance.

Question 23 A
Among other things, a binder provides

A. Applicable coverages and limits.

B. Operations of the insured.

C. Policy conditions and exclusions.

D. Insureds financial profile.

Question 24 C
Insurance Company is updating its marketing plan for the coming year. The management team is proposing a combi-
nation of the distribution systems and channels. A consideration unique to combining distribution systems and chan-
nels is

A. Which system will be the most cost effective to implement that the insurer has used successfully in the past.

B. How the insurer will market to all channels so that potential customers will know what is available to them.

C. If customers will receive a consistent message about products and services no matter which system they choose.

D. If the insurer will offer a discount in premium to customers who purchase insurance over the Internet.

76
Question 25 D
Bill is an insurance producer for ABC Insurance Company (ABC). He represents only ABC when marketing insurance
products. ABC compensates Bill primarily through salary, and he has no ownership rights to policy expirations. Bill is a
producer in the

A. Independent agency marketing system.

B. Exclusive agency marketing system.

C. Captive agency marketing system.

D. Direct writer marketing system.

Question 26 B
Juan received a phone call from an angry policyholder. The insured received a bill as a result of a workers compensa-
tion premium audit. The individual does not understand why premium has increased when his total payroll is lower
than originally projected. Juan advises the insured that he will contact the auditor, review the audit, and call back
with an explanation. Which one of the following producer functions is Juan performing?

A. Consulting

B. Customer service

C. Premium collection

D. Risk management review

Question 27 D
Donna is an insurance producer. She makes an appointment with a potential policyholder to review the prospect's loss
exposures. She asks the prospect to complete a questionnaire to assist her in this effort. Donna is conducting a

A. Customer service analysis.

B. Sales closing call.

C. Cold canvas review.

D. Risk management review.

Question 28 C
Because the agent represents the insurer, the law presumes that

A. Agents are employees of the insurer.

B. Knowledge acquired by the insurer is knowledge acquired by the agent.

C. Knowledge acquired by the agent is knowledge acquired by the insurer.

77
D. All communications between the agent and the insurer will be in writing.

Question 29 B
A nonadmitted insurer is typically a

A. Reinsurer.

B. Surplus lines insurer.

C. Reciprocal insurer.

D. Captive insurer.

Question 30 A
An independent agent received his monthly commissions for the new policies he sold. Even though he wrote 20 new
policies that month, his commission was low. Which one of the following explains why this could occur?

A. Commissions are not fully earned at the time of a sale; other written policies might have been canceled with the
unearned portion of the commission deducted from the insurance company's current payment.

B. The agent failed to provide the level of service required by the insurance company. The insurance commission was
reduced as an incentive for better performance.

C. The agent did not meet the premium volume and profitability levels that were specified in his contract; therefore,
his commission was reduced.

D. The agent is a salaried employee of the insurance company and receives only contingent commissions.

78
CHAPTER 5

Question 1 B
Which one of the following is a condition that increases the likelihood that a person will intentionally cause or exag-
gerate a loss?

A. Physical hazard

B. Moral hazard

C. Morale hazard

D. Legal hazard

Question 2 A
Underwriting management develops underwriting guidelines, which

A. Specify the insurer's attitude toward applicants exhibiting certain characteristics.

B. Vary from one underwriter to another.

C. Set firm standards that rarely, if ever, are modified.

D. Ensure that unprofitable business will be offset by profitable business.

Question 3 C
Victor, sales manager for Hallbinger's Insurance Company wants the company to grow in written premium. He wants
his agents to write more auto and homeowner business. In order to do so, the company must have adequate capacity.
Which one of the following impacts Victor's agents and the company the most concerning surplus and capacity?

A. Claims must pay only covered losses and have low expenses.

B. Research and development must create policies which are marketable.

C. Underwriting practices must generate policy premiums that exceed losses and expenses.

D. Sales must write profitable business not loss producing business.

Question 4 C
To achieve profitability, the underwriting function serves which one of the following additional purposes?

A. Enforcing underwriting goals

B. Adding to policyholder surplus

C. Guarding against adverse selection

79
D. Transferring the risks of individuals and businesses

Question 5 D
Which one of the following will draft a manuscript policy or endorsement that is worded to address the specific needs
of the insured?

A. Underwriting supervisor

B. Staff underwriter

C. Underwriting specialist

D. Line underwriter

Question 6 C
While developing goals for its book of business, an insurer's staff decides to increase its market share of workers com-
pensation insurance in the construction industry. This type of underwriting activity is known as

A. Reviewing and revising rating plans.

B. Developing underwriting guides.

C. Formulating underwriting policy.

D. Providing assistance to policyholders.

Question 7 B
Unfair discrimination is prohibited under state insurance laws as an unfair trade practice. However, an insurer's ability
to discriminate fairly is essential so

A. Insurers can select the specific people in a given area to be offered coverage.

B. Insureds are charged a premium commensurate with their loss exposures.

C. Insureds can be guaranteed the coverages they want at the price they request.

D. Insurers can refuse to provide coverage based on an applicant's age and marital status.

Question 8 D
Which one of the following is true regarding the delegation of underwriting authority by underwriting management?

A. Highly centralized insurers restrict underwriting authority to both the home and field offices.

B. Front-line underwriters operate primarily from the home office.

C. Most insurers' underwriting decisions are either highly centralized or highly decentralized.

D. The extent of underwriting authority granted to an agent generally depends on premium volume and loss experi-
ence.

80
Question 9 B
Which one of the following describes the effect underwriting standards can have on premium?

A. If more causes of loss are covered, premiums will increase.

B. Insurers can lower premiums for better-than-average risks.

C. The more services provided, the more premiums will increase.

D. Reduced cash flow drives up premium interest charges.

Question 10 D
An underwriter receives a renewal application for a property quote from his producer. The applicant has had a series
of small losses in the recent past. The underwriter is debating whether to reject the submission or quote it with a
property deductible of $5,000. During which one of the following steps in the underwriting process is the underwriter
involved?

A. Evaluating the submission

B. Implementing the underwriting decision

C. Monitoring underwriting decisions

D. Developing underwriting alternatives

Question 11 B
Underwriting management monitors underwriting results

A. By issuing periodic bulletins to field offices.

B. Through a process called an underwriting audit.

C. To determine if coverage changes will increase sales.

D. To allow the insured to adjust goals if results are not as expected.

Question 12 D
The state advisory organization has developed a new homeowners form, which increases the personal property limits
for guns, silverware, jewelry, and boats. Jancy Insurance Company is considering modifying this form for its own use.
Which one of the following of Jancy's staff would be involved in this research and development of this new form?

A. Merve, the claims manager

B. Antonia, the accounting manager

C. Clara, the underwriting manager

D. Emma, the staff underwriter

81
Question 13 D
A situation that occurs because people with the greatest probability of loss are the ones likely to purchase insurance is
known as

A. Underwriting selection.

B. Applicant "pre-qualification."

C. Loss exposure evaluation.

D. Adverse selection.

Question 14 A
One of the responsibilities of underwriting management is to arrange reinsurance. One type of reinsurance is ar-
ranged to automatically reinsure a portion of all eligible risks of the primary insurer. This arrangement is called

A. Treaty reinsurance.

B. Temporary reinsurance.

C. Facultative reinsurance.

D. Underwriting reinsurance.

Question 15 B
Which one of the following is the term used to refer to a situation that occurs because people with the greatest proba-
bility of loss are the ones most likely to purchase insurance?

A. Spread of risk

B. Adverse selection

C. Probable maximum loss

D. Risk inconsistency

Question 16 D
A rating plan available to commercial insurance applicants that awards debits and credits to a submission based on
specific categories is known as a(n)

A. Experience rating plan.

B. Financial rating plan.

C. Retrospective rating plan.

D. Schedule rating plan.

82
Question 17 A
An insurance company needs to hire an employee with knowledge of insurance policy forms and the ability to relate
policy provisions to the loss exposures of individual policyholders. The person will also prepare premium quotes and
process cancellations. This company is currently in need of a

A. Line underwriter.

B. Staff underwriter.

C. Producer.

D. Premium auditor.

Question 18 B
Underwriting guidelines reflect the levels of

A. Written premium by type of insurance.

B. Underwriting authority.

C. Loss exposure.

D. Responsibility by job title.

Question 19 B
According to state insurance laws, unfair discrimination occurs when insurers deny coverage based solely on certain
factors. Those factors include all of the following, EXCEPT:

A. The applicant's geographic location

B. The applicant's ability to pay the premium

C. The applicant's gender

D. The applicant's race

Question 20 C
The law of large numbers states that as the number of similar but independent exposure units increases,

A. The relative accuracy of predictions about future outcomes decreases.

B. Divergence from the norm will be in direct proportion to the increase in size of the sample.

C. The relative accuracy of predictions about future outcomes increases.

D. Divergence from the norm will be in inverse proportion to the increase in size of the sample.

83
Question 21 B
Which one of the following tasks is the responsibility of a staff underwriter as opposed to a line underwriter?

A. Assist producers and insureds in determining appropriate coverage

B. Review and revise rating plans

C. Verify that policies are issued with appropriate forms

D. Prepare premium quotations

Question 22 B
Sports, Inc. is looking to obtain a commercial property policy on its building for $400,000. If their insurer decides to
charge them a rate of $0.50 per $100 of building insurance, what premium must Sports, Inc. pay?

A. $1,000

B. $2,000

C. $4,000

D. $8,000

Question 23 A
All of the following are principal sources of underwriting information, EXCEPT:

A. Underwriting authority reports

B. Producers

C. Financial rating services

D. Applications

Question 24 B
Why do states require that insurers notify the insured before a policy is to be canceled or nonrenewed?

A. To provide the state an opportunity to investigate the reason for the cancellation or nonrenewal

B. To give the policyholder an opportunity to replace the coverage

C. To eliminate the possibility of an insurer canceling all policies covering a category of business or in a geographic
area

D. To reduce the opportunity for an insurer to redline

84
Question 25 C
Class rating relies on

A. All insureds being charged the same rate.

B. A highly refined classification.

C. Insureds with similar characteristics.

D. Similar frequency and severity of loss.

Question 26 C
Which one of the following is the fundamental measure of loss exposure used in insurance rating?

A. Premium

B. Rate

C. Exposure unit

D. Risk

Question 27 D
In individual rating,

A. Underwriters are often forced to make an arbitrary judgment.

B. Individual insureds are assigned to a specific class.

C. The law of large numbers allows accurate premium determination.

D. Points are added or subtracted to arrive at a rate.

Question 28 A
If a manufacturer has $1 million in annual sales, its liability insurance is priced per 1,000 of sales, and the per-unit rate
is $0.50, what is the manufacturer's liability insurance premium?

A. $500

B. $1,000

C. $5,000

D. $10,000

85
Question 29 A
If an insurance applicant charges that an insurer has rejected the application due to prohibited unfair discrimination,
what is a likely valid defense to the charge?

A. The risk did not meet objective underwriting criteria.

B. The risk is located in a distressed area of a city.

C. The applicant could not demonstrate stability through marriage records or other community ties.

D. Unfair trade practices laws do not prohibit denying coverage for all discrimination, only in cases of racial discrimi-
nation.

Question 30 B
For the law of large numbers to function properly, exposure units must be

A. Equal.

B. Independent.

C. Random.

D. Accurate.

Question 1 B
An underwriter is working with a risk control representative on a new opportunity. Given the complexity of the expo-
sures, a physical survey is conducted by the risk control representative, and a formal report is prepared. The report
reveals that the insured owns several unattended properties, and that they were unlocked when the risk control repre-
sentative conducted the survey. This failure to properly secure an unattended building would be an indicator to the
underwriter of which one of the following types of hazard?

A. Moral

B. Morale

C. Legal

D. Physical

Question 2 C
Staff underwriters share research of the market responsibilities with actuarial and marketing departments. Research
includes an ongoing evaluation of which one of the following?
A. Revision to coverage forms

B. Average loss ratio for a book of business

C. Optimal product mix in the book of business

86
D. The hit ratios by underwriter
Question 3 D
A rating plan available to commercial insurance applicants that awards debits and credits to a submission based on
specific categories is known as a(n)

A. Experience rating plan.

B. Financial rating plan.

C. Retrospective rating plan.

D. Schedule rating plan.

Question 4 C
How do states prohibit unfair discrimination by insurers?

A. By reviewing each policy cancellation or nonrenewal and the justification for the insurer's action

B. By requiring reunderwriting of policyholder complaints

C. By prohibiting rejection of applications due to certain characteristics without justification

D. By examining insurers' mission statements and objectives

Question 5 C
Which one of the following is the fundamental measure of loss exposure used in insurance rating?

A. Premium

B. Rate

C. Exposure unit

D. Risk

Question 6 D
Tania receives an application for homeowner's insurance for a home valued at $500,000 from Patrick, the producer for
Keithly Insurance Company. Since Tania's underwriting authority is $350,000 on property risks, she must refer it to
Lachlan, her underwriting supervisor. Which one of the following additional functions for underwriting profitability
does this case illustrate?

A. Guarding against adverse selection

B. Adequacy of trained staff

C. Prevention of errors and omissions

D. Enforcing underwriting guidelines

87
Question 7 D
Judgment rates are used

A. Because no two risks are identical.

B. Because wide variations in actual losses occur from one insured to the next.

C. Because the underwriter has experience with comparable exposures.

D. Because there is no established premium-determination system.

Question 8 D
The state advisory organization has developed a new homeowners form, which increases the personal property limits
for guns, silverware, jewelry, and boats. Jancy Insurance Company is considering modifying this form for its own use.
Which one of the following of Jancy's staff would be involved in this research and development of this new form?

A. Merve, the claims manager

B. Antonia, the accounting manager

C. Clara, the underwriting manager

D. Emma, the staff underwriter

Question 9 B
An insurance company must have adequate policyholders' surplus if it wishes to

A. Avoid adverse selection.

B. Increase its written premium volume.

C. Produce profitable business.

D. Generate premiums that exceed losses.

Question 10 D
Underwriters can best minimize the effects of adverse selection by

A. Rejecting applicants who have had any prior losses.

B. Choosing to insure only those risks with a high probability of loss.

C. Accepting all applicants that are submitted by the producer.

D. Carefully selecting the applicants whose loss exposures they are willing to insure.

88
Question 11 A
If a manufacturer has $1 million in annual sales, its liability insurance is priced per 1,000 of sales, and the per-unit rate
is $0.50, what is the manufacturer's liability insurance premium?

A. $500

B. $1,000

C. $5,000

D. $10,000

Question 12 C
To achieve profitability, the underwriting function serves which one of the following additional purposes?

A. Enforcing underwriting goals

B. Adding to policyholder surplus

C. Guarding against adverse selection

D. Transferring the risks of individuals and businesses

Question 13 B
During which one of the following steps in the underwriting process would an underwriter request a risk control visit
to a prospective policyholder's location?

A. Monitoring the loss exposures

B. Evaluating the submission

C. Determining underwriting alternatives

D. Implementing the underwriting decision

Question 14 A
Underwriting management develops underwriting guidelines, which

A. Specify the insurer's attitude toward applicants exhibiting certain characteristics.

B. Vary from one underwriter to another.

C. Set firm standards that rarely, if ever, are modified.

D. Ensure that unprofitable business will be offset by profitable business.

89
Question 15 C
The focus of line underwriter is

A. Managing the risk selection process.

B. Conducting education and training.

C. Evaluating new submissions and renewal underwriting.

D. Securing and maintaining treaty reinsurance.

Question 16 B
Sports, Inc. is looking to obtain a commercial property policy on its building for $400,000. If their insurer decides to
charge them a rate of $0.50 per $100 of building insurance, what premium must Sports, Inc. pay?

A. $1,000

B. $2,000

C. $4,000

D. $8,000

Question 17 B
Underwriting management arranges treaty reinsurance to reinsure

A. Individual accounts.

B. A group or group of insureds.

C. All insurance of the primary insurer.

D. Insureds that do not meet underwriting guidelines.

Question 18 A
How does the law of large numbers assist insurers?

A. Allows reliable estimates of future loss amounts

B. Assures a profit, given sufficient number of policies

C. Prevents catastrophic losses from any single event

D. Predicts who will likely suffer losses

90
Question 19 D
Rating systems rely on

A. Grouping insureds with similar loss characteristics.

B. Investment earnings supplementing premiums.

C. Assigning all insureds to classes.

D. Statistical analysis of past losses.

Question 20 A
Produce, such as apples or grapes, may be priced by the unit or by the pound. In insurance, these measures are similar
to

A. Exposure units.

B. Rates.

C. Premiums.

D. Construction cost index.

Question 21 C
The Bakery, an applicant for commercial property insurance, has experienced a series of property losses over the past
three years. In order to find this applicant viable, the underwriter agrees to increase the property deductible. This type
of underwriting modification is known as

A. Requiring loss control measures.

B. Using schedule rating modifications.

C. Amending policy terms and conditions.

D. Using facultative reinsurance.

Question 22 B
While evaluating an application, an underwriter thinks that the class of business is not one that the company wishes
to write, but would need to physically inspect to be certain. The underwriter also realizes that the business is located
in an undesirable section of the city. Based on the location alone, the underwriter decides not to issue the policy. Ac-
cording to many state insurance laws, this would be an example of

A. Diversification.

B. Unfair discrimination.

C. Fair discrimination.

D. Regulated discrimination.

91
Question 23 B
Which one of the following tasks is the responsibility of a staff underwriter as opposed to a line underwriter?

A. Assist producers and insureds in determining appropriate coverage

B. Review and revise rating plans

C. Verify that policies are issued with appropriate forms

D. Prepare premium quotations

Question 24 B
Julio, a producer for Argot's Insurance Company (Argot) submits an application for a fifty year old residence to
Mathilde, the underwriter. Julio wants to know why the homeowner rate is higher for his fifty year old residence.
Mathilde explains that the homeowner rate for the fifty year old residence is not the same for a five year old resi-
dence. Which one of the following underwriting profitability functions of Argot is Mathilde's explanation supporting?

A. Guarding against adverse selection

B. Ensuring adequate policyholder's surplus

C. Enforcing policy guidelines

D. Protecting the homeowner rate

Question 25 D
For an insurer to succeed in the long run,

A. Its investment income must exceed claims and expenses.

B. Its premium income must be adequate to pay claims.

C. Its investment income must be adequate to generate profit for investors.

D. Its premium and investment income must be adequate to pay claims and expenses.

Question 26 B
Why do states require that insurers notify the insured before a policy is to be canceled or nonrenewed?

A. To provide the state an opportunity to investigate the reason for the cancellation or nonrenewal

B. To give the policyholder an opportunity to replace the coverage

C. To eliminate the possibility of an insurer canceling all policies covering a category of business or in a geographic
area

D. To reduce the opportunity for an insurer to redline

92
Question 27 D
Which one of the following will draft a manuscript policy or endorsement that is worded to address the specific needs
of the insured?

A. Underwriting supervisor

B. Staff underwriter

C. Underwriting specialist

D. Line underwriter

Question 28 C
How do underwriting guidelines help an insurer achieve its goals?

A. By establishing the criteria required for treaty reinsurance to apply

B. By creating a channel for communication for the insurer's vision, mission, and objectives

C. By providing rules that guide underwriters toward consistent decisions

D. By maintaining a consistently applied set of behavioral measurements against which an individual's performance
will be measured

Question 29 A
One of the responsibilities of underwriting management is to arrange reinsurance. One type of reinsurance is ar-
ranged to automatically reinsure a portion of all eligible risks of the primary insurer. This arrangement is called

A. Treaty reinsurance.

B. Temporary reinsurance.

C. Facultative reinsurance.

D. Underwriting reinsurance.

Question 30 B
In most states, how many days' notice must an insured be given of an insurer's intent to cancel or nonrenew cover-
age?

A. No more than 10 days

B. At least 30 days

C. Not fewer than 45 days

D. At least 60 days

93
Question 1 C
The principal purpose of underwriting is to

A. Only insure those applicants with minimal or no loss exposure.

B. Educate producers regarding the insurers underwriting guidelines.

C. Develop and maintain a profitable portfolio for the insurer.

D. Decline to insure those applicants with a risk of incurring a severe loss.

Question 2 A
An insurance company needs to hire an employee with knowledge of insurance policy forms and the ability to relate
policy provisions to the loss exposures of individual policyholders. The person will also prepare premium quotes and
process cancellations. This company is currently in need of a

A. Line underwriter.

B. Staff underwriter.

C. Producer.

D. Premium auditor.

Question 3 B
While evaluating an application, an underwriter thinks that the class of business is not one that the company wishes
to write, but would need to physically inspect to be certain. The underwriter also realizes that the business is located
in an undesirable section of the city. Based on the location alone, the underwriter decides not to issue the policy. Ac-
cording to many state insurance laws, this would be an example of

A. Diversification.

B. Unfair discrimination.

C. Fair discrimination.

D. Regulated discrimination.

Question 4 D
All of the following are underwriting activities typically performed by staff underwriters, EXCEPT:

A. Research the market

B. Formulate underwriting policy

C. Conduct underwriting audits

D. Support producers and insureds

94
Question 5 D
Bob is shopping for auto insurance. Insurance Comany A's rating system determines that Bob should be charge $800
per auto. If Bob is looking to insure four autos, what premium will he be charged by Insurance Company A?

A. $200

B. $800

C. $2,400

D. $3,200

Question 6 B
A type of individual rate that is used to develop a premium for a unique exposure for which there is no established
rate is known as

A. A retrospective rate.

B. A judgment rate.

C. An experience rate.

D. A schedule rate.

Question 7 C
While developing goals for its book of business, an insurer's staff decides to increase its market share of workers com-
pensation insurance in the construction industry. This type of underwriting activity is known as

A. Reviewing and revising rating plans.

B. Developing underwriting guides.

C. Formulating underwriting policy.

D. Providing assistance to policyholders.

Question 8 B
Underwriting management monitors underwriting results

A. By issuing periodic bulletins to field offices.

B. Through a process called an underwriting audit.

C. To determine if coverage changes will increase sales.

D. To allow the insured to adjust goals if results are not as expected.

95
Question 9 D
Which one of the following is true regarding the delegation of underwriting authority by underwriting management?

A. Highly centralized insurers restrict underwriting authority to both the home and field offices.

B. Front-line underwriters operate primarily from the home office.

C. Most insurers' underwriting decisions are either highly centralized or highly decentralized.

D. The extent of underwriting authority granted to an agent generally depends on premium volume and loss experi-
ence.

Question 10 D
For an insurer to succeed in the long run,

A. Its investment income must exceed claims and expenses.

B. Its premium income must be adequate to pay claims.

C. Its investment income must be adequate to generate profit for investors.

D. Its premium and investment income must be adequate to pay claims and expenses.

Question 11 B
An underwriter is working with a risk control representative on a new opportunity. Given the complexity of the expo-
sures, a physical survey is conducted by the risk control representative, and a formal report is prepared. The report
reveals that the insured owns several unattended properties, and that they were unlocked when the risk control repre-
sentative conducted the survey. This failure to properly secure an unattended building would be an indicator to the
underwriter of which one of the following types of hazard?
A. Moral

B. Morale

C. Legal

D. Physical

Question 12 B
Why do states require that insurers notify the insured before a policy is to be canceled or nonrenewed?

A. To provide the state an opportunity to investigate the reason for the cancellation or nonrenewal

B. To give the policyholder an opportunity to replace the coverage

C. To eliminate the possibility of an insurer canceling all policies covering a category of business or in a geographic
area

D. To reduce the opportunity for an insurer to redline

96
Question 13 A
Produce, such as apples or grapes, may be priced by the unit or by the pound. In insurance, these measures are similar
to

A. Exposure units.

B. Rates.

C. Premiums.

D. Construction cost index.

Question 14 A
Rating agencies such as Dun & Bradstreet (D&B) provide

A. Data on credit ratings of businesses.

B. Information about business operations' safety records.

C. Data on motor vehicle reports.

D. Aggregate loss experience data.

Question 15 B
According to state insurance laws, unfair discrimination occurs when insurers deny coverage based solely on certain
factors. Those factors include all of the following, EXCEPT:

A. The applicant's geographic location

B. The applicant's ability to pay the premium

C. The applicant's gender

D. The applicant's race

Question 16 A
If a manufacturer has $1 million in annual sales, its liability insurance is priced per 1,000 of sales, and the per-unit rate
is $0.50, what is the manufacturer's liability insurance premium?

A. $500

B. $1,000

C. $5,000

D. $10,000

97
Question 17 D
Maja's, a professional liability underwriter, primary responsibilities involve formulating underwriting policy and revis-
ing underwriting guides for various lines of professional liability insurance that will be used by personnel in the com-
panys field offices. Maja is functioning as a

A. Chartered underwriter.

B. Field underwriter.

C. Line underwriter.

D. Staff underwriter.

Question 18 D
Which one of the following statements regarding underwriter responsibilities is correct?

A. Line underwriters often serve on industry committees that study standard policy forms and recommend changes.

B. Staff underwriters offer valuable technical assistance to the insured's risk manager and the producer who is directly
responsible for determining what coverage best meets the insured's needs.

C. Line underwriters are generally responsible for completing underwriting audits of the branch or region in which
they work.

D. Staff underwriters typically share responsibility for researching fundamental insurance issues with the actuarial
and marketing departments.

Question 19 B
Sofie and the members of her homeowner's association have homeowner's policies on their coastal dwellings. They
have heard that there will be eleven named hurricane storms predicted for their coastal area this year. Sofie and the
homeowner association members plan to increase their homeowner's property coverage limits to handle their concern
about the possible hurricane damage. Their action is known as

A. Property limits assessment.

B. Adverse selection.

C. Prevention of errors and omission claims.

D. Prevention of inadequate property limits.

98
Question 20 D
Which one of the following statements is correct with regard to the submission evaluation step in the underwriting
process?

A. While producers for direct writing and exclusive agency insurers perform field underwriting to screen applicants,
independent agents and brokers generally do not.

B. When an insurance application form is completed properly, an underwriter normally does not need any additional
information.

C. An applicants financial status as indicated by a financial rating service is an incidental piece of underwriting infor-
mation for evaluating potential loss exposures.

D. Government records that provide underwriting information include motor vehicle reports, criminal records, and civil
court records.

Question 21 A
The overarching purpose of underwriting is

A. To develop and maintain a profitable book of business.

B. Make and sustain a profit.

C. Achieve the underwriting goals of the company.

D. Add to policyholder surplus.

Question 22 A
Underwriting management develops underwriting guidelines, which

A. Specify the insurer's attitude toward applicants exhibiting certain characteristics.

B. Vary from one underwriter to another.

C. Set firm standards that rarely, if ever, are modified.

D. Ensure that unprofitable business will be offset by profitable business.

Question 23 B
If a line of insurance is class rated, with all insureds possessing similar characteristics and paying the same rate, what
determines the difference in premiums insureds pay?

A. Considerations for profits and expenses

B. Number of exposure units

C. Credits and debits for changes in exposures

99
D. Data from insurance advisory organizations

Question 24 C
Which one of the following is the fundamental measure of loss exposure used in insurance rating?

A. Premium

B. Rate

C. Exposure unit

D. Risk

Question 25 A
One of the roles of underwriting management is arranging reinsurance. Which one of the following statements ex-
plains how treaty reinsurance helps an insurer meet its obligations?

A. The reinsurer automatically assumes a portion of all of the primary insurer's insurance that is eligible under the
treaty.

B. The reinsurer delegates underwriting authority to the primary insurer, helping the primary insurer achieve con-
sistent results.

C. The reinsurer monitors the results of the primary insurer's underwriting guidelines to ensure compliance.

D. The reinsurer assumes a portion of the losses from all policies that have been specifically listed and insured under
the treaty.

Question 26 C
An applicant for commercial property insurance is found unacceptable by an underwriter due to the nature of its oper-
ations and the construction of the building it occupies. If the applicant installs an automatic fire-extinguishing sprin-
kler system, the underwriter will approve the application. This type of underwriting modification is known as

A. Amending the policy terms and conditions.

B. Using facultative reinsurance.

C. Requiring risk control measures.

D. Changing insurance rates, rating plans, or policy limits.

Question 27 B
For underwriting to achieve its purpose,
A. Underwriters must select applicants with legal hazards.
B. Insurers must minimize the effects of adverse selection on the book of business.
C. Underwriters must randomly select applicants to insure.
D. Insurers must follow the steps in the underwriting process in strict sequence.

100
Question 28 B
In most states, how many days' notice must an insured be given of an insurer's intent to cancel or nonrenew cover-
age?

A. No more than 10 days

B. At least 30 days

C. Not fewer than 45 days

D. At least 60 days

Question 29 B
Underwriting guidelines reflect the levels of

A. Written premium by type of insurance.

B. Underwriting authority.

C. Loss exposure.

D. Responsibility by job title.

Question 30 C
The law of large numbers states that as the number of similar but independent exposure units increases,

A. The relative accuracy of predictions about future outcomes decreases.

B. Divergence from the norm will be in direct proportion to the increase in size of the sample.

C. The relative accuracy of predictions about future outcomes increases.

D. Divergence from the norm will be in inverse proportion to the increase in size of the sample.

101
CHAPTER 6

Question 1 D
The value of an insurance contract in the event of a claim

A. Is purely financial.

B. Is purely remedial.

C. Includes the opportunity to profit.

D. Includes peace of mind.

Question 2 B
An employee of an insurance company who performs some or all of the insurer's claim-handling activities is called

A. An inside claim representative.

B. A staff claim representative.

C. A public adjuster.

D. A field claim adjuster.

Question 3 B
Marvin suffered a fire loss at his residence. The claim representative valued several antiques at a very low price.
Marvin refuses to settle his claim with his insurer. Which one of the following is Marvins recourse?

A. Marvins only recourse is to sue the insurer for coverage and damages to the property.

B. Marvin may follow the appraisal provision in the policy to settle the dispute over the value of the damaged prop-
erty.

C. Marvin may obtain a quote from a certified antique dealer to determine the value the insurer must pay.

D. Marvin may choose to arbitrate the value of the antiques before a judge.

Question 4 C
Which one of the following is a common alternative dispute resolution (ADR) technique.

A. Motion for summary judgment

B. Pleadings

C. Arbitration

D. Interrogatories

102
Question 5 B
If a claim is complex or if settlement negotiations are not progressing with the insurer, the insured may hire

A. An independent adjuster.

B. A public adjuster.

C. A senior claim representative.

D. A public defender.

Question 6 B
Jorgen is a claim supervisor who is devising a new response plan for his employer so that it can handle catastrophe
claims more effectively. All of the following are factors he should consider, EXCEPT:

A. Retaining independent adjuster firms

B. Requesting that personnel staff temporary claim offices without pay.

C. Increasing the role of producers through increased settlement authority

D. Giving the policyholder the benefit of any doubt as to the valuation of property

Question 7 C
One way for claim managers to help maintain an insurer's underwriting profit is to

A. Retain high-priced experts whenever possible.

B. Achieve the lowest settlements possible.

C. Establish appropriate spending policies.

D. Grant claim staff complete autonomy as to spending decisions.

Question 8 C
A goal of the claim function is to support the profit goal of the

A. Insured.

B. Claimant.

C. Insurer.

D. Policyholder.

103
Question 9 B
In a constructive total loss, the property cannot be repaired

A. For less than salvage value minus actual cash value.

B. For less than actual cash value minus anticipated salvage value.

C. For less than subrogation value minus salvage value.

D. For less than salvage value minus agreed value.

Question 10 B
An insurance contract is designed to be all of the following in the event of a claim, EXCEPT:

A. A financial mechanism that can benefit policyholders and other claimants

B. A means to provide a profit to the insured

C. A means to afford peace of mind to the insured

D. A way to restore a claimant to pre-loss condition

Question 11 C
The foundation and primary focus of good faith is

A. Training.

B. Prompt and courteous service.

C. Thorough, timely, and unbiased investigations.

D. Efficiency.

Question 12 D
People who handle claims fall into a number of categories. Those who handle claims, keep claim records, and perform
statistical analysis on behalf of self-insureds are called

A. Claim managers.

B. Independent adjusters.

C. Staff claim representatives.

D. Third-party administrators.

104
Question 13 D
Insurers handling catastrophe claims should be prepared for

A. Claimants who are not eager to assert their rights.

B. Mortgagees that are eager to abbreviate their procedures.

C. News media eager to promote aggressive behavior.

D. Claimants who are aggressive in their pursuit of settlement.

Question 14 C
One of the greatest challenges for an insurer dealing with a catastrophe is staffing. All of the following are likely re-
sponses to this challenge, EXCEPT:

A. Identifying and training staff from non-claim areas to assist.

B. Establishing relationships with independent adjusters to help manage overflow.

C. Purchasing catastrophe reinsurance.

D. Bringing in catastrophe teams of claim representatives from other regions.

Question 15 A
A liability adjuster must be concerned with specific, out-of-pocket expenses when evaluating the damages being
claimed. The technical term for such expenses is

A. Special damages.

B. Punitive damages.

C. General damages.

D. Measurable damages.

Question 16 B
An important aspect of acknowledging a claimant's bodily injury liability claim, upon first contact, is to try to obtain
information on the

A. Claimant's medical history.

B. Nature and extent of the injury.

C. Claimant's accident history.

D. Claimant's sources of additional income.

105
Question 17 D
Damages for pain and suffering are a form of

A. Out-of-pocket damages.

B. Punitive damages.

C. Special damages.

D. General damages.

Question 18 C
Bill's three-year-old stereo was completely destroyed in a covered loss. Its expected useful life was ten years. Its re-
placement cost is $600. Its actual cash value (ACV) is

A. $ 60.

B. $180.

C. $420.

D. $600.

Question 19 C
People who handle claims may be staff claim representatives, independent adjusters, employees of third-party admin-
istrators, or

A. Private claims representatives.

B. Self-insurer claim representatives.

C. Producers who sell policies to insureds.

D. Special claims representatives.

Question 20 A
Actual cash value (ACV) is

A. Replacement cost minus depreciation.

B. Agreed value minus depreciation.

C. Replacement cost plus depreciation.

D. Replacement cost minus agreed value.

106
Question 21 D
The most typical third-party bad-faith claim alleges

A. Breach of contract.

B. Bad-faith claims made against the first-party policyholder relating to coverage issues.

C. Failure of a claim representative to divulge details of a loss investigation that would lead to an increased claim pay-
ment to the insured.

D. An improper failure to settle a third party's liability claim within policy limits followed by a verdict that exceeds the
policy limits.

Question 22 D
Loss reserves are funds held by an insurer to pay claims for losses that have occurred but have not yet been settled.
Such funds assigned to an individual claim create a

A. Damage reserve.

B. Suit reserve.

C. Claim reserve.

D. Case reserve.

Question 23 B
The insurers rights to recover and sell or otherwise dispose of insured property on which the insurer has paid a total
loss are called

A. Fiduciary rights.

B. Salvage rights.

C. Constructive rights.

D. Catastrophe rights.

Question 24 C
In the claim evaluation process, specific, out-of-pocket expenses are called

A. Compensatory damages.

B. General damages.

C. Special damages.

D. Negotiated damages.

107
Question 25 D
In order to expedite claim handling during a catastrophe, an insurer will consider all of the following modifications to
procedures, EXCEPT:

A. Temporarily increasing claim handling authority for producers

B. Making advance payments to policyholders

C. Using abbreviated claim handling procedures

D. Suspending the use of independent adjusting firms

Question 26 C
Which one of the following statements is true regarding laws prohibiting unfair claims practices?

A. The laws apply only to actions by the insurer and not by individual claim representatives.

B. The federal government has enacted an unfair claims practices law to which all states must adhere, unless their law
is more stringent.

C. Each state that has such a law will usually list insurer or claim representative practices that are prohibited.

D. The laws specifically list acceptable actions or processes that insurers may take.

Question 27 D
All of the following correctly describe reasons for the use of independent adjusters, EXCEPT:

A. Insurers may use independent adjusters when special skills and expertise are needed, for example to investigate
aircraft accidents.

B. In the case of a catastrophic loss such as a hurricane, an insurer may not have sufficient staff to manage the large
number of claims, and may use independent adjusters.

C. In some areas it is not economically feasible to set up claim offices, and insurers may contract with independent
adjusters to handle claims in these remote areas.

D. The insured may retain an independent adjuster if settlement negotiations with the insurer are not progressing sat-
isfactorily.

Question 28 B
The best description of a first-party claim is a claim
A. Made by the liable party against the policyholder.

B. Made by the policyholder against the insurer.

C. Made by the insurer against the policyholder.

D. Made by the policyholder against someone to whom the policyholder may be liable.

108
Question 29 C
If a claim representative's investigation determines that no coverage applies for a liability claim, the insurer will

A. Pay for the defense but not the settlement of the claim.

B. Settle the claim and then seek reimbursement from the insured.

C. Deny the claim.

D. Try to quickly settle the claim.

Question 30 B
Claim representatives should understand the subjective interpretation of negligence and gross misconduct. The differ-
ence between negligence and gross misconduct is determined by

A. The actual intentions of the claim representative.

B. The court's or jury's interpretation of the facts.

C. Federal law.

D. The claim representative's exercised standard of care.

Question 1 C
Property insurers are subject to the possibility of many claims occurring from a single event and straining their re-
sources. This type of event is called

A. An accident.

B. An occurrence.

C. A catastrophe.

D. A simulation.

Question 2 C
People who handle claims may be staff claim representatives, independent adjusters, employees of third-party admin-
istrators, or

A. Private claims representatives.

B. Self-insurer claim representatives.

C. Producers who sell policies to insureds.

D. Special claims representatives.

109
Question 3 A
The adequacy of the case reserve for a complicated claim should be evaluated

A. As new information is obtained.

B. At year's end.

C. On a weekly basis.

D. When the claimant submits a settlement offer.

Question 4 D
A person or entity that would suffer a financial loss if property were damaged has

A. A contractual interest.

B. A discretionary interest.

C. A renewable interest.

D. An insurable interest.

Question 5 D
Insurers handling catastrophe claims should be prepared for

A. Claimants who are not eager to assert their rights.

B. Mortgagees that are eager to abbreviate their procedures.

C. News media eager to promote aggressive behavior.

D. Claimants who are aggressive in their pursuit of settlement.

Question 6 A
A liability loss has the following damages:

Medical expenses:
$ 10,000
Damages for disfigurement:
$ 50,000

Damages for pain and suffering:


$ 100,000

What is the amount of special damages?


A. $ 10,000
B. $ 50,000
C. $100,000
D. $160,000

110
Question 7 C
Which one of the following is a negotiation process in which a neutral outside party helps participants examine the
issues and develop a mutually agreeable settlement?

A. Appraisal

B. Arbitration

C. Mediation

D. Mini-trial

Question 8 B
An insurance contract is designed to be all of the following in the event of a claim, EXCEPT:

A. A financial mechanism that can benefit policyholders and other claimants

B. A means to provide a profit to the insured

C. A means to afford peace of mind to the insured

D. A way to restore a claimant to pre-loss condition

Question 9 D
Most insurers give some producers the authority to pay claims up to a certain amount. In this role, producers function
much like an

A. Outside claim representative.

B. Independent adjuster.

C. Insurance appraiser.

D. Inside claim representative.

Question 10 D
Which one of the following is an example of general damages?

A. Hospital expenses

B. Lost wages

C. Prescription costs

D. Compensation for disfigurement

111
Question 11 B
An insurance company employee who investigates the scene of a loss; meets with insureds, claimants, lawyers, and
others; and inspects damage is most accurately called

A. A third-party administrator.

B. A field claim representative.

C. An independent adjuster.

D. An adjusting agent.

Question 12 A
Work on a new claim begins for the claim representative upon receipt of the notice of loss. The representative obtains
initial information and verifies coverage. Next, the representative should

A. Set a loss reserve.

B. Make an offer to settle the claim.

C. Retain causation experts.

D. Assign the claim to defense counsel.

Question 13 C
Because insurers control how claims are resolved, courts reason that insurers should be responsible for the outcome of
their claim handling if they have acted in bad faith. Therefore, courts hold insurers to a higher standard of conduct to

A. Encourage insurers to be equitable in all insurance transactions.

B. Encourage insurers to keep rates accurate, fair, and reasonable.

C. Discourage insurers from abusing their position of power.

D. Discourage insurers from deceiving the public in marketing and advertising practices.

Question 14 D
All of the following explain when the use of independent adjusters might be advantageous, EXCEPT:

A. Insurers may use independent adjusters for specific types of losses, for example, aircraft accidents, in which special-
ized expertise is required to settle the claim.
B. In the case of a catastrophic loss such as a hurricane, an insurer may not have sufficient staff to manage the claim
volume and may temporarily depend upon independent adjusters to handle the excess.
C. In some areas it is not economically feasible to set up remote claim offices and in these areas insurers may depend
on independent adjusters to handle their insureds' losses.
D. The insured may retain an independent adjuster after becoming dissatisfied with the pace of negotiations with the
insurer.

112
Question 15 C
Bill's three-year-old stereo was completely destroyed in a covered loss. Its expected useful life was ten years. Its re-
placement cost is $600. Its actual cash value (ACV) is

A. $ 60.

B. $180.

C. $420.

D. $600.

Question 16 C
In the claim evaluation process, specific, out-of-pocket expenses are called

A. Compensatory damages.

B. General damages.

C. Special damages.

D. Negotiated damages.

Question 17 B
Marvin suffered a fire loss at his residence. The claim representative valued several antiques at a very low price.
Marvin refuses to settle his claim with his insurer. Which one of the following is Marvins recourse?

A. Marvins only recourse is to sue the insurer for coverage and damages to the property.

B. Marvin may follow the appraisal provision in the policy to settle the dispute over the value of the damaged prop-
erty.

C. Marvin may obtain a quote from a certified antique dealer to determine the value the insurer must pay.

D. Marvin may choose to arbitrate the value of the antiques before a judge.

Question 18 D
All of the following correctly describe reasons for the use of independent adjusters, EXCEPT:

A. Insurers may use independent adjusters when special skills and expertise are needed, for example to investigate
aircraft accidents.
B. In the case of a catastrophic loss such as a hurricane, an insurer may not have sufficient staff to manage the large
number of claims, and may use independent adjusters.
C. In some areas it is not economically feasible to set up claim offices, and insurers may contract with independent
adjusters to handle claims in these remote areas.
D. The insured may retain an independent adjuster if settlement negotiations with the insurer are not progressing sat-
isfactorily.

113
Question 19 C
One of the greatest challenges for an insurer dealing with a catastrophe is staffing. All of the following are likely re-
sponses to this challenge, EXCEPT:

A. Identifying and training staff from non-claim areas to assist.

B. Establishing relationships with independent adjusters to help manage overflow.

C. Purchasing catastrophe reinsurance.

D. Bringing in catastrophe teams of claim representatives from other regions.

Question 20 D
The best description of a third-party claim is a claim

A. Made by the liable party against the policyholder.

B. Made by the policyholder against the insurer.

C. Made by the insurer against the policyholder.

D. Made against the policyholder by someone to whom the policyholder may be liable.

Question 21 C
If an insureds house burns to the ground due to faulty wiring, the most difficult part of the claim handling process will
probably be

A. Determining the cause of loss.

B. Verifying coverage.

C. Determining the amount of damages.

D. Determining insurable interest.

Question 22 A
Which one of the following statements about claim evaluation is correct?

A. Prompt evaluation helps to ensure compliance with time limits specified by unfair claims settlement practices acts.

B. The limit of liability should be considered when determining the value of a liability claim.

C. The claim representative's opinion of the value of a claim is the most important evaluation factor.

D. Evaluation of a liability claim should be made after the claimant makes his or her first settlement demand.

114
Question 23 B
Jorgen is a claim supervisor who is devising a new response plan for his employer so that it can handle catastrophe
claims more effectively. All of the following are factors he should consider, EXCEPT:

A. Retaining independent adjuster firms

B. Requesting that personnel staff temporary claim offices without pay.

C. Increasing the role of producers through increased settlement authority

D. Giving the policyholder the benefit of any doubt as to the valuation of property

Question 24 B
All of the following are correct with respect to fair evaluation of a claim, EXCEPT:

A. A fair approach to evaluating liability claims is to evaluate them as if no coverage limit existed.

B. Claim representatives are prohibited from discussing the evaluation of a claim with anyone not directly connected
with that claim.

C. Legislation often specifies a time limit within which evaluations of coverage and damages must be completed.

D. Information about settlements or trial results from similar cases can help claim representatives knowledgably eval-
uate a claim.

Question 25 C
One way for claim managers to help maintain an insurer's underwriting profit is to

A. Retain high-priced experts whenever possible.

B. Achieve the lowest settlements possible.

C. Establish appropriate spending policies.

D. Grant claim staff complete autonomy as to spending decisions.

Question 26 C
A goal of the claim function is to support the profit goal of the

A. Insured.

B. Claimant.

C. Insurer.

D. Policyholder.

115
Question 27 D
Lou is a claim manager at a small property-casualty insurance company. Lou takes pride in the fact that because of his
leadership, his company has gained a public reputation for resisting legitimate claims. Ultimately, this reputation

A. Satisfies the mandate of regulators.

B. Makes the company's policies more marketable.

C. Supports his company's profit goals.

D. Undermines the effectiveness of his company's advertising.

Question 28 B
Adjuster Toni is evaluating damages in a bodily injury liability claim where she has determined that the insured is
completely at fault for the accident. The claimant suffered a sprain and strain injury to his neck. His special damages
total $1,500. He was out of work for a week, was disabled and prevented from performing some regular activities for
a month, and attended physical therapy for six weeks. In assessing general damages, Toni should

A. Multiply the amount of special damages of $1,500 by three as a rule of thumb, and pay $4,500.

B. Use past cases which are similar as a guide.

C. Consider what she would want to be paid if she were in claimant's shoes.

D. Multiply $1,500, the amount of special damages, by six, the number of weeks of physical therapy, and pay $9,000.

Question 29 C
Which one of the following would have an insurable interest in real property?

A. Bank that has provided a personal credit card to the owner of the property

B. Former owner of the property

C. Mortgagee that has provided a mortgage loan on the property

D. Someone expressing interest in purchasing the property

Question 30 D
Claim management involves many duties crucial to good-faith claim handling such as consistent supervision, thorough
training, and

A. Expense management.

B. Organizational skills.

C. Customer service soft skills.

116
D. Manageable caseloads.

Question 1 C
A goal of the claim function is to support the profit goal of the

A. Insured.

B. Claimant.

C. Insurer.

D. Policyholder.

Question 2 A
A liability loss has the following damages:

Medical expenses:
$ 10,000
Damages for disfigurement:
$ 50,000

Damages for pain and suffering:


$ 100,000

What is the amount of special damages?

A. $ 10,000

B. $ 50,000

C. $100,000

D. $160,000

Question 3 D
Despite the unique challenges and variations from case to case, the last step in the claim handling process is usually

A. Determining cause of loss.

B. Determining the amount of damages.

C. Verifying coverages.

D. Negotiating and settling.

117
Question 4 C
People who handle claims may be staff claim representatives, independent adjusters, employees of third-party admin-
istrators, or

A. Private claims representatives.

B. Self-insurer claim representatives.

C. Producers who sell policies to insureds.

D. Special claims representatives.

Question 5 B
Worthley Insurance Company uses claim audits to ensure compliance with best practices. These audits evaluate both
quantitative and qualitative factors. Which one of the following factors that Worthley evaluates is a qualitative fac-
tor?

A. Timeliness of reserving

B. Accurate evaluation of insureds liability

C. Average claim settlement value by claim type

D. Accuracy of data entry

Question 6 C
Compensatory damages represent

A. Out-of-pocket expenses.

B. Non-economic loss.

C. Actual harm suffered.

D. Damages to deter others from committing wrongful acts.

Question 7 C
One way for claim managers to help maintain an insurer's underwriting profit is to

A. Retain high-priced experts whenever possible.

B. Achieve the lowest settlements possible.

C. Establish appropriate spending policies.

D. Grant claim staff complete autonomy as to spending decisions.

118
Question 8 C
If an insureds house burns to the ground due to faulty wiring, the most difficult part of the claim handling process will
probably be

A. Determining the cause of loss.

B. Verifying coverage.

C. Determining the amount of damages.

D. Determining insurable interest.

Question 9 B
Adjuster Toni is evaluating damages in a bodily injury liability claim where she has determined that the insured is
completely at fault for the accident. The claimant suffered a sprain and strain injury to his neck. His special damages
total $1,500. He was out of work for a week, was disabled and prevented from performing some regular activities for
a month, and attended physical therapy for six weeks. In assessing general damages, Toni should

A. Multiply the amount of special damages of $1,500 by three as a rule of thumb, and pay $4,500.

B. Use past cases which are similar as a guide.

C. Consider what she would want to be paid if she were in claimant's shoes.

D. Multiply $1,500, the amount of special damages, by six, the number of weeks of physical therapy, and pay $9,000.

Question 10 D
Lou is a claim manager at a small property-casualty insurance company. Lou takes pride in the fact that because of his
leadership, his company has gained a public reputation for resisting legitimate claims. Ultimately, this reputation

A. Satisfies the mandate of regulators.

B. Makes the company's policies more marketable.

C. Supports his company's profit goals.

D. Undermines the effectiveness of his company's advertising.

Question 11 C
In the claim evaluation process, specific, out-of-pocket expenses are called

A. Compensatory damages.

B. General damages.

C. Special damages.

D. Negotiated damages.

119
Question 12 C
One of the greatest challenges for an insurer dealing with a catastrophe is staffing. All of the following are likely re-
sponses to this challenge, EXCEPT:

A. Identifying and training staff from non-claim areas to assist.

B. Establishing relationships with independent adjusters to help manage overflow.

C. Purchasing catastrophe reinsurance.

D. Bringing in catastrophe teams of claim representatives from other regions.

Question 13 B
An insurance contract is designed to be all of the following in the event of a claim, EXCEPT:

A. A financial mechanism that can benefit policyholders and other claimants

B. A means to provide a profit to the insured

C. A means to afford peace of mind to the insured

D. A way to restore a claimant to pre-loss condition

Question 14 A
The insured's washing machine was damaged by a covered loss. At the time of the loss, the actual cash value (ACV) of
the machine was $300. Repairs would cost $250. The scrap metal dealer would pay $100 for what remains of it. Which
one of the following calculations correctly determines whether or not the washing machine was a constructive total
loss?

A. $300 ACV - $100 salvage value = $200. This loss was a constructive total loss.

B. $300 ACV - $100 salvage value = $200. This loss was not a constructive total loss.

C. $300 ACV - $250 repairs = $50. This loss was a constructive total loss.

D. $300 ACV - $250 repairs = $50. This loss was not a constructive total loss.

Question 15 B
Fair evaluation of claims is an element of good-faith claim handling. All of the following are true regarding fair evalu-
ation of a claim, EXCEPT:
A. A fair approach to evaluating liability claims is to evaluate them as if no coverage limit existed.
B. Claim representatives are prohibited from discussing the evaluation of a claim with anyone not directly connected
with that claim.
C. Unfair claims settlement practices acts often specify a time limit within which evaluations of coverage and damages
must be completed.
D. Information about settlements or trial results from similar cases can help claim representatives knowledgably eval-
uate a claim.

120
Question 16 A
Defending liability claims through litigation can be a very long and expensive process. Mediation and arbitration are
two common forums for settling claims outside the traditional court system. What is the difference between arbitra-
tion and mediation?

A. Someone other than the insurer and the claimant decides the case in arbitration.

B. The arbitrators decision is always binding.

C. Only one insured and one insurer can be involved in arbitration.

D. Solutions are proposed in arbitration to help the parties settle.

Question 17 B
An important aspect of acknowledging a claimant's bodily injury liability claim, upon first contact, is to try to obtain
information on the

A. Claimant's medical history.

B. Nature and extent of the injury.

C. Claimant's accident history.

D. Claimant's sources of additional income.

Question 18 B
Jorgen is a claim supervisor who is devising a new response plan for his employer so that it can handle catastrophe
claims more effectively. All of the following are factors he should consider, EXCEPT:

A. Retaining independent adjuster firms

B. Requesting that personnel staff temporary claim offices without pay.

C. Increasing the role of producers through increased settlement authority

D. Giving the policyholder the benefit of any doubt as to the valuation of property

Question 19 B
Incurred losses (including loss adjustment expenses) divided by earned premium is the formula for

A. Combined ratio.

B. Loss ratio.

C. Expense ratio.

D. Underwriting profit ratio.

121
Question 20 C
Which one of the following allegations in insurance claims handling result from criticisms of claim representatives'
actions, sometimes regardless of validity?

A. Use of improper valuation techniques allegations

B. Unfair rating practices allegations

C. Bad faith allegations

D. Contract or policy manipulation allegations

Question 21 C
Because insurers control how claims are resolved, courts reason that insurers should be responsible for the outcome of
their claim handling if they have acted in bad faith. Therefore, courts hold insurers to a higher standard of conduct to

A. Encourage insurers to be equitable in all insurance transactions.

B. Encourage insurers to keep rates accurate, fair, and reasonable.

C. Discourage insurers from abusing their position of power.

D. Discourage insurers from deceiving the public in marketing and advertising practices.

Question 22 C
Jerry is an experienced claim adjuster who handles claims for Jackson Insurance Company during hurricane season
when Jacksons staff adjusters need assistance. Jerry is

A. A public adjuster.

B. A third-party adjuster.

C. An independent adjuster.

D. A hurricane adjuster.

Question 23 D
Insurers handling catastrophe claims should be prepared for

A. Claimants who are not eager to assert their rights.

B. Mortgagees that are eager to abbreviate their procedures.

C. News media eager to promote aggressive behavior.

D. Claimants who are aggressive in their pursuit of settlement.

122
Question 24 A
One aspect of good-faith claim handling is the fair evaluation of the claim. A fair approach to evaluating liability
claims is for the claim representative to evaluate them

A. As if no coverage limit existed.

B. From the claimant's perspective.

C. As they relate to similar claims with similar coverage and limits.

D. By forming a claim committee for the purpose of achieving greater impartiality.

Question 25 D
A reservation of rights letter

A. Lists all of the rights that an insurer has under an insurance policy.

B. Advises the insurer that the insured has made a claim, but retains the right to withdraw the claim later.

C. Lists all of the rights that an insured has under an insurance policy.

D. Advises the insured that the insurer is investigating the claim but retains the right to deny coverage later.

Question 26 D
Most insurers give some producers the authority to pay claims up to a certain amount. In this role, producers function
much like an

A. Outside claim representative.

B. Independent adjuster.

C. Insurance appraiser.

D. Inside claim representative.

Question 27 A
All of the following are factors in determining the replacement cost of real property, EXCEPT:

A. Actual cash value of the property

B. Construction cost per square foot

C. Square footage of the property

D. Type and quality of the construction

123
Question 28 C
Property insurers are subject to the possibility of many claims occurring from a single event and straining their re-
sources. This type of event is called

A. An accident.

B. An occurrence.

C. A catastrophe.

D. A simulation.

Question 29 D
Mira raised chickens in a coop that she constructed in her backyard. Due to her careless smoking, the coop was de-
stroyed by fire. She submitted a claim to her homeowners insurer, which assigned an adjuster. This is the adjuster's
first claim involving a building detached from a dwelling. Assuming the adjuster has established Mira's insurable in-
terest in the property, which one of the following should the adjuster be asking next?

A. Was the chicken coop constructed within the standards of the local building code?

B. How old is the chicken coop?

C. How many chickens died in the fire?

D. Is the damaged property covered by the policy?

Question 30 D
The best description of a third-party claim is a claim

A. Made by the liable party against the policyholder.

B. Made by the policyholder against the insurer.

C. Made by the insurer against the policyholder.

D. Made against the policyholder by someone to whom the policyholder may be liable.

124
CHAPTER 7

Question 1 A
When implementing the selected risk management techniques, the risk management program must include

A. A communications plan.

B. A mission statement.

C. A loss analysis.

D. An analysis of expected results.

Question 2 D
Which one of the following best describes how effective risk management benefits insurers?

A. Stimulating economic growth

B. Causing fewer disruption in the social environment

C. Increased ability to accurately predict future losses

D. Encourages insurers to create innovative products and offer competitive prices

Question 3 C
Jeff recently started a consulting business. One of his concerns is that he will be sued for giving erroneous advice to a
client. Which one of the following would most likely be the best risk management technique for Jeffs use in this situa-
tion?

A. Retention

B. Avoidance

C. Insurance

D. Duplication

Question 4 C
Traditionally, the risk management professional's role has been associated with loss exposures related to

A. Business risk.

B. Operational risk.

C. Pure risk.

D. Speculative risk.

125
Question 5 C
Properly estimating loss severity is essential in treating a loss exposure because the potential severity is a major con-
sideration in determining whether to

A. Use risk control or risk financing.

B. Use loss avoidance or loss prevention.

C. Insure or retain a particular loss exposure.

D. Physically inspect or rely on a loss exposure survey.

Question 6 B
Which one of the following identifies a benefit that a business can receive by applying sound risk management?

A. It will meet state and federal safety regulations.

B. It will have a better opportunity to achieve business goals.

C. It will not have to worry about losses.

D. It will be able to attract and retain talented employees and managers.

Question 7 D
It is easier to gauge the potential severity of property losses than of liability losses because property loss exposures

A. Are confined to the building and contents.

B. Can be determined as an average according to the type of business.

C. Have a calculable frequency.

D. Typically have a finite value.

Question 8 D
Which one of the following is true regarding enterprise-wide risk management (ERM)?

A. ERM is an approach to risk management that focuses primarily on loss exposures associated with pure risk.

B. In practice, implementation of ERM occurs at the departmental or business unit level.

C. Implementation of ERM is fairly consistent among organizations, regardless of their size, nature, or complexity.

D. ERM is an approach to managing all of an organization's key risks and opportunities.

126
Question 9 C
Which one of the following best describes how effective risk management benefits society?

A. Creating a positive effect on an insurers underwriting results

B. Providing more thoughtful consumers of insurance

C. Causing fewer disruptions in the economic and social environment

D. Increasing the types of charitable and governmental agencies available to the general public

Question 10 A
In order to monitor and modify the risk management program, the risk manager must periodically

A. Identify and analyze new and existing loss exposures.

B. Purchase insurance.

C. Rewrite the risk management mission statement.

D. Change insurers.

Question 11 C
Katie and Kevin have a small stream in their back yard. Last year was exceptionally rainy and they were concerned
about flood exposures to their home. They recently had a contractor regrade the back yard, building a large berm,
which provides a raised barrier to divert the flow of water away from the house. In this situation, Katie and Kevin are
using which one of the following risk management techniques?

A. Separation

B. Avoidance

C. Loss prevention

D. Loss reduction

Question 12 C
Grocers' Warehouse is a newly created business that will open next month. It will include a retail operation as well as
500,000 square feet of warehouse space with loading docks. Which one of the following methods of identifying loss
exposures would be least applicable in this situation?

A. Physical inspection

B. Loss exposure survey

C. Loss histories

D. Interviews with management

127
Question 13 C
A physical inspection can help to identify loss exposures that are not identified through other means because it

A. Is based on direct information from others.

B. Is required in any sound physical fitness program.

C. Provides the opportunity to see things firsthand.

D. Requires the use of a thorough checklist.

Question 14 C
Analyzing the potential frequency and severity of a loss exposure enables the risk manager to

A. Retain loss exposures.

B. Avoid loss exposures.

C. Prioritize loss exposures.

D. Transfer loss exposures.

Question 15 D
A business installs a sprinkler system to reduce the amount of fire damage from potential fires. This is an example of

A. Avoidance.

B. Separation.

C. Loss prevention.

D. Loss reduction.

Question 16 A
Businesses, individuals, and families that practice sound risk management can benefit society in all of the following
ways, EXCEPT:

A. Increasing interest in leisure activities

B. Reducing the overall number of losses

C. Controlling medical expenses through reduced injuries

D. Stimulating economic growth

128
Question 17 D
Sho Ching is risk manager for Market Sales Company. Market Sales owns a large fleet of autos used by the sales em-
ployees. The fleet is insured with $1,000 physical damage deductibles. Sho Ching is concerned about an increasing fre-
quency of auto accidents in recent years. Which one of the following is the best risk management option for address-
ing the increased frequency of accidents from the fleet of autos?

A. Decrease the deductible to $500

B. Increase the deductible to $2,000

C. Implement loss reduction programs

D. Implement loss prevention programs

Question 18 B
Risk management activities under the enterprise-wide risk management approach occur at the

A. Business unit level.

B. Corporate level.

C. Departmental level.

D. Regional level.

Question 19 B
Helen, the risk manager of a supermarket, has decided that the store needs a sprinkler system. She needs to check on
the local water supply and building permits and decide what is necessary to comply with local ordinances as well as
decide how much the supermarket can afford to spend on the system. Helen is making these decisions as part of

A. Examining the feasibility of the risk management technique.

B. Implementing the selected risk management technique.

C. Analyzing loss exposures.

D. Selecting the appropriate risk management technique.

Question 20 C
A taxi firm maintains a few spare vehicles to keep all drivers on the road even if one vehicle needs repair. Which one
of the following identifies the risk management technique being used in this example?

A. Diversification

B. Avoidance

C. Duplication

D. Separation

129
Question 21 D
Which one of the following is the goal of enterprise-wide risk management (ERM)?

A. Coordinate loss reduction efforts

B. Reduce risk management costs

C. Decentralize control of business decisions

D. Maximize the organizations value

Question 22 C
A business installs bars on windows and door deadbolts to prevent burglaries. This is an example of

A. Avoidance.

B. Separation.

C. Loss prevention.

D. Loss reduction.

Question 23 B
Randy and Ida are concerned that a buried heating oil tank in their yard might be leaking, but they have decided that
replacing that tank must wait until they have the funds in a year or two. Randy and Ida have discovered that pollution
resulting from a leak would not be covered by their homeowners insurance policy. They fear the oil might seep into
the water table and contaminate their neighbors' well water. Illness and damage that might result could be very ex-
pensive. Randy and Ida can purchase an endorsement that will provide coverage for this pollution exposure. The en-
dorsement would cost $50 per year. Is this an effective risk management selection for Randy and Ida until they can
replace the tank?

A. Yes, they should do this indefinitely and not replace the tank.

B. Yes, they are exchanging a large exposure for a little premium.

C. No, they are spending a lot of money for little protection.

D. No, each neighbor will have insurance to cover any damage to their wells.

Question 24 B
John Young is a risk manager of a medium-size business with both auto and real property exposures. John is looking
for ways to save money on insurance premiums. John should consider
A. Retaining the auto liability loss exposures.
B. Retaining some or all of the auto physical damage exposures.
C. Canceling the insurance and implementing noninsurance transfer techniques.
D. Retaining the property loss exposures.

130
Question 25 D
When implementing selected risk management techniques, a risk manager must

A. Base decisions on financial criteria.

B. Base decisions on informal guidelines.

C. Analyze the benefits of the risk management program.

D. Determine how to allocate the costs of the program.

Question 26 D
Frank and Jan recently married and purchased their first home together. What is an effective way for Frank and Jan to
begin identifying the liability loss exposures associated with their new home?

A. Hire a professional risk manager

B. Determine the value of the home and contents

C. Purchase a homeowners insurance policy

D. Inspect the home for loss exposures

Question 27 C
When selecting the appropriate risk management techniques, financial management decisions are typically made
with the objective of

A. Decreasing the worry factor.

B. Increasing the cost of risk.

C. Increasing profits.

D. Decreasing expenses.

Question 28 B
Estimating how large the losses may be and how often they may occur is required under which one of the steps of the
risk management process?

A. Identifying loss exposures

B. Analyzing loss exposures

C. Examining the feasibility of risk management techniques

D. Monitoring results

131
Question 29 A
Which one of the following describes a benefit to businesses of making insurance part of an overall risk management
program instead of relying solely on insurance?

A. Improved access to affordable insurance

B. Stimulating economic growth

C. Reducing the number of persons dependent on society for support

D. Increased use of exposure avoidance

Question 30 B
Retention of losses is

A. More expensive than insurance in the long run.

B. Less expensive than insurance in the long run.

C. More effective than insurance in handling loss frequency problems.

D. Very similar to noninsurance transfer.

Question 1 C
Traditionally, the risk management professional's role has been associated with loss exposures related to

A. Business risk.

B. Operational risk.

C. Pure risk.

D. Speculative risk.

Question 2 A
Outdoor Designs Company (ODC) makes cedar patio furniture. ODC's lumber supplier, L&L Wood Products, is the larg-
est and most competitive supplier of lumber in the area. ODC's risk manager is concerned that a disruption in the sup-
ply of lumber from L&L due to a large fire loss would adversely affect its production. Which one of the following would
best minimize the adverse effect to ODC of a large loss experienced by L&L?

A. Separation

B. Duplication

C. Loss prevention

D. Avoidance

132
Question 3 C
Waking up on a cold February morning, Amy discovers the roads are icy and snow covered. Concerned about driving
to work and possibly having an accident, she decides to take the day off. Amys decision is an example of which one of
the following risk management techniques?

A. Loss prevention

B. Loss reduction

C. Avoidance

D. Separation

Question 4 D
Which one of the following is the goal of enterprise-wide risk management (ERM)?

A. Coordinate loss reduction efforts

B. Reduce risk management costs

C. Decentralize control of business decisions

D. Maximize the organizations value

Question 5 B
Risk management activities under the enterprise-wide risk management approach occur at the

A. Business unit level.

B. Corporate level.

C. Departmental level.

D. Regional level.

Question 6 B
Dudley drives to work in the city each day and until recently, parked on a street near his office. Dudley began parking
in a lot that has an attendant to reduce the chance of his vehicle being damaged while it is parked. Which one of the
following risk management techniques is Dudley applying?

A. Avoidance

B. Loss prevention

C. Noninsurance transfer

D. Retention

133
Question 7 C
Some businesses require key executives to fly on different flights. This is an example of which one of the following risk
control techniques?

A. Avoidance

B. Loss prevention

C. Separation

D. Duplication

Question 8 C
Analyzing the potential frequency and severity of a loss exposure enables the risk manager to

A. Retain loss exposures.

B. Avoid loss exposures.

C. Prioritize loss exposures.

D. Transfer loss exposures.

Question 9 C
Alan Peachtree owns a hobby shop, which he runs from a detached garage on his property. Alan has set aside funds to
pay for possible property losses rather than purchasing insurance. Which one of the following risk management tech-
niques is Alan using?

A. Avoidance

B. Non-Insurance transfer

C. Retention

D. Loss control

Question 10 D
Which one of the following is true regarding loss histories?

A. Changing environments have little effect on the quality of loss histories.

B. Changes in an organizations operations have little effect on the quality of loss histories.

C. Loss histories are not commonly used to identify loss exposures.

D. The quality of loss histories depends on whether they are organized and consistent.

134
Question 11 A
Businesses, individuals, and families that practice sound risk management can benefit society in all of the following
ways, EXCEPT:

A. Increasing interest in leisure activities

B. Reducing the overall number of losses

C. Controlling medical expenses through reduced injuries

D. Stimulating economic growth

Question 12 D
In implementing risk management techniques, the risk manager usually

A. Is responsible for communicating guidelines but is otherwise not involved in implementing techniques.

B. Relies on consultants outside the organization to implement the techniques.

C. Has complete authority to make decisions and implement the techniques.

D. Depends on others to implement the techniques based on the risk managers advice.

Question 13 D
Which one of the following best describes how effective risk management benefits insurers?

A. Stimulating economic growth

B. Causing fewer disruption in the social environment

C. Increased ability to accurately predict future losses

D. Encourages insurers to create innovative products and offer competitive prices

Question 14 D
Which one of the following is true regarding enterprise-wide risk management (ERM)?

A. ERM is an approach to risk management that focuses primarily on loss exposures associated with pure risk.

B. In practice, implementation of ERM occurs at the departmental or business unit level.

C. Implementation of ERM is fairly consistent among organizations, regardless of their size, nature, or complexity.

D. ERM is an approach to managing all of an organization's key risks and opportunities.

135
Question 15 B
Loss exposure surveys or checklists are comprehensive and apply to almost any organization. The surveys major
weakness is that they

A. Are expensive.

B. May omit an important exposure.

C. Have to be custom designed for each business.

D. Take too long to complete.

Question 16 C
There have been several recent burglaries in Sallys neighborhood. Which one of the following best describes a risk
management technique and a corresponding example of that technique that Sally should consider due to the recent
increase in burglaries?

A. Separation, such as installing a new alarm system

B. Loss reduction, such as keeping doors and windows locked

C. Loss prevention, such as keeping doors and windows locked

D. Loss prevention, such as installing a safe in her home

Question 17 A
Which one of the following statements describes the monitoring and revising step in the risk management process?

A. Check to make sure the decisions made are still valid, and make changes as needed.

B. Create a new workflow and identify new bottlenecks that have occurred.

C. Make sure insurance is not being used as a substitute for loss control.

D. Identify noninsurance transfers through hold-harmless agreements.

Question 18 A
Barton Industries keeps copies of key documents stored at a second location. The risk control technique Barton Indus-
tries is using is

A. Duplication.

B. Separation.

C. Loss prevention.

D. Avoidance.

136
Question 19 B
Individuals and families benefit from effective risk management in which one of the following ways?

A. Increasing their personal cash flows by retaining rather than insuring their property exposures

B. Continuing activities following an accident or other loss, and thus reducing inconvenience.

C. Stimulating economic growth because fewer losses mean that more funds are available for other uses

D. Creating a positive effect on an insurers underwriting results

Question 20 B
Helen, the risk manager of a supermarket, has decided that the store needs a sprinkler system. She needs to check on
the local water supply and building permits and decide what is necessary to comply with local ordinances as well as
decide how much the supermarket can afford to spend on the system. Helen is making these decisions as part of

A. Examining the feasibility of the risk management technique.

B. Implementing the selected risk management technique.

C. Analyzing loss exposures.

D. Selecting the appropriate risk management technique.

Question 21 C
Which one of the following is true regarding loss exposure surveys?

A. Households and organizations are likely to face all of the loss exposures detailed in surveys.

B. Loss exposures for unique operations are usually identified on surveys.

C. Surveys usually group questions on similar exposures together.

D. Risk managers can depend solely on a comprehensive survey to identify loss exposures.

Question 22 A
In order to monitor and modify the risk management program, the risk manager must periodically

A. Identify and analyze new and existing loss exposures.

B. Purchase insurance.

C. Rewrite the risk management mission statement.

D. Change insurers.

137
Question 23 A
Malvern Imports has several retail stores in a three-state area. They receive and store imports in a large central ware-
house. The risk manager is considering ways to lower the severity exposure due to a fire loss to the warehouse. Which
one of the following would most effectively decrease the severity exposure of fire loss to the warehouse?

A. Separation

B. Duplication

C. Loss prevention

D. Avoidance

Question 24 A
Financial statement analysis, loss history analysis, and flowcharts are used in

A. Identifying loss exposures.

B. Analyzing loss exposures.

C. Selecting risk management techniques.

D. Implementing risk management techniques.

Question 25 D
An accurate measure of loss frequency is important because

A. Loss frequency is a major consideration in determining whether a flowchart is required.

B. Loss frequency is a major consideration in determining whether a physical inspection is necessary.

C. If loss frequency has been increasing, loss reduction methods will likely need to be implemented.

D. The proper treatment of the loss exposure often depends on how frequently the loss is expected to occur.

Question 26 B
Randy and Ida are concerned that a buried heating oil tank in their yard might be leaking, but they have decided that
replacing that tank must wait until they have the funds in a year or two. Randy and Ida have discovered that pollution
resulting from a leak would not be covered by their homeowners insurance policy. They fear the oil might seep into
the water table and contaminate their neighbors' well water. Illness and damage that might result could be very ex-
pensive. Randy and Ida can purchase an endorsement that will provide coverage for this pollution exposure. The en-
dorsement would cost $50 per year. Is this an effective risk management selection for Randy and Ida until they can
replace the tank?
A. Yes, they should do this indefinitely and not replace the tank.

B. Yes, they are exchanging a large exposure for a little premium.

C. No, they are spending a lot of money for little protection.

138
D. No, each neighbor will have insurance to cover any damage to their wells.

Question 27 B
Sound risk management benefits society in each of the following ways, EXCEPT:

A. Causing fewer disruptions in the economic and social environment

B. Increasing opportunity costs

C. Reducing the number of people dependent on society for support

D. Stimulating economic growth

Question 28 C
Exposures with the potential of low frequency but high severity should generally be

A. Retained because they are fairly predictable.

B. Retained because this would encourage loss control.

C. Insured because they are highly unpredictable.

D. Written with a large deductible to reduce premium costs.

Question 29 B
John Young is a risk manager of a medium-size business with both auto and real property exposures. John is looking
for ways to save money on insurance premiums. John should consider

A. Retaining the auto liability loss exposures.

B. Retaining some or all of the auto physical damage exposures.

C. Canceling the insurance and implementing noninsurance transfer techniques.

D. Retaining the property loss exposures.

Question 30 A
Which one of the following describes a benefit to businesses of making insurance part of an overall risk management
program instead of relying solely on insurance?

A. Improved access to affordable insurance

B. Stimulating economic growth

C. Reducing the number of persons dependent on society for support

D. Increased use of exposure avoidance

139
Question 1 A
Outdoor Designs Company (ODC) makes cedar patio furniture. ODC's lumber supplier, L&L Wood Products, is the larg-
est and most competitive supplier of lumber in the area. ODC's risk manager is concerned that a disruption in the sup-
ply of lumber from L&L due to a large fire loss would adversely affect its production. Which one of the following would
best minimize the adverse effect to ODC of a large loss experienced by L&L?

A. Separation

B. Duplication

C. Loss prevention

D. Avoidance

Question 2 C
A business installs bars on windows and door deadbolts to prevent burglaries. This is an example of

A. Avoidance.

B. Separation.

C. Loss prevention.

D. Loss reduction.

Question 3 B
Risk management activities under the enterprise-wide risk management approach occur at the

A. Business unit level.

B. Corporate level.

C. Departmental level.

D. Regional level.

Question 4 D
In the selection of appropriate risk management techniques, financial management decisions are often made with the
objective of

A. Decreasing the worry factor.

B. Decreasing expenses.

C. Increasing the cost of risk.

D. Increasing operating efficiency.

140
Question 5 C
Which one of the following best describes how effective risk management benefits society?

A. Creating a positive effect on an insurers underwriting results

B. Providing more thoughtful consumers of insurance

C. Causing fewer disruptions in the economic and social environment

D. Increasing the types of charitable and governmental agencies available to the general public

Question 6 D
Which one of the following is true regarding enterprise-wide risk management (ERM)?

A. ERM is an approach to risk management that focuses primarily on loss exposures associated with pure risk.

B. In practice, implementation of ERM occurs at the departmental or business unit level.

C. Implementation of ERM is fairly consistent among organizations, regardless of their size, nature, or complexity.

D. ERM is an approach to managing all of an organization's key risks and opportunities.

Question 7 A
Malvern Imports has several retail stores in a three-state area. They receive and store imports in a large central ware-
house. The risk manager is considering ways to lower the severity exposure due to a fire loss to the warehouse. Which
one of the following would most effectively decrease the severity exposure of fire loss to the warehouse?

A. Separation

B. Duplication

C. Loss prevention

D. Avoidance

Question 8 C
Properly estimating loss severity is essential in treating a loss exposure because the potential severity is a major con-
sideration in determining whether to

A. Use risk control or risk financing.

B. Use loss avoidance or loss prevention.

C. Insure or retain a particular loss exposure.

D. Physically inspect or rely on a loss exposure survey.

141
Question 9 D
It is easier to gauge the potential severity of property losses than of liability losses because property loss exposures

A. Are confined to the building and contents.

B. Can be determined as an average according to the type of business.

C. Have a calculable frequency.

D. Typically have a finite value.

Question 10 A
One of the benefits to a business of retaining a loss exposure instead of insuring it is

A. A reduction in expenses.

B. A greater peace of mind.

C. Access to the insurer's loss control services.

D. Increase in expenses.

Question 11 B
Chuck and Sally discovered they have flaking lead paint on the walls and trim inside their recently purchased house.
They researched lead paint hazards and abatement techniques on the Internet and discovered the best action they can
take is to remove loose paint chips and dust, replace the windows, and seal the walls with a paint designed for that
purpose. Which one of the following risk management techniques does this activity involve?

A. Avoidance

B. Loss control

C. Retention

D. Noninsurance transfer

Question 12 D
What will individuals gain as a benefit of applying sound risk management to automobile loss exposures?

A. No future increases in insurance premiums

B. A loss free future

C. Economic growth

D. Greater peace of mind

142
Question 13 C
Alan Peachtree owns a hobby shop, which he runs from a detached garage on his property. Alan has set aside funds to
pay for possible property losses rather than purchasing insurance. Which one of the following risk management tech-
niques is Alan using?

A. Avoidance

B. Non-Insurance transfer

C. Retention

D. Loss control

Question 14 B
Individuals and families benefit from effective risk management in which one of the following ways?

A. Increasing their personal cash flows by retaining rather than insuring their property exposures

B. Continuing activities following an accident or other loss, and thus reducing inconvenience.

C. Stimulating economic growth because fewer losses mean that more funds are available for other uses

D. Creating a positive effect on an insurers underwriting results

Question 15 C
The first step in the risk management process is to

A. Implement the selected technique.

B. Select the appropriate technique.

C. Identify loss exposures.

D. Analyze loss exposures.

Question 16 A
In managing loss exposures using the risk management process, the key to identifying loss exposures is

A. Understanding how the household or organization operates.

B. An organizational process flowchart.

C. A financial analysis of customers and suppliers.

D. Understanding the loss frequency and loss severity.

143
Question 17 D
The last step in the risk management process, monitoring results and revising the risk management program,

A. Is complete once expected results are achieved.

B. Does not include complex decisions as do the previous steps of the risk management process.

C. Is not a necessary step for households but is an important step for organizations since their needs often change over
time.

D. Is a return to the first step and begins the risk management process once again.

Question 18 B
Sound risk management benefits society in each of the following ways, EXCEPT:

A. Causing fewer disruptions in the economic and social environment

B. Increasing opportunity costs

C. Reducing the number of people dependent on society for support

D. Stimulating economic growth

Question 19 D
The last step in the risk management process is to

A. Implement the selected technique.

B. Select the appropriate technique.

C. Identify loss exposures.

D. Monitor results.

Question 20 A
Which one of the following identifies the two broad categories of risk management techniques?

A. Risk control and risk financing

B. Loss prevention and loss reduction

C. Separation and duplication

D. Insurance and noninsurance

144
Question 21 C
Katie and Kevin have a small stream in their back yard. Last year was exceptionally rainy and they were concerned
about flood exposures to their home. They recently had a contractor regrade the back yard, building a large berm,
which provides a raised barrier to divert the flow of water away from the house. In this situation, Katie and Kevin are
using which one of the following risk management techniques?

A. Separation

B. Avoidance

C. Loss prevention

D. Loss reduction

Question 22 D
Frank and Jan recently married and purchased their first home together. What is an effective way for Frank and Jan to
begin identifying the liability loss exposures associated with their new home?

A. Hire a professional risk manager

B. Determine the value of the home and contents

C. Purchase a homeowners insurance policy

D. Inspect the home for loss exposures

Question 23 D
Which one of the following is the goal of enterprise-wide risk management (ERM)?

A. Coordinate loss reduction efforts

B. Reduce risk management costs

C. Decentralize control of business decisions

D. Maximize the organizations value

Question 24 D
Which one of the following best describes how effective risk management benefits insurers?

A. Stimulating economic growth

B. Causing fewer disruption in the social environment

C. Increased ability to accurately predict future losses

D. Encourages insurers to create innovative products and offer competitive prices

145
Question 25 C
Traditionally, the risk management professional's role has been associated with loss exposures related to

A. Business risk.

B. Operational risk.

C. Pure risk.

D. Speculative risk.

Question 26 B
A document listing potential loss exposures that a household or an organization may face is a

A. Flowchart.

B. Loss exposure survey or checklist.

C. Loss exposure analysis.

D. Method of monitoring loss exposures.

Question 27 D
In implementing risk management techniques, the risk manager usually

A. Is responsible for communicating guidelines but is otherwise not involved in implementing techniques.

B. Relies on consultants outside the organization to implement the techniques.

C. Has complete authority to make decisions and implement the techniques.

D. Depends on others to implement the techniques based on the risk managers advice.

Question 28 C
Jeff recently started a consulting business. One of his concerns is that he will be sued for giving erroneous advice to a
client. Which one of the following would most likely be the best risk management technique for Jeffs use in this situa-
tion?

A. Retention

B. Avoidance

C. Insurance

D. Duplication

146
Question 29 D
When implementing selected risk management techniques, a risk manager must

A. Base decisions on financial criteria.

B. Base decisions on informal guidelines.

C. Analyze the benefits of the risk management program.

D. Determine how to allocate the costs of the program.

Question 30 D
Risk management departments of large organizations generally rely on a manual to inform others of how to identify
new exposures, what risk management techniques are currently in place, how to report insurance claims, and other
important information. This communication of risk management information is part of which one of the following
steps in the risk management process?

A. Analyzing loss exposures

B. Monitoring results

C. Examining the feasibility of techniques

D. Implementing the selected risk management techniques

147
CHAPTER 8

Question 1 A
The loss of or damage to property can have adverse financial consequences including reduced property value. Which
one of the following statements is correct with respect to a reduction in value?

A. A valuable painting that is repaired after being damaged is likely to have declined in value.

B. For insurance purposes, the dollar amount of a reduction in value is the same regardless of the valuation measure
used.

C. If a cell phone is lost or stolen, its decline in value is measured as its replacement cost minus its salvage value.

D. When property is lost or destroyed, the limit of insurance is deemed to be the amount of the reduction in value.

Question 2 C
To analyze the possible financial consequences of an income loss to his business, Joe estimates his normal revenue and
expenses and compares them to his estimated revenue and expenses should a loss occur.

Normal Operations
Post-Loss

Revenue
$120,000
$0

Expenses
$70,000
$40,000

Net Income
$50,000
($40,000)

Based on Joe's estimates, which one of the following represents his projected net income loss?

A. $ 40,000

B. $ 50,000

C. $ 90,000

D. $120,000

148
Question 3 C
Buildings, investments, patents, and human resources are all examples of

A. Causes of loss, or perils.

B. Financial consequences of loss.

C. Assets exposed to loss.

D. Tangible and intangible hazards.

Question 4 B
When property is used to secure a loan, which of the following is exposed to loss?

A. The secured lender only

B. The property owner and the secured lender

C. The property owner only

D. Neither the property owner nor the secured lender

Question 5 A
Wrongful acts that society deems harmful to the public welfare are addressed by

A. Criminal law.

B. Common law.

C. Contract law.

D. Constitutional law.

Question 6 A
A licensed electrician completes the job of rewiring a house and returns to his shop. With regard to this job, the elec-
trician has a

A. Completed operations loss exposure.

B. Premises liability loss exposure.

C. Business operations loss exposure.

D. Professional liability loss exposure.

149
Question 7 B
With regard to negligence, the plaintiff can make no recovery unless he or she

A. Has a witness.

B. Suffers injury or damage.

C. Has a contract.

D. Breaks the chain of events.

Question 8 C
For insurance purposes, money and securities are separate from other types of contents because

A. They are not susceptible to the same perils as other property.

B. The insurance industry considers them uninsurable.

C. They are highly susceptible to loss by theft.

D. The burden of proof for a loss is on the insurer.

Question 9 C
Kid Smart manufacturing makes a line of childrens toys. The possibility that a child may be injured because of a defect
in a Kid Smart toy is what type of liability loss exposure for this toy manufacturer?

A. Business operations

B. Completed operations

C. Products

D. Professional activities

Question 10 B
While involuntary employee separations may expose an organization to additional liability loss exposures, such as
wrongful termination accusations, they generally are not considered a personnel loss exposure because

A. The organization may not have a risk management person measuring personnel exposures.

B. The organization has determined that it is better off without that employee.

C. The reasons for involuntary employee separations are too numerous and too varied to be measure with high accu-
racy.

D. Employees leave organizations for non-business related reasons and therefore, are not considered a personnel loss.

150
Question 11 A
In insurance terminology, trucks, trailers, buses, fire engines, and ambulances designed for road use are examples of
which one of the following categories of motor vehicles?

A. Autos

B. Mobile equipment

C. Heavy vehicles

D. Recreational vehicles

Question 12 A
When property is lost or damaged, the value of the property is decreased. In addition to direct damage to the prop-
erty, there could be indirect losses to the business. This latter type of loss is known as

A. Net income losses.

B. Deferred income.

C. Historical income.

D. Residual income losses.

Question 13 B
Josephine Redmond who is the sole proprietor of Dream Quilts, Inc., operates out of three branch offices with 17 em-
ployees. What happens to Josephines business if she dies or retires?

A. The business is divided among the employees.

B. The business ceases to exist.

C. The business is divided among the remaining partners.

D. The disposition of the business must be resolved in a court of law.

Question 14 B
A completed operations liability loss exposure differs from a products liability loss exposure in that the completed op-
erations liability loss exposure

A. Is based solely on strict liability rather than negligence or strict liability.

B. Arises out of the entitys completed work, including defective parts or materials furnished with the work.

C. Is based solely on negligence and breach of warranty rather than strict liability.

D. Arises out of the entity's defective product, whether or not it has been installed as part of the finished work.

151
Question 15 B
Other than for money, for what primary reasons or types of reasons would an internationally based American organi-
zations employees be kidnapped or held for ransom?

A. Cultural reasons

B. Political reasons

C. Microeconomic reasons

D. Psychological reasons

Question 16 C
When a newspaper's printing press is damaged, the company may spend extra money to have the newspaper printed
on another company's press. The additional cost of printing the newspaper is an example of which one of the follow-
ing?

A. Reduction in value

B. Lost income

C. Extra expense

D. Altered condition

Question 17 D
Personnel loss exposures can be comprised of all of the following categories of key personnel, EXCEPT:

A. Individual employees

B. Owners, officers, and managers

C. Groups of employees

D. Shareholders

Question 18 A
When a business suffers loss of income following a liability suit that causes customers to lose confidence in the busi-
ness' products, what type of loss has the business suffered?

A. Damage to reputation

B. Punitive damage

C. Special damage

D. Confidence loss

152
Question 19 C
SEB Holdings has grown into a multi-billion dollar financial services firm. For 38 years, Big Hal has been its CEO and
has ruled the company with an iron first. Big Hal is 78 years old, and financial analysts are concerned about his health
and longevity and how closely these factors relate to the companys success. Which one of the following might help
assuage the analysts concerns?

A. A letter attesting to Big Hal's good health

B. Key man life coverage on Big Hal

C. A corporate succession plan

D. Key man retirement plan

Question 20 D
County Janitorial Service has contracted with local bars and taverns to clean their restrooms on a daily basis. Countys
greatest liability exposure is probably

A. Liquor liability.

B. Professional liability.

C. Advertising liability.

D. Completed operations.

Question 21 C
Damages awarded to pay surgeon's fees following an auto accident, are referred to as

A. Punitive damages.

B. General damages.

C. Special damages.

D. Surgical damages.

Question 22 C
Telephone Company installs fiber-optic cable using a trenching machine that digs a trench, buries the cable, fills in the
trench, and reseeds, all in one pass. The trenching machine can install cable in a fifty-home neighborhood in one day.
Soon after the trenching machine passes by, a member of the community steps on the trenched area, sinks in, and
breaks his leg. This is an example of a liability loss exposure arising out of

A. Premises.

B. Products.

C. Completed operations.

153
D. Mobile equipment.

Question 23 C
Lars was running an errand for his employer when he ran a red light and crashed into the side of another vehicle. Un-
der which one of the following tort liability concepts could Lars' employer be held liable?

A. Expanded liability concepts

B. Concert of action

C. Vicarious liability

D. Alternative liability

Question 24 D
No-fault auto laws are an example of

A. Tort law.

B. Contract law.

C. Criminal law.

D. Statutory law.

Question 25 C
The term "net income losses" is usually associated with

A. Loss of goodwill.

B. Liability losses.

C. Property losses.

D. Missed opportunities.

Question 26 C
The difference between an organizations total revenues and its total expenses is

A. Operating profit.

B. Net sales.

C. Net income.

D. Gross profit.

154
Question 27 C
An auto dealer's showroom is destroyed by fire, and the dealer has to temporarily rent an adjacent building to use as
a showroom. This is an example of

A. A reduction in value.

B. Lost income.

C. An extra expense.

D. An altered condition.

Question 28 A
ABC Company manufactures hot beverage containers and distributes them to various retail outlets. A customer who
purchased one of these containers at a retail store is burned by hot coffee after the lid on a container falls off. Which
one of the following principles of law would be the basis for a suit brought by the injured party?

A. Implied warranty

B. Express warranty

C. Misrepresentation

D. Assumption of risk

Question 29 B
The cause of loss that occurs more frequently than death, causing personnel losses, is

A. Retirement.

B. Disability.

C. Kidnap.

D. War/terrorism.

Question 30 D
Property insurance policies not only protect the property owner but also generally protect a secured lenders interest
in the financed property by

A. Issuing a mortgagee interest-only policy.

B. Sending the lender a letter of intent.

C. Issuing a separate policy.

D. Naming the lender on the owners policy.

155
Question 1 B
Other than some type of merger, layoff, or organizational change, which one of the following is an example of a situa-
tion where an entire group of employees might leave an organization?

A. When the economy is in a long term period of inflation

B. When employees follow a manager to a new organization

C. When statutes make it more attractive to pursue other career opportunities

D. When an organization offers excessive compensation

Question 2 D
A company turnover rate well above company and industry averages could be a sign of

A. Employee slacking.

B. Financial problems that need to be addressed.

C. Claims-consciousness.

D. Personnel problems that need to be addressed.

Question 3 D
The possibility that a retailer might be sued because it used a customer's photo in a sales brochure without the cus-
tomer's permission is what kind of liability exposure for the retailer?

A. Products

B. Business operations

C. Professional activities

D. Advertising

Question 4 C
Jennifer was injured in an auto accident and claimed that her medical expenses were $10,000, the physical therapy
costs were $5,000, and her lost wages were $3,000. Additionally, she claimed $10,000 for pain and suffering. If a court
awards Jennifer only special damages and denies any general damages, she will receive

A. $10,000.

B. $13,000.

C. $18,000.

156
D. $25,000.

Question 5 A
The possibility that a plumbing contractor might start a fire at a customer's house while soldering a pipe is what type
of liability loss exposure for the plumber?

A. Business operations

B. Completed operations

C. Products

D. Professional activities

Question 6 A
When John was driving his car, he was inattentive and was unable to stop in time to avoid hitting a car stopped at a
traffic light. The collision damaged the stopped car. What is the basis for John's liability?

A. Negligence

B. Absolute liability

C. Statutory liability

D. No-fault statutes

Question 7 B
Josephine Redmond who is the sole proprietor of Dream Quilts, Inc., operates out of three branch offices with 17 em-
ployees. What happens to Josephines business if she dies or retires?

A. The business is divided among the employees.

B. The business ceases to exist.

C. The business is divided among the remaining partners.

D. The disposition of the business must be resolved in a court of law.

Question 8 C
Renata owns a valuable antique sculpture by a famous artist. One day she accidentally dropped the sculpture and a
piece of it broke off. Renata took the sculpture to an art restorer, who reattached the broken piece and repaired the
surface. From a distance, the statue looks the same as it did before Renata dropped it, but a close inspection reveals
the repair. Which one of the following best describes the financial consequences of the accident?
A. There are no financial consequences because the piece looks the same as it did before the accident.
B. The only financial consequence of the accident is the money Renata paid the art restorer for the repair.
C. The financial consequences include the cost of the repair and the reduction in value resulting from the repaired
damage.
D. Renata has incurred a total loss because original artwork is impossible to replace.

157
Question 9 A
Benny lost track of where his property ended and ran over a neighbor's sapling with his lawn mower. The neighbor
John sued Benny for $700 stating it was an exotic tree that could not easily be replaced. John also claimed that be-
cause of the loss, he was undergoing emotional stress for which he claimed $5,000. Additionally, John asserted that
the noise of Benny's mower had caused him to lose hearing in his right ear and he was also suing Benny for $5,000 for
loss of hearing. The court awarded no general damages but did award special damages related to John's lawsuit. The
amount John will receive as a special damages award is

A. $ 700.

B. $ 5,000.

C. $ 5,700.

D. $10,000.

.ID: 5849922

Question 10 A
When property is lost or damaged, the value of the property is decreased. In addition to direct damage to the prop-
erty, there could be indirect losses to the business. This latter type of loss is known as

A. Net income losses.

B. Deferred income.

C. Historical income.

D. Residual income losses.

Question 11 C
The term "net income losses" is usually associated with

A. Loss of goodwill.
B. Liability losses.
C. Property losses.
D. Missed opportunities.

Question 12 C
Kid Smart manufacturing makes a line of childrens toys. The possibility that a child may be injured because of a defect
in a Kid Smart toy is what type of liability loss exposure for this toy manufacturer?

A. Business operations

B. Completed operations

C. Products

D. Professional activities

158
Question 13 B
Because attorneys, physicians, architects, and engineers are considered to be experts in their fields and are expected
to perform accordingly, what liability exposure do they face if a client is injured when such an expert fails to exercise
the appropriate standard of care?

A. Completed operations

B. Professional liability

C. Business operations

D. Products

Question 14 C
A net income loss exposure is a condition that presents the possibility of loss caused by a reduction in net income. In a
given time period, net income equals revenue minus

A. Costs of goods sold.

B. Accounts payable and income taxes.

C. Expenses and income taxes.

D. Continuing expenses and outstanding liabilities.

Question 15 B
When property is used to secure a loan, which of the following is exposed to loss?

A. The secured lender only

B. The property owner and the secured lender

C. The property owner only

D. Neither the property owner nor the secured lender

Question 16 B
The cause of loss that occurs more frequently than death, causing personnel losses, is

A. Retirement.

B. Disability.

C. Kidnap.

D. War/terrorism.

159
Question 17 B
Contract law is a branch of

A. Common law.

B. Civil law.

C. Criminal law.

D. Constitutional law.

Question 18 C
Which one of the following best defines a hazard?

A. The actual means by which property is damaged or destroyed

B. Anything that shifts the burden of proof to the insurer

C. Anything that increases the frequency or the severity of a loss

D. An uninterrupted chain of events

Question 19 D
Property insurance policies not only protect the property owner but also generally protect a secured lenders interest
in the financed property by

A. Issuing a mortgagee interest-only policy.

B. Sending the lender a letter of intent.

C. Issuing a separate policy.

D. Naming the lender on the owners policy.

Question 20 A
The death of a shareholder in a close corporation is often a significant event for the corporation for which one of the
following reasons?

A. Ownership in a close corporation is typically concentrated in just the few major shareholders, most of whom are
also managers.

B. The death or disability of one of the shareholders generally results in payment of dividends to the remaining share-
holders.

C. Key shareholders are paid high salaries and are difficult to replace.

D. Shareholders in a close corporation also serve on the board of directors resulting in a disparity of shareholder votes.

160
Question 21 C
Another term for cause of loss is

A. Negative outcome.

B. Exposure.

C. Peril.

D. Hazard.

Question 22 C
The difference between an organizations total revenues and its total expenses is

A. Operating profit.

B. Net sales.

C. Net income.

D. Gross profit.

Question 23 C
Telephone Company installs fiber-optic cable using a trenching machine that digs a trench, buries the cable, fills in the
trench, and reseeds, all in one pass. The trenching machine can install cable in a fifty-home neighborhood in one day.
Soon after the trenching machine passes by, a member of the community steps on the trenched area, sinks in, and
breaks his leg. This is an example of a liability loss exposure arising out of

A. Premises.

B. Products.

C. Completed operations.

D. Mobile equipment.

Question 24 C
Which one of the following elements is necessary for a plaintiff to prevail in an assault case?

A. There is a fear that there may be physical contact immediately or sometime in the future.

B. A threat is made regardless of whether the individual making the threat can actually act on it.

C. A threat is made with the intent of creating fear or apprehension.

D. The damages resulting from an assault are significant and measurable in nature.

161
Question 25 D
Extra expenses are a common financial consequence of property losses. Which one of the following is an example of
an extra expense?

A. A fence worth $7,000 was damaged by a car, and the fence owner has to pay $2,000 to have the damage repaired.

B. When a grocery store in a mall burns, other nearby stores have to close and lose revenue until the grocery store is
repaired.

C. Even though fire has made a building untenantable, the landlord must continue making mortgage and tax pay-
ments.

D. When a familys house is damaged, the family must pay rent to live in a hotel temporarily.

Question 26 A
ABC Company manufactures hot beverage containers and distributes them to various retail outlets. A customer who
purchased one of these containers at a retail store is burned by hot coffee after the lid on a container falls off. Which
one of the following principles of law would be the basis for a suit brought by the injured party?

A. Implied warranty

B. Express warranty

C. Misrepresentation

D. Assumption of risk

Question 27 B
All for-profit organizations are exposed to net income losses. Net income is defined as

A. Total assets minus total liabilities.

B. Total revenue minus total expenses.

C. Total profit minus total expenses.

D. Total income minus indirect losses.

Question 28 D
County Janitorial Service has contracted with local bars and taverns to clean their restrooms on a daily basis. Countys
greatest liability exposure is probably

A. Liquor liability.

B. Professional liability.

C. Advertising liability.

162
D. Completed operations.

Question 29 A
Which one of the following is a financial consequence of personnel losses to an organization?

A. Losses to an organizations value caused by negative publicity

B. Expenses of paying replacement employees more than departing employees

C. Cost allocations relating to departing employees benefit plans

D. Costs of hiring new management personnel to manage a younger workforce

Question 30 C
Perils are an important aspect of property insurance. Which one of the following statements is correct with respect to
perils?

A. In order to be covered by an insurance policy, a peril must be defined and described in that policy.

B. A peril is anything that increases the frequency or the severity of a loss.

C. A peril is the actual means by which property is damaged or destroyed such as fire, lightning, windstorm, hail, or
theft.

D. Property insurance policies can cover specific perils and cannot exclude other perils.

Question 1 C
The body of principles and rules established over time by courts on a case-by-case basis is known as

A. Statutory law.

B. Informal law.

C. Common law.

D. State law.

Question 2 D
Ships and their cargoes represent a special category of property insurance. Which one of the following statements is
correct with respect to this category?

A. Because of the high risk of loss, ships and their cargoes are generally considered to be uninsurable.

B. If a ship cannot reach its intended destination, the cargo cannot be sold in a different port.

C. Ships and their cargoes are exposed only to the perils of windstorm and sinking.

D. Even more than other property, ocean cargoes fluctuate in value according to their location.

163
Question 3 B
When property is used to secure a loan, which of the following is exposed to loss?

A. The secured lender only

B. The property owner and the secured lender

C. The property owner only

D. Neither the property owner nor the secured lender

Question 4 D
John was involved in a serious accident that included injuries to the other party, and charged with driving while intoxi-
cated. The actions that are likely to be brought against John are

A. Statutory and common.

B. Civil and common.

C. Constitutional and criminal.

D. Civil and criminal.

Question 5 C
A neighbor has sued Andy for property damage that is potentially covered under Andy's homeowners liability cover-
age. Even if this lawsuit is found to be groundless, Andy's insurer will still probably be obligated to pay

A. Special damages.

B. General damages.

C. Defense costs.

D. Compensatory damages.

Question 6 A
Recruiting, interviewing, and training a new employee who replaces a worker who has just resigned are examples of
which one of the following types of financial consequences flowing from personnel losses?

A. Replacement costs

B. Depreciation costs

C. Loss of value

D. Compensation costs

164
Question 7 A
Products liability loss exposures arise out of injuries or damage that result from an organizations

A. Defective product.

B. Operations away from premises.

C. Types of bailments.

D. Manufacturing operations.

Question 8 B
All for-profit organizations are exposed to net income losses. Net income is defined as

A. Total assets minus total liabilities.

B. Total revenue minus total expenses.

C. Total profit minus total expenses.

D. Total income minus indirect losses.

Question 9 A
Colson makes knives and swords as a hobby selling his products on a non-advertised basis. He does not package his
products but does provide a warranty against defects. If a parent whose child is injured by one of Colson's ever-sharp
knives sues Colson under strict liability, which one of the following statements may be true?

A. Colson would be strictly liable only if the knife was unreasonably dangerous in normal use.

B. Colson would be liable because of failure to warn that the knife was sharp and dangerous.

C. Knives are generally known to be dangerous instrumentalities and Colson could not be held liable.

D. Colson is not liable for harm to a third party.

Question 10 C
The difference between an organizations total revenues and its total expenses is

A. Operating profit.

B. Net sales.

C. Net income.

D. Gross profit.

165
Question 11 B
Which one of the following is an element of a loss exposure?

A. The verification of risk

B. A cause of loss

C. The probability of a loss

D. The occurrence of a loss

Question 12 C
An auto dealer's showroom is destroyed by fire, and the dealer has to temporarily rent an adjacent building to use as
a showroom. This is an example of

A. A reduction in value.

B. Lost income.

C. An extra expense.

D. An altered condition.

Question 13 B
Because attorneys, physicians, architects, and engineers are considered to be experts in their fields and are expected
to perform accordingly, what liability exposure do they face if a client is injured when such an expert fails to exercise
the appropriate standard of care?

A. Completed operations

B. Professional liability

C. Business operations

D. Products

Question 14 D
Extra expenses are a common financial consequence of property losses. Which one of the following is an example of
an extra expense?
A. A fence worth $7,000 was damaged by a car, and the fence owner has to pay $2,000 to have the damage repaired.

B. When a grocery store in a mall burns, other nearby stores have to close and lose revenue until the grocery store is
repaired.

C. Even though fire has made a building untenantable, the landlord must continue making mortgage and tax pay-
ments.

D. When a familys house is damaged, the family must pay rent to live in a hotel temporarily.

166
Question 15 C
Anyone who owns or occupies property has a

A. Products liability loss exposure.

B. Professional liability loss exposure.

C. Premises liability loss exposure.

D. Pollution liability loss exposure.

Question 16 D
The possibility that a retailer might be sued because it used a customer's photo in a sales brochure without the cus-
tomer's permission is what kind of liability exposure for the retailer?

A. Products

B. Business operations

C. Professional activities

D. Advertising

Question 17 A
Jim's Computer Repair will often take customers' computers into the shop for repairs and service. While Jim has a cus-
tomer's computer in his possession, he is in the role of

A. A bailee.

B. A bailor.

C. A secured lender.

D. An agent.

Question 18 A
Which one of the following is a financial consequence of personnel losses to an organization?

A. Losses to an organizations value caused by negative publicity

B. Expenses of paying replacement employees more than departing employees

C. Cost allocations relating to departing employees benefit plans

D. Costs of hiring new management personnel to manage a younger workforce

167
Question 19 C
Buildings, investments, patents, and human resources are all examples of

A. Causes of loss, or perils.

B. Financial consequences of loss.

C. Assets exposed to loss.

D. Tangible and intangible hazards.

Question 20 D
Personnel loss exposures can be comprised of all of the following categories of key personnel, EXCEPT:

A. Individual employees

B. Owners, officers, and managers

C. Groups of employees

D. Shareholders

Question 21 C
The term "net income losses" is usually associated with

A. Loss of goodwill.

B. Liability losses.

C. Property losses.

D. Missed opportunities.

Question 22 B
Other than some type of merger, layoff, or organizational change, which one of the following is an example of a situa-
tion where an entire group of employees might leave an organization?

A. When the economy is in a long term period of inflation

B. When employees follow a manager to a new organization

C. When statutes make it more attractive to pursue other career opportunities

D. When an organization offers excessive compensation

168
Question 23 B
Brenda drove too rapidly through a work zone and ran over a worker's foot. Because she was afraid she would have to
pay for the worker's injury, she did not stop to assist the worker. The liability imposed on Brenda for leaving the scene
rather than providing assistance comes from

A. Public duty.

B. Criminal law.

C. Contractual law.

D. Negligence liability.

Question 24 A
The death of a shareholder in a close corporation is often a significant event for the corporation for which one of the
following reasons?

A. Ownership in a close corporation is typically concentrated in just the few major shareholders, most of whom are
also managers.

B. The death or disability of one of the shareholders generally results in payment of dividends to the remaining share-
holders.

C. Key shareholders are paid high salaries and are difficult to replace.

D. Shareholders in a close corporation also serve on the board of directors resulting in a disparity of shareholder votes.

Question 25 B
A contractual provision that obligates one party to assume the financial consequences of legal liability for another
party is

A. A statute.

B. A hold-harmless agreement.

C. An example of strict liability.

D. A warranty.

Question 26 C
Which one of the following elements is necessary for a plaintiff to prevail in an assault case?

A. There is a fear that there may be physical contact immediately or sometime in the future.

B. A threat is made regardless of whether the individual making the threat can actually act on it.

C. A threat is made with the intent of creating fear or apprehension.

D. The damages resulting from an assault are significant and measurable in nature.

169
Question 27 C
For insurance purposes, money and securities are separate from other types of contents because

A. They are not susceptible to the same perils as other property.

B. The insurance industry considers them uninsurable.

C. They are highly susceptible to loss by theft.

D. The burden of proof for a loss is on the insurer.

Question 28 D
Which one of the following potential financial consequences of liability loss exposures is often difficult to quantify?

A. Special damages

B. Defense costs

C. Costs of investigation

D. Damage to reputation

Question 29 B
Other than for money, for what primary reasons or types of reasons would an internationally based American organi-
zations employees be kidnapped or held for ransom?

A. Cultural reasons

B. Political reasons

C. Microeconomic reasons

D. Psychological reasons

Question 30 C
When a newspaper's printing press is damaged, the company may spend extra money to have the newspaper printed
on another company's press. The additional cost of printing the newspaper is an example of which one of the follow-
ing?

A. Reduction in value

B. Lost income

C. Extra expense

D. Altered condition

170
CHAPTER 9

Question 1 C
An insurer will make a loss payment if an insured loss occurs and if the insured performs certain duties. This illustrates
that an insurance policy is a
A. Contract of adhesion.

B. Contract of indemnity.

C. Conditional contract.

D. Personal contract.

Question 2 B
Beas Restaurant suffered a serious fire due to a lightning strike, and the structure had to be torn down. The damage
to the building is an example of

A. An indirect loss.

B. A direct loss.

C. A time element loss.

D. A consequential loss.

Question 3 A
Which one of the following types of policies provides the broadest coverage for personal property in terms of the cov-
ered locations?

A. Homeowners

B. Transportation

C. Floater

D. Commercial property

Question 4 B
Basheer sold his car to Carl. For an additional sum, he also transferred his auto policy to Carl, to go with the car. Is Carl
now entitled to the benefits promised by Basheer's policy?

A. Yes, because the assignment was a proper transaction.

B. No, because Basheer's policy is not transferable.

C. It depends on whether or not Carl suffers a subsequent loss.

D. It depends on the intent of the parties.

171
Question 5 B
Rachel is an insurance broker. She has received a sales referral from another broker concerning an airport manage-
ment company, Airport Ltd. The company plans to operate a large, national airport in a major city and has requested
a quote for airport liability coverage. Rachel will probably recommend the use of which one of the following types of
policy forms?

A. Miscellaneous

B. Manuscript

C. Preprinted

D. Self-contained

Question 6 B
A broad statement of an insurer's promises to the insured may be found in a policy's

A. Declarations.

B. Insuring agreements.

C. Conditions.

D. Endorsements.

Question 7 C
Which one of the following statements about policy sections is true?

A. An insuring agreement establishes procedures for implementing the policy.

B. Exclusions define terms to clarify ambiguity.

C. A condition clarifies an insurer's promise or an insured's duty.

D. Declarations state what an insurer will not cover.

Question 8 D
Which one of the following statements is true regarding insurance policies?

A. Because of their special nature, insurance policies must meet several requirements in addition to those required for
other valid contracts.

B. Because the insurer's promise to pay claims spans several years, insurance policies have a separate set of require-
ments than those for any other valid contract.

C. Insurance policies are approved in advance and therefore are presumed to meet all of the requirements of a valid
contract.

172
D. Insurance policies must meet the same requirements as any other valid contract.

Question 9 D
All commercial package policies begin with two components, namely, common policy conditions and common

A. Coverages.

B. Manuscripts.

C. Modules.

D. Declarations.

Question 10 A
The validity of a contract depends on all of the following essential elements, EXCEPT:

A. Adhesion

B. Legal purpose

C. Consideration

D. Agreement

Question 11 D
Because insurance contracts impose an obligation of complete honesty on the parties, an insurance contract is called a

A. Contract of indemnity.

B. Contract of adhesion.

C. Conditional contract.

D. Contract of utmost good faith.

Question 12 B
In creation of a contract of insurance, what is the consideration that is provided by the insurer?

A. The prompt payment of claims

B. The insurer's promise to pay claims for covered losses

C. The insurer's delivering the insurance policy to the insured

D. The acceptance of the application

173
Question 13 C
An insuring agreement may state the insurer's promise to do all of the following in the event of a covered loss, EX-
CEPT:

A. Indemnify the insured

B. Defend against the claim

C. Surcharge the insured

D. Pay the claim

Question 14 A
Standard forms

A. May result in different insurers issuing identical policies.

B. Prevent insurers from offering significantly different coverage.

C. Are commonly used in professional liability insurance.

D. Allow insurers to modify terms and conditions to accommodate differences in applicants' needs.

Question 15 C
Broad pollution coverage is not typically included in liability policies primarily as an attempt to

A. Eliminate duplicate coverage.

B. Avoid covering uninsurable losses.

C. Keep premiums reasonable.

D. Avoid insuring losses that could be prevented.

Question 16 D
Which one of the following types of insurers is the most likely to include its bylaws as part of its policies?

A. No-fault insurers

B. Auto insurers

C. Workers compensation insurers

D. Mutual insurers

174
Question 17 A
When Amy applied for insurance, her agent asked her if she had received more than three traffic tickets in the last five
years. Amy had received four, but remembered only three. Thus, she told the agent she had received only three. Amy's
statement is an example of

A. Misrepresentation.

B. Concealment.

C. Intent to deceive.

D. Intent to withhold information.

Question 18 C
In commercial insurance, the person who is responsible for paying premiums and who also has the right to receive any
return premiums is the

A. Named insured.

B. Additional insured.

C. First named insured.

D. Other insured.

Question 19 A
Which one of the following can be covered by a floater?

A. Jewelry

B. Swimming pools

C. Carports

D. Antenna towers

Question 20 A
Documents that can become part of an insurance policy include all of the following, EXCEPT:

A. Summary of available coverage options

B. Relevant statutory terms and provisions

C. Completed application

D. Insurer's bylaws

175
Question 21 D
If duress, coercion, fraud, or mistake is involved in the creation of a contract, then the contract is

A. Nontransferable.

B. Under consideration.

C. Binding.

D. Unenforceable.

Question 22 B
What purpose do insurance-to-value provisions serve in property insurance policies?

A. They discourage insurance fraud by ensuring that the property is worth the value requested in the policy limits.

B. They encourage insureds to purchase an amount of insurance that is equal to, or close to, the value of the covered
property.

C. They establish the maximum amount the insurance company will pay for any loss.

D. They establish the options available to the insurance company in settling the loss.

Question 23 B
If an insurer mistakenly writes an insurance policy in a state in which it is not licensed, the insured might later success-
fully argue on what grounds that the contract is not valid?

A. No legal purpose

B. The lack of competent parties

C. The lack of consideration

D. No agreement

Question 24 A
The definitions section of the policy

A. Helps clarify real or perceived ambiguity.

B. Contains a definitive statement of coverage.

C. Consists of dictionary meanings of words.

D. Describes the services provided by the insurer.

176
Question 25 C
All of the following explain the importance of a policy limit, EXCEPT:

A. It tells the insurer the maximum amount it may have to pay for a covered loss.

B. It helps the insured determine whether the amount of insurance is adequate.

C. It tells the insurer whether to value an item at replacement cost or actual cash value.

D. It helps the insurer keep track of its overall obligations in any one geographical area.

Question 26 A
Because the insurance company usually determines policy wording and the insured has little choice but to "take it or
leave it," an insurance contract is a

A. Contract of adhesion.

B. Contract of indemnity.

C. Nontransferable contract.

D. Bilateral contract.

Question 27 A
Most homeowners policies provide coverage for relatives of the named insured who

A. Reside in the same household.

B. Are less than 18 years old.

C. Are the legal responsibility of the named insured or spouse.

D. Are related by blood to the named insured.

Question 28 C
Bob has purchased an auto insurance policy with $300,000 of single limit liability coverage. This policy has replaced
his previous auto policy, which had split limits for bodily injury and property damage. Bob was at fault in an auto acci-
dent, which resulted in a $250,000 bodily injury settlement and payment of $75,000 for property damage. How much
will Bob's new policy pay in damages for this loss?

A. $ 75,000

B. $250,000

C. $300,000

D. $325,000

177
Question 29 D
All of the following are typical examples of conditions in an insurance policy pertaining to the insured's obligations,
EXCEPT:

A. Documenting losses

B. Reporting losses in a timely manner

C. Paying premiums

D. Defending itself from lawsuits

Question 30 B
Which one of the following correctly describes a property insurance policy covering property of others?

A. Commercial property policies generally extend coverage to the dwellings and personal property of company own-
ers, officers, and partners.

B. Homeowners policies provide coverage for the personal property of others, such as guests or employees, while the
property is in the insureds home.

C. Floaters typically extend coverage to include the personal property of others while it is in the care, custody, or con-
trol of the insured.

D. The personal auto policy extends physical damage coverage to any automobile leased by the insured but not listed
on the policy.

Question 1 B
A commercial package policy is an example of a
A. Manuscript policy.
B. Modular policy.
C. Standard self-contained policy.
D. Nonstandard self-contained policy.

Question 2 B
Commercial property insurance policies usually refer to the contents of buildings as business personal property. Which
one of the following best describes the coverage typically provided for business personal property?

A. Commercial policies cover all business personal property on a worldwide basis.

B. Business personal property includes furniture, machinery, equipment, and stock that are not part of the building.

C. Commercial policies cover only those items of business personal property specifically listed on the declarations
page.

D. Business personal property also includes owned vehicles while parked within 100 feet of the described premises.

178
Question 3 D
Most property policies providing coverage on buildings and personal property at fixed locations exclude coverage for

A. Riot and civil commotion.

B. Vandalism.

C. Explosion.

D. Earthquake and flood.

Question 4 B
In creation of a contract of insurance, what is the consideration that is provided by the insurer?

A. The prompt payment of claims

B. The insurer's promise to pay claims for covered losses

C. The insurer's delivering the insurance policy to the insured

D. The acceptance of the application

Question 5 A
A policy exclusion can serve all of the following purposes, EXCEPT:

A. State the coverages exclusively provided by the policy.

B. Eliminate coverage for uninsurable loss exposures.

C. Help manage morale hazards.

D. Eliminate coverages requiring special treatments.

Question 6 A
The policy section that usually contains the policyholders name and mailing address, policy limits, and premium
amount is the

A. Declarations.

B. Definitions.

C. Conditions.

D. Miscellaneous provisions.

179
Question 7 D
All commercial package policies begin with two components, namely, common policy conditions and common

A. Coverages.

B. Manuscripts.

C. Modules.

D. Declarations.

Question 8 D
Which one of the following statements is true regarding insurance policies?

A. Because of their special nature, insurance policies must meet several requirements in addition to those required for
other valid contracts.

B. Because the insurer's promise to pay claims spans several years, insurance policies have a separate set of require-
ments than those for any other valid contract.

C. Insurance policies are approved in advance and therefore are presumed to meet all of the requirements of a valid
contract.

D. Insurance policies must meet the same requirements as any other valid contract.

Question 9 A
The validity of a contract depends on all of the following essential elements, EXCEPT:

A. Adhesion

B. Legal purpose

C. Consideration

D. Agreement

Question 10 C
Insurance policies are written in such a way that the parties have to perform only under certain conditions, because an
insurance contract is a

A. Contract of indemnity.

B. Contract of adhesion.

C. Conditional contract.

D. Contract of utmost good faith.

180
Question 11 B
The aggregate limit in a liability policy is the maximum amount an insurer will pay for all covered losses during the

A. Calendar year.

B. Policy period.

C. Retroactive period.

D. Fiscal year.

Question 12 C
Under a liability policy, the maximum amount an insurer will pay for both bodily injury and property damage arising
from a single occurrence is known as the

A. Aggregate limit.

B. Split limit.

C. Occurrence limit.

D. Defined limit.

Question 13 C
An insurance policy is considered a contract of adhesion when it is drafted by the

A. Insured.

B. Third party.

C. Insurer.

D. Insurance commissioner.

Question 14 C
Joe owns a building covered by a property insurance policy. Joe intentionally sets fire to and destroys this building. He
then files a claim with his insurance company. Which one of the following statements best describes why Joe would be
precluded from recovering under his policy for this loss?

A. Joe acted under emotional duress

B. Joe acted irrationally

C. Payment of the claim would be a violation of public policy

D. Payment of the claim would violate the principle of indemnity

181
Question 15 A
Iron Works, Inc., is insured under a commercial liability policy listing several names on the declarations page. The dec-
larations lists Bart Danner, Paul Binder, and Iron Works, Inc., in that order. A policy provision would generally identify
which of the following as the insured(s) with whom the insurer has primary contact?

A. Bart Danner

B. Paul Binder

C. Iron Works, Inc.

D. All of the above

Question 16 D
All of the following are typical examples of conditions in an insurance policy pertaining to the insured's obligations,
EXCEPT:

A. Documenting losses

B. Reporting losses in a timely manner

C. Paying premiums

D. Defending itself from lawsuits

Question 17 C
Which one of the following statements about policy parts is true?

A. An insuring agreement establishes procedures for implementing the policy.

B. Exclusions define terms to clarify ambiguity.

C. A condition clarifies an insurer's promise or an insured's duty.

D. Declarations state what an insurer will not cover.

Question 18 A
When Amy applied for insurance, her agent asked her if she had received more than three traffic tickets in the last five
years. Amy had received four, but remembered only three. Thus, she told the agent she had received only three. Amy's
statement is an example of

A. Misrepresentation.

B. Concealment.

C. Intent to deceive.

D. Intent to withhold information.

182
Question 19 C
Wear and tear, rust, marring, and scratching are examples of maintenance perils that are usually excluded by prop-
erty insurance policies because

A. There might not be enough claim adjusters in the event of a loss.

B. It is against public policy to cover these types of perils.

C. Losses arising from these perils are certain to occur.

D. It is illegal to cover these types of perils.

Question 20 D
War is generally excluded from property insurance policies because

A. War losses can be prevented through careful planning.

B. The federal government covers war losses.

C. Most families and businesses do not face a war exposure.

D. War affects many people at the same time with widespread losses.

Question 21 B
Claims-made coverage typically is used to insure businesses that face certain types of liability exposures, such as

A. Bailee liability.

B. Medical malpractice.

C. Homeowners personal liability.

D. Garagekeepers liability.

Question 22 A
Standard forms

A. May result in different insurers issuing identical policies.

B. Prevent insurers from offering significantly different coverage.

C. Are commonly used in professional liability insurance.

D. Allow insurers to modify terms and conditions to accommodate differences in applicants' needs.

183
Question 23 C
A liability policy has an aggregate limit of $1 million, an each occurrence limit of $500,000, and an each person limit of
$100,000. A court ordered the insured under this policy to pay $50,000 in damages to each of three claimants as the
result of a single accident. What is the total amount payable under this policy to all three claimants?

A. $50,000

B. $100,000

C. $150,000

D. $500,000

Question 24 D
Which one of the following is an example of material misrepresentation by Albert, a businessman applying for prop-
erty insurance on a building he owns?

A. Albert does not tell the insurance agent he is about to file for bankruptcy.

B. Albert does not tell the insurance agent his building has a defective heating system that could explode at any time.

C. Albert tells the insurance agent he has owned the building for seventeen years, but he has actually owned it for
eighteen years.

D. Albert tells the insurance agent the building is used to store steel, but it is actually used to store steel drums con-
taining flammable liquids.

Question 25 B
Ching is a commercial claim representative who has been assigned a large fire loss. Ching suspects that the insured
may not have been completely honest in describing the property before the policy was issued such that coverage for
the claim may be denied. In which document might Ching find the representations of the insured regarding the prop-
erty's description?

A. Declarations

B. Application

C. Endorsements

D. Manuscript

Question 26 A
The definitions section of the policy

A. Helps clarify real or perceived ambiguity.

B. Contains a definitive statement of coverage.

C. Consists of dictionary meanings of words.

184
D. Describes the services provided by the insurer.

Question 27 B
For a contract to be enforceable, something of value must be exchanged. The thing of value exchanged is called

A. Mutual assent.

B. Consideration.

C. Indemnity.

D. Adhesion.

Question 28 B
An insurer has decided to take an extremely narrow interpretation of a property insurance policy provision to limit the
number of loss payments it will need to make. It realizes that its interpretation is probably wrong, but it knows that
individual insured's loss amounts will be small, such that most insureds will not take the trouble to file lawsuits
against it. The insurer may be violating the principle that an insurance policy is

A. A contract of adhesion.

B. A contract of utmost good faith.

C. A conditional contract.

D. A contract of indemnity.

Question 29 B
A property insurance policy specifies what property is covered and where it is covered.Which one of the following is
correct with regard to the property or location covered for a specific type of insurance policy?

A. Personal auto policies typically cover collision damage to a described auto on a worldwide basis.

B. Homeowners policies typically cover personal property on a worldwide basis, with some limitations.

C. The building coverage of a commercial property policy usually excludes permanently installed fixtures and equip-
ment.

D. The building coverage of a commercial property policy usually excludes fire extinguishing equipment.

Question 30 A
Self-contained policies

A. Address complete coverage needs.

B. Require more than one form to address coverage needs.

C. Address unique coverage needs.

185
D. Do not address all of an insured's coverage needs.

Question 1 A
In terms of the elements of a contract, an insurance application is

A. An offer.

B. An acceptance.

C. A consideration.

D. An agreement.

Question 2 B
Law Firm asked Insurance Company to provide both a commercial liability policy and a professional liability policy to
meet its coverage needs. Insurance Company believes that there could be some types of losses that would be covered
under both policies, a situation it wishes to avoid. What type of policy provisions could Insurance Company use to ad-
dress this problem?

A. Coverage sections

B. Exclusions

C. Additions

D. Extensions

Question 3 A
Documents that can become part of an insurance policy include all of the following, EXCEPT:

A. Summary of available coverage options

B. Relevant statutory terms and provisions

C. Completed application

D. Insurer's bylaws

Question 4 B
Many property insurance policies contain a provision that reduces the amount of recovery for loss if the property is
underinsured. This provision is known as

A. Reinsurance.

B. Coinsurance.

C. The deductible.

D. The other insurance clause.

186
Question 5 C
In commercial insurance policies, several different individuals and businesses may be listed as named insureds.Which
one of the following is the term used to refer to the party that is responsible for paying premiums and that has the
right to receive any return premiums and cancel the policy?

A. The named insured

B. The contact person

C. The first named insured

D. The authorized agent

Question 6 C
Which one of the following statements about policy parts is true?

A. An insuring agreement establishes procedures for implementing the policy.

B. Exclusions define terms to clarify ambiguity.

C. A condition clarifies an insurer's promise or an insured's duty.

D. Declarations state what an insurer will not cover.

Question 7 D
Which one of the following is an example of material misrepresentation by Albert, a businessman applying for prop-
erty insurance on a building he owns?

A. Albert does not tell the insurance agent he is about to file for bankruptcy.

B. Albert does not tell the insurance agent his building has a defective heating system that could explode at any time.

C. Albert tells the insurance agent he has owned the building for seventeen years, but he has actually owned it for
eighteen years.

D. Albert tells the insurance agent the building is used to store steel, but it is actually used to store steel drums con-
taining flammable liquids.

Question 8 D
Because insurance contracts impose an obligation of complete honesty on the parties, an insurance contract is called a

A. Contract of indemnity.

B. Contract of adhesion.

C. Conditional contract.

D. Contract of utmost good faith.

187
Question 9 B
Ching is a commercial claim representative who has been assigned a large fire loss. Ching suspects that the insured
may not have been completely honest in describing the property before the policy was issued such that coverage for
the claim may be denied. In which document might Ching find the representations of the insured regarding the prop-
erty's description?

A. Declarations

B. Application

C. Endorsements

D. Manuscript

Question 10 B
Commercial property insurance policies usually refer to the contents of buildings as business personal property. Which
one of the following best describes the coverage typically provided for business personal property?

A. Commercial policies cover all business personal property on a worldwide basis.

B. Business personal property includes furniture, machinery, equipment, and stock that are not part of the building.

C. Commercial policies cover only those items of business personal property specifically listed on the declarations
page.

D. Business personal property also includes owned vehicles while parked within 100 feet of the described premises.

Question 11 D
Which one of the following statements is true regarding insurance policies?

A. Because of their special nature, insurance policies must meet several requirements in addition to those required for
other valid contracts.

B. Because the insurer's promise to pay claims spans several years, insurance policies have a separate set of require-
ments than those for any other valid contract.

C. Insurance policies are approved in advance and therefore are presumed to meet all of the requirements of a valid
contract.

D. Insurance policies must meet the same requirements as any other valid contract.

Question 12 B
A broad statement of an insurer's promises to the insured may be found in a policy's
A. Declarations.
B. Insuring agreements.
C. Conditions.
D. Endorsements.

188
Question 13 B
The homeowners policy excludes coverage for aircraft in an attempt to

A. Avoid covering uninsurable losses.

B. Eliminate coverage that most insureds do not need.

C. Eliminate duplicate coverage.

D. To avoid insuring losses that could be prevented.

Question 14 D
If you were a property owner and wanted the broadest coverage available to protect your building, you would pur-
chase

A. Basic form coverage.

B. Broader form coverage.

C. Broad form coverage.

D. Special form coverage.

Question 15 B
Rachel is an insurance broker. She has received a sales referral from another broker concerning an airport manage-
ment company, Airport Ltd. The company plans to operate a large, national airport in a major city and has requested
a quote for airport liability coverage. Rachel will probably recommend the use of which one of the following types of
policy forms?

A. Miscellaneous

B. Manuscript

C. Preprinted

D. Self-contained

Question 16 B
Most insurance policies exclude loss caused by war in order to

A. Eliminate duplicate coverage.

B. Avoid covering uninsurable losses.

C. Keep premiums reasonable.

D. Avoid insuring losses that could be prevented.

189
Question 17 D
If Katie purchases auto insurance from an out-of-state insurer without a license to sell insurance in Katies state, the
policy may be an invalid contract because

A. The contract does not have a legal purpose.

B. Utmost good faith was not practiced by the parties.

C. The principle of indemnity has been violated.

D. The insurer was not legally competent to make the agreement.

Question 18 C
An insurer will make a loss payment if an insured loss occurs and if the insured performs certain duties. This illustrates
that an insurance policy is a
A. Contract of adhesion.

B. Contract of indemnity.

C. Conditional contract.

D. Personal contract.

Question 19 D
Under a liability policy, the maximum an insurer will pay for injury to any one person is known as the

A. Per individual limit.

B. Single limit.

C. Each occurrence limit.

D. Each person limit.

Question 20 A
When Amy applied for insurance, her agent asked her if she had received more than three traffic tickets in the last five
years. Amy had received four, but remembered only three. Thus, she told the agent she had received only three. Amy's
statement is an example of

A. Misrepresentation.

B. Concealment.

C. Intent to deceive.

D. Intent to withhold information.

190
Question 21 C
While driving to work, John loses control of his automobile, which jumps a curb and strikes a tree. Which one of the
following coverages must John have purchased in order for the damage to his vehicle to be covered?

A. Comprehensive

B. Specified causes of loss

C. Collision

D. Vehicle damage

Question 22 D
All of the following are typical examples of conditions in an insurance policy pertaining to the insured's obligations,
EXCEPT:

A. Documenting losses

B. Reporting losses in a timely manner

C. Paying premiums

D. Defending itself from lawsuits

Question 23 A
The definitions section of the policy

A. Helps clarify real or perceived ambiguity.

B. Contains a definitive statement of coverage.

C. Consists of dictionary meanings of words.

D. Describes the services provided by the insurer.

Question 24 A
The validity of a contract depends on all of the following essential elements, EXCEPT:

A. Adhesion

B. Legal purpose

C. Consideration

D. Agreement

191
Question 25 D
All commercial package policies begin with two components, namely, common policy conditions and common

A. Coverages.

B. Manuscripts.

C. Modules.

D. Declarations.

Question 26 B
In creation of a contract of insurance, what is the consideration that is provided by the insurer?

A. The prompt payment of claims

B. The insurer's promise to pay claims for covered losses

C. The insurer's delivering the insurance policy to the insured

D. The acceptance of the application

Question 27 A
Angela is having a dispute with her insurance company regarding a claim and the definition of an insured loss. The
definition is subject to two reasonable interpretations, one that favors Angela and one that favors the insurer. A court
of law is likely to find in favor of

A. Angela, because an insurance contract is a contract of adhesion.

B. The insurer, because an insurance contract is a contract of adhesion.

C. Neither party, because each side has failed to meet the burden of proof.

D. Both parties, because the definition is subject to two interpretations.

Question 28 A
Depending on the policy terms and conditions, property insurance can protect other parties in addition to the named
insured. Which one of the following statements best describes the coverage for property of others provided under a
homeowners policy?

A. Homeowners policies can cover property of others who reside in the named insured's household relatives of any
age, and other persons under the age of twenty-one.
B. The spouse receives the same coverage as the named insured, even if the spouse is not named on the declarations
and is not residing in the same household.
C. The personal property of tenants of the named insured is automatically covered under a homeowners policy while
in the part of the residence that the tenant usually occupies.
D. A homeowners policy can provide worldwide coverage for property belonging to the named insured's guests, resi-
dence employees, and others.

192
Question 29 A
Standard forms

A. May result in different insurers issuing identical policies.

B. Prevent insurers from offering significantly different coverage.

C. Are commonly used in professional liability insurance.

D. Allow insurers to modify terms and conditions to accommodate differences in applicants' needs.

Question 30 D
A portion of a covered loss that is subtracted from the amount the insurer would otherwise be obligated to pay is the

A. Amount of the loan still outstanding.

B. Pro-rata premium.

C. Reinsurance.

D. Deductible.

193
CHAPTER 10

Question 1 D
The insured's young son hits a baseball through a neighbor's living room window. The damage caused by the child
would be covered under which one of the following coverages of the insured's homeowners policy?

A. Dwelling coverage

B. Other structures coverage

C. Loss of use coverage

D. Personal liability coverage

Question 2 A
To be eligible for coverage as an other structure under a homeowners policy, the other structure must be

A. Separated from the house by a clear space or fence.

B. Attached to the house.

C. Rented to others and not used by the insured.

D. Used as a garage, but not used as a utility shed.

Question 3 A
Loss exposures covered under an unendorsed dwelling policy include which one of the following consequences?

A. Wind damage to an insured's roof

B. Flood damage to an insured's dwelling

C. Injury by a tenant on the insured's rental property

D. Fire damage to an insured's automobile

Question 4 C
Damage from flood is covered under most

A. Policies that cover property at a fixed location.

B. Homeowners policies.

C. Auto policies.

D. Dwelling policies.

194
Question 5 B
Which one of the following best describes what is insured under dwelling coverage, as provided by a homeowners pol-
icy?

A. Dwelling only.

B. Dwelling and any structures attached to the house only.

C. Dwelling, any structures attached to the house and liability exposures.

D. Dwelling, structures attached to the house, liability exposures and loss of use.

Question 6 A
Liability coverage under a personal auto policy (PAP) applies to vehicles shown in the declarations and also applies to
insureds

A. If they rent a car while on vacation.

B. If they drive employer owned vehicles on a regular basis.

C. If they drive a forklift at their place of employment.

D. If they drive a neighbor's lawn tractor.

Question 7 C
Which one of the following coverages of a homeowners policy provides coverage for bodily injury and property dam-
age for which the insured is legally liable?

A. Dwelling coverage

B. Personal property coverage

C. Personal liability coverage

D. Loss of use coverage

Question 8 D
A self-insured retention (SIR) is

A. Applied to a dwelling policy as a policy deductible.

B. Applied to a personal inland marine policy as a policy deductible.

C. An amount that is deducted from claims that are payable under an umbrella liability policy and that are covered in
excess of a primary policy.

D. An amount that is deducted from claims that are payable under an umbrella liability policy and that are not cov-
ered at all by any primary policy.

195
Question 9 A
The dwelling program is often used to insure

A. A two-family rental dwelling owned by an insured who lives elsewhere in a primary home.

B. An apartment complex owned by an insured who lives elsewhere in a primary home.

C. A upscale single-family home in which the owner resides.

D. A residence for limits in excess of its homeowners policy.

Question 10 A
Which one of the following best describes applicable coverage under a personal auto policy (PAP) if an insured incurs
serious bodily injury caused by a hit-and-run driver?

A. Uninsured motorists coverage

B. Bodily injury liability coverage

C. Medical payments coverage

D. Physical damage coverage

Question 11 C
All homeowners policy forms

A. Provide coverages on a named perils basis.

B. Provide coverages on an open perils basis.

C. Follow a similar general pattern of construction but may vary in terms of coverage specifics.

D. Include the same coverage specifics but vary in terms of the general construction of the policy.

Question 12 A
Flood insurance provides

A. Building and contents coverage.

B. Bodily injury liability coverage.

C. Property damage liability coverage.

D. Umbrella liability coverage.

196
Question 13 A
While driving home from work on a dark and rainy evening, a driver loses control of his vehicle and strikes a pedes-
trian, causing severe injuries. The driver has purchased all primary coverages under a personal auto policy. Which one
of the following coverages applies to this accident?

A. Liability coverage

B. Medical payments coverage

C. Uninsured motorists coverage

D. Physical damage coverage

Question 14 A
Which one of the following best describes applicable coverage under a personal auto policy (PAP) if an insured incurs
serious bodily injuries caused by a driver whose insurer is insolvent?

A. Uninsured motorists coverage

B. Bodily injury liability coverage

C. Medical payments coverage

D. Physical damage coverage

Question 15 D
A delivery person trips and falls over the insured's uneven sidewalk. The delivery person sues the insured for his medi-
cal bills. This claim falls under which one of the following coverages of the insured's homeowners policy?

A. Other structures coverage

B. Dwelling coverage

C. Personal property coverage

D. Personal liability coverage

Question 16 C
Which one of the following is true about collision coverage under the personal auto policy (PAP)?

A. It is only provided if other than collision coverage is not provided.

B. It is subject to the property damage liability limit of the policy.

C. It is typically available only when other than collision coverage also is provided.

D. It is a mandatory coverage when other than collision coverage also is provided.

197
Question 17 B
Other than collision loss exposures under a personal auto policy (PAP) include all of the following, EXCEPT:

A. Auto theft

B. Collision with a building

C. Fire damage

D. Contact with a deer

Question 18 B
Under a homeowners policy, loss of use coverage provides coverage to an insured for

A. Bodily injury for which the insured is legally liable.

B. Periods during which the insured residence is uninhabitable by a covered loss.

C. Property damage for which the insured is legally liable.

D. Goodwill medical payments due to minor injuries incurred on the insured's premises.

Question 19 D
The personal auto policy (PAP) contains two sets of physical damage coverage, which are

A. Bodily injury coverage and property damage coverage.

B. Property damage coverage and collision coverage.

C. Uninsured motorists coverage and underinsured motorists coverage.

D. Other than collision coverage and collision coverage.

Question 20 C
Personal property coverage as provided on a homeowners policy typically protects against damage to all of the fol-
lowing, EXCEPT:

A. Clothing

B. Furniture

C. Utility shed

D. Appliances

198
Question 1 C
Damage from flood is covered under most

A. Policies that cover property at a fixed location.

B. Homeowners policies.

C. Auto policies.

D. Dwelling policies.

Question 2 B
Which one of the following best describes what is insured under dwelling coverage, as provided by a homeowners pol-
icy?

A. Dwelling only.

B. Dwelling and any structures attached to the house only.

C. Dwelling, any structures attached to the house and liability exposures.

D. Dwelling, structures attached to the house, liability exposures and loss of use.

Question 3 A
The dwelling program is often used to insure

A. A two-family rental dwelling owned by an insured who lives elsewhere in a primary home.

B. An apartment complex owned by an insured who lives elsewhere in a primary home.

C. A upscale single-family home in which the owner resides.

D. A residence for limits in excess of its homeowners policy.

Question 4 A
Flood insurance provides

A. Building and contents coverage.

B. Bodily injury liability coverage.

C. Property damage liability coverage.

D. Umbrella liability coverage.

199
Question 5 A
Which one of the following best describes a type of loss covered under a personal umbrella policy?

A. A catastrophic auto accident that exceeds an insured's auto liability limit

B. A catastrophic fire that totally destroys the insured residence

C. Theft of high valued items such as jewelry and silverware from the insured's residence

D. A severe flood that totally destroys the insured's vacation property

Question 6 C
Which one of the following is true about collision coverage under the personal auto policy (PAP)?

A. It is only provided if other than collision coverage is not provided.

B. It is subject to the property damage liability limit of the policy.

C. It is typically available only when other than collision coverage also is provided.

D. It is a mandatory coverage when other than collision coverage also is provided.

Question 7 B
Under a homeowners policy, loss of use coverage provides coverage to an insured for

A. Bodily injury for which the insured is legally liable.

B. Periods during which the insured residence is uninhabitable by a covered loss.

C. Property damage for which the insured is legally liable.

D. Goodwill medical payments due to minor injuries incurred on the insured's premises.

Question 8 A
While driving home from work on a dark and rainy evening, a driver loses control of his vehicle and strikes a pedes-
trian, causing severe injuries. The driver has purchased all primary coverages under a personal auto policy. Which one
of the following coverages applies to this accident?

A. Liability coverage

B. Medical payments coverage

C. Uninsured motorists coverage

D. Physical damage coverage

200
Question 9 D
A delivery person trips and falls over the insured's uneven sidewalk. The delivery person sues the insured for his medi-
cal bills. This claim falls under which one of the following coverages of the insured's homeowners policy?

A. Other structures coverage

B. Dwelling coverage

C. Personal property coverage

D. Personal liability coverage

Question 10 A
Liability coverage under a personal auto policy (PAP) applies to vehicles shown in the declarations and also applies to
insureds

A. If they rent a car while on vacation.

B. If they drive employer owned vehicles on a regular basis.

C. If they drive a forklift at their place of employment.

D. If they drive a neighbor's lawn tractor.

Question 11 A
Which one of the following best describes applicable coverage under a personal auto policy (PAP) if an insured incurs
serious bodily injury caused by a hit-and-run driver?

A. Uninsured motorists coverage

B. Bodily injury liability coverage

C. Medical payments coverage

D. Physical damage coverage

Question 12 D
The personal auto policy (PAP) contains two sets of physical damage coverage, which are

A. Bodily injury coverage and property damage coverage.

B. Property damage coverage and collision coverage.

C. Uninsured motorists coverage and underinsured motorists coverage.

201
D. Other than collision coverage and collision coverage.

Question 13 B
Other than collision loss exposures under a personal auto policy (PAP) include all of the following, EXCEPT:

A. Auto theft

B. Collision with a building

C. Fire damage

D. Contact with a deer

Question 14 C
Which one of the following coverages of a homeowners policy provides coverage for bodily injury and property dam-
age for which the insured is legally liable?

A. Dwelling coverage

B. Personal property coverage

C. Personal liability coverage

D. Loss of use coverage

Question 15 C
All homeowners policy forms

A. Provide coverages on a named perils basis.

B. Provide coverages on an open perils basis.

C. Follow a similar general pattern of construction but may vary in terms of coverage specifics.

D. Include the same coverage specifics but vary in terms of the general construction of the policy.

Question 16 C
Personal property coverage as provided on a homeowners policy typically protects against damage to all of the fol-
lowing, EXCEPT:

A. Clothing

B. Furniture

C. Utility shed

D. Appliances

202
Question 17 A
Which one of the following best describes applicable coverage under a personal auto policy (PAP) if an insured incurs
serious bodily injuries caused by a driver whose insurer is insolvent?

A. Uninsured motorists coverage

B. Bodily injury liability coverage

C. Medical payments coverage

D. Physical damage coverage

Question 18 C
The unendorsed dwelling policy provides coverage for

A. The dwelling, but not other structures.

B. The dwelling and other structures, but not personal property.

C. The dwelling, other structures, and personal property.

D. The dwelling, other structures, personal property and personal liability coverage.

Question 19 D
The insured's young son hits a baseball through a neighbor's living room window. The damage caused by the child
would be covered under which one of the following coverages of the insured's homeowners policy?

A. Dwelling coverage

B. Other structures coverage

C. Loss of use coverage

D. Personal liability coverage

Question 20 A
To be eligible for coverage as an other structure under a homeowners policy, the other structure must be

A. Separated from the house by a clear space or fence.

B. Attached to the house.

C. Rented to others and not used by the insured.

D. Used as a garage, but not used as a utility shed.

203
Question 1 C
Personal property coverage as provided on a homeowners policy typically protects against damage to all of the fol-
lowing, EXCEPT:

A. Clothing

B. Furniture

C. Utility shed

D. Appliances

Question 2 C
Damage from flood is covered under most

A. Policies that cover property at a fixed location.

B. Homeowners policies.

C. Auto policies.

D. Dwelling policies.

Question 3 A
To be eligible for coverage as an other structure under a homeowners policy, the other structure must be

A. Separated from the house by a clear space or fence.

B. Attached to the house.

C. Rented to others and not used by the insured.

D. Used as a garage, but not used as a utility shed.

Question 4 C
The unendorsed dwelling policy provides coverage for

A. The dwelling, but not other structures.

B. The dwelling and other structures, but not personal property.

C. The dwelling, other structures, and personal property.

D. The dwelling, other structures, personal property and personal liability coverage.

204
Question 5 A
While driving home from work on a dark and rainy evening, a driver loses control of his vehicle and strikes a pedes-
trian, causing severe injuries. The driver has purchased all primary coverages under a personal auto policy. Which one
of the following coverages applies to this accident?

A. Liability coverage

B. Medical payments coverage

C. Uninsured motorists coverage

D. Physical damage coverage

Question 6 D
A delivery person trips and falls over the insured's uneven sidewalk. The delivery person sues the insured for his medi-
cal bills. This claim falls under which one of the following coverages of the insured's homeowners policy?

A. Other structures coverage

B. Dwelling coverage

C. Personal property coverage

D. Personal liability coverage

Question 7 A
Flood insurance provides

A. Building and contents coverage.

B. Bodily injury liability coverage.

C. Property damage liability coverage.

D. Umbrella liability coverage.

Question 8 C
Which one of the following coverages of a homeowners policy provides coverage for bodily injury and property dam-
age for which the insured is legally liable?

A. Dwelling coverage

B. Personal property coverage

C. Personal liability coverage

D. Loss of use coverage

205
Question 9 A
Which one of the following best describes a type of loss covered under a personal umbrella policy?

A. A catastrophic auto accident that exceeds an insured's auto liability limit

B. A catastrophic fire that totally destroys the insured residence

C. Theft of high valued items such as jewelry and silverware from the insured's residence

D. A severe flood that totally destroys the insured's vacation property

Question 10 A
Which one of the following best describes applicable coverage under a personal auto policy (PAP) if an insured incurs
serious bodily injury caused by a hit-and-run driver?

A. Uninsured motorists coverage

B. Bodily injury liability coverage

C. Medical payments coverage

D. Physical damage coverage

Question 11 C
Which one of the following is true about collision coverage under the personal auto policy (PAP)?

A. It is only provided if other than collision coverage is not provided.

B. It is subject to the property damage liability limit of the policy.

C. It is typically available only when other than collision coverage also is provided.

D. It is a mandatory coverage when other than collision coverage also is provided.

Question 12 D
A self-insured retention (SIR) is

A. Applied to a dwelling policy as a policy deductible.

B. Applied to a personal inland marine policy as a policy deductible.

C. An amount that is deducted from claims that are payable under an umbrella liability policy and that are covered in
excess of a primary policy.

D. An amount that is deducted from claims that are payable under an umbrella liability policy and that are not cov-
ered at all by any primary policy.

206
Question 13 A
Liability coverage under a personal auto policy (PAP) applies to vehicles shown in the declarations and also applies to
insureds

A. If they rent a car while on vacation.

B. If they drive employer owned vehicles on a regular basis.

C. If they drive a forklift at their place of employment.

D. If they drive a neighbor's lawn tractor.

Question 14 B
Other than collision loss exposures under a personal auto policy (PAP) include all of the following, EXCEPT:

A. Auto theft

B. Collision with a building

C. Fire damage

D. Contact with a deer

Question 15 C
All homeowners policy forms

A. Provide coverages on a named perils basis.

B. Provide coverages on an open perils basis.

C. Follow a similar general pattern of construction but may vary in terms of coverage specifics.

D. Include the same coverage specifics but vary in terms of the general construction of the policy.

Question 16 D
The insured's young son hits a baseball through a neighbor's living room window. The damage caused by the child
would be covered under which one of the following coverages of the insured's homeowners policy?

A. Dwelling coverage

B. Other structures coverage

C. Loss of use coverage

D. Personal liability coverage

207
Question 17 B
Under a homeowners policy, loss of use coverage provides coverage to an insured for

A. Bodily injury for which the insured is legally liable.

B. Periods during which the insured residence is uninhabitable by a covered loss.

C. Property damage for which the insured is legally liable.

D. Goodwill medical payments due to minor injuries incurred on the insured's premises.

Question 18 B
Which one of the following best describes what is insured under dwelling coverage, as provided by a homeowners pol-
icy?

A. Dwelling only.

B. Dwelling and any structures attached to the house only.

C. Dwelling, any structures attached to the house and liability exposures.

D. Dwelling, structures attached to the house, liability exposures and loss of use.

Question 19 A
Which one of the following best describes applicable coverage under a personal auto policy (PAP) if an insured incurs
serious bodily injuries caused by a driver whose insurer is insolvent?

A. Uninsured motorists coverage

B. Bodily injury liability coverage

C. Medical payments coverage

D. Physical damage coverage

Question 20 D
The personal auto policy (PAP) contains two sets of physical damage coverage, which are

A. Bodily injury coverage and property damage coverage.

B. Property damage coverage and collision coverage.

C. Uninsured motorists coverage and underinsured motorists coverage.

D. Other than collision coverage and collision coverage.

208
CHAPTER 11

Question 1 B
For which of the following does the Building and Personal Property Coverage Form (BPP) usually provide very limited
coverage?

A. Building

B. Property away from the insured's premises

C. Personal property

D. Personal property of others

Question 2 B
Umbrella liability insurance is often referred to as "excess" insurance because it

A. Is not needed by most insureds.

B. Applies in excess of the primary policies.

C. Provides unlimited coverage.

D. Often provides broader coverage than the primary policies.

Question 3 A
ABC Company (ABC) manufactures welding torches and sells them to contractors. One of the torches used by a con-
tractor malfunctions and causes a fire that destroys a building under construction. Which one of the following would
cover ABC against a subsequent products liability claim for the fire loss?

A. Coverage ABodily Injury and Property Damage Liability of ABC's commercial general liability (CGL) policy

B. ABC's employers liability insurance

C. Coverage BPersonal and Advertising Injury Liability of ABC's commercial general liability (CGL) policy

D. ABC's premises liability insurance

Question 4 B
Most policies that provide coverage on buildings and personal property at fixed locations exclude coverage for
A. Fire and lightning losses.
B. Earthquake and flood losses.
C. Riot or civil commotion losses.
D. Sinkhole collapse and volcanic action losses.

209
Question 5 C
Building and contents insurance policies usually provide either a nominal amount of coverage or no coverage for all of
the following, EXCEPT:

A. Merchandise shipped by railcar

B. Property that moves from site-to-site

C. Completed products in inventory

D. Property in the possession of others

Question 6 A
The main purpose of umbrella liability insurance is to provide

A. Limits of insurance that apply in addition to those of the insured's primary policies.

B. Unlimited medical expense coverage.

C. Adequate limits of insurance for workers compensation benefits.

D. Disability income benefits.

Question 7 B
Under a Commercial General Liability (CGL) Coverage Form, the purpose of Coverage CMedical Payments is to pro-
vide

A. Unlimited medical expense coverage.

B. A modest amount of medical expense coverage for settling minor injury cases.

C. Medical expense coverage for injury arising from products.

D. Medical expense coverage for injury arising from personal injury.

Question 8 D
Which one of the following is likely to be covered by inland marine insurance rather than by the Building and Personal
Property Coverage Form (BPP)?

A. Completed additions to covered buildings

B. Permanently installed production machinery

C. Personal property used to service a covered building

D. Personal property situated at the premises of others

210
Question 9 C
Which one of the following is excluded under the Building and Personal Property Coverage Form (BPP)?

A. Completed additions to covered buildings

B. Business personal property located within 100 feet of the premises

C. Property in transit

D. Personal property used to maintain the building

Question 10 A
The benefits that workers compensation laws require employers to pay include all of the following, EXCEPT:

A. Meal expenses

B. Medical expenses

C. Disability income

D. Rehabilitation benefits

Question 11 B
Under a Commercial General Liability (CGL) Coverage Form, medical payments coverage pays

A. Only if the insured is not legally liable.

B. Regardless of whether the insured is legally liable.

C. Only if the insured is legally liable.

D. Regardless of whether the insured incurs any medical expenses.

Question 12 B
Which one of the following causes of loss forms is the lowest-cost version that provides coverage for approximately
one dozen named perils?

A. Special form

B. Basic form

C. Intermediate form

D. Broad form

211
Question 13 D
Under the Building and Personal Property Coverage Form (BPP), "your business personal property" is covered under all
of the following conditions, EXCEPT:

A. In the described building

B. In the open within 100 feet of the premises

C. On the described building

D. Situated at the premises of others on a permanent basis

Question 14 D
Virtually all employers purchase workers compensation and employers liability insurance to cover workers compensa-
tion benefits and liability

A. To employees for advertising injury.

B. To employees for property damage.

C. For employee injury in which the employee subsequently dies.

D. For employee injury not covered by workers compensation statutes.

Question 15 A
Extra expense insurance covers additional expenses incurred to

A. Reduce the length of a business interruption.

B. Suspend the insured's operations.

C. Cover "floating property."

D. Cover the "maintenance" perils.

Question 16 D
An architect makes false statements about one of his competitors, who sues him for slander. Which one of the follow-
ing would cover the architect for a loss arising out of this situation?

A. Coverage A Bodily Injury and Property Damage Liability of the Commercial General Liability (CGL) Coverage Form.

B. Employers liability insurance

C. Operations liability insurance

D. Coverage B Personal and Advertising Injury Liability of the Commercial General Liability (CGL) Coverage Form

212
Question 17 C
Which one of the following is true with regard to auto liability coverage under business auto insurance?

A. It does not cover liability for injury resulting from faulty maintenance of an auto.

B. It covers bodily injury and property damage regardless of whether the insured is legally liable.

C. It can be arranged to cover liability arising from autos the insured does not own, lease, or hire but that are used on
the insured's behalf.

D. Commercial general liability (CGL) insurance and business auto insurance overlap and cover the same auto liability
losses.

Question 18 D
Which one of the following can be insured by the Building and Personal Property Coverage Form (BPP)?

A. A bridge

B. Money

C. Securities

D. Merchandise

Question 19 A
Under Coverage A of the Commercial General Liability (CGL) Coverage Form, the insurer agrees to pay those sums that
the insured becomes legally obligated to pay as damages because of property damage or

A. Bodily injury.

B. Personal injury.

C. Advertising injury.

D. Medical injury.

Question 20 B
Damage to property caused by rust is uninsurable because it is

A. Difficult to value.

B. Certain to occur over time.

C. Usually due to latent defects.

D. Catastrophic in nature.

213
Question 21 C
Which one of the following is considered to be a "maintenance peril"?

A. Nuclear reaction

B. Sprinkler leakage

C. Damage by insects

D. Falling objects

Question 22 C
Coverage ABodily Injury and Property Damage Liability of a Commercial General Liability (CGL) Coverage Form in-
sures two general categories of loss exposures. One is premises and operations liability and the other is

A. Medical payments.

B. Personal injury liability.

C. Products and completed operations liability.

D. Advertising liability.

Question 23 B
Under the Building and Personal Property Coverage Form (BPP), personal property of others is covered

A. Only if the insured is legally liable for it.

B. While in the insured's care, custody, or control.

C. While in transit.

D. Up to 500 feet from the described premises.

Question 24 D
Under a Commercial General Liability (CGL) Coverage Form, property damage means physical injury to property and
the

A. Cost of repairing the property.

B. Related premises liability.

C. Cost of defending claims for property damage.

D. Resulting loss of use.

214
Question 25 C
All of the following conditions must exist in order for business income insurance to cover a loss, EXCEPT:

A. There must be a reduction in the insured's net income.

B. There must be a suspension of the insured's operations.

C. There must be a reduction in extra expenses.

D. There must be damage by a covered cause of loss at the insured's location.

Question 1 C
All of the following conditions must exist in order for business income insurance to cover a loss, EXCEPT:

A. There must be a reduction in the insured's net income.

B. There must be a suspension of the insured's operations.

C. There must be a reduction in extra expenses.

D. There must be damage by a covered cause of loss at the insured's location.

Question 2 C
Which one of the following is true with regard to auto liability coverage under business auto insurance?

A. It does not cover liability for injury resulting from faulty maintenance of an auto.

B. It covers bodily injury and property damage regardless of whether the insured is legally liable.

C. It can be arranged to cover liability arising from autos the insured does not own, lease, or hire but that are used on
the insured's behalf.

D. Commercial general liability (CGL) insurance and business auto insurance overlap and cover the same auto liability
losses.

Question 3 C
Coverage ABodily Injury and Property Damage Liability of a Commercial General Liability (CGL) Coverage Form in-
sures two general categories of loss exposures. One is premises and operations liability and the other is

A. Medical payments.

B. Personal injury liability.

C. Products and completed operations liability.

D. Advertising liability.

215
Question 4 B
Umbrella liability insurance is often referred to as "excess" insurance because it

A. Is not needed by most insureds.

B. Applies in excess of the primary policies.

C. Provides unlimited coverage.

D. Often provides broader coverage than the primary policies.

Question 5 D
Which one of the following can be insured by the Building and Personal Property Coverage Form (BPP)?

A. A bridge

B. Money

C. Securities

D. Merchandise

Question 6 D
Which one of the following is likely to be covered by inland marine insurance rather than by the Building and Personal
Property Coverage Form (BPP)?

A. Completed additions to covered buildings

B. Permanently installed production machinery

C. Personal property used to service a covered building

D. Personal property situated at the premises of others

Question 7 D
Under the Building and Personal Property Coverage Form (BPP), "your business personal property" is covered under all
of the following conditions, EXCEPT:

A. In the described building

B. In the open within 100 feet of the premises

C. On the described building

D. Situated at the premises of others on a permanent basis

216
Question 8 C
Which one of the following is considered to be a "maintenance peril"?

A. Nuclear reaction

B. Sprinkler leakage

C. Damage by insects

D. Falling objects

Question 9 B
For which of the following does the Building and Personal Property Coverage Form (BPP) usually provide very limited
coverage?

A. Building

B. Property away from the insured's premises

C. Personal property

D. Personal property of others

Question 10 C
Which one of the following is excluded under the Building and Personal Property Coverage Form (BPP)?

A. Completed additions to covered buildings

B. Business personal property located within 100 feet of the premises

C. Property in transit

D. Personal property used to maintain the building

Question 11 B
Most policies that provide coverage on buildings and personal property at fixed locations exclude coverage for

A. Fire and lightning losses.

B. Earthquake and flood losses.

C. Riot or civil commotion losses.

D. Sinkhole collapse and volcanic action losses.

217
Question 12 A
Extra expense insurance covers additional expenses incurred to

A. Reduce the length of a business interruption.

B. Suspend the insured's operations.

C. Cover "floating property."

D. Cover the "maintenance" perils.

Question 13 A
The benefits that workers compensation laws require employers to pay include all of the following, EXCEPT:

A. Meal expenses

B. Medical expenses

C. Disability income

D. Rehabilitation benefits

Question 14 D
Under a Commercial General Liability (CGL) Coverage Form, property damage means physical injury to property and
the

A. Cost of repairing the property.

B. Related premises liability.

C. Cost of defending claims for property damage.

D. Resulting loss of use.

Question 15 A
The main purpose of umbrella liability insurance is to provide

A. Limits of insurance that apply in addition to those of the insured's primary policies.

B. Unlimited medical expense coverage.

C. Adequate limits of insurance for workers compensation benefits.

D. Disability income benefits.

218
Question 16 B
Which one of the following causes of loss forms is the lowest-cost version that provides coverage for approximately
one dozen named perils?

A. Special form

B. Basic form

C. Intermediate form

D. Broad form

Question 17 B
Under the Building and Personal Property Coverage Form (BPP), personal property of others is covered

A. Only if the insured is legally liable for it.

B. While in the insured's care, custody, or control.

C. While in transit.

D. Up to 500 feet from the described premises.

Question 18 D
Virtually all employers purchase workers compensation and employers liability insurance to cover workers compensa-
tion benefits and liability

A. To employees for advertising injury.

B. To employees for property damage.

C. For employee injury in which the employee subsequently dies.

D. For employee injury not covered by workers compensation statutes.

Question 19 C
Building and contents insurance policies usually provide either a nominal amount of coverage or no coverage for all of
the following, EXCEPT:

A. Merchandise shipped by railcar

B. Property that moves from site-to-site

C. Completed products in inventory

D. Property in the possession of others

219
Question 20 A
Under Coverage A of the Commercial General Liability (CGL) Coverage Form, the insurer agrees to pay those sums that
the insured becomes legally obligated to pay as damages because of property damage or

A. Bodily injury.

B. Personal injury.

C. Advertising injury.

D. Medical injury.

Question 21 A
ABC Company (ABC) manufactures welding torches and sells them to contractors. One of the torches used by a con-
tractor malfunctions and causes a fire that destroys a building under construction. Which one of the following would
cover ABC against a subsequent products liability claim for the fire loss?

A. Coverage ABodily Injury and Property Damage Liability of ABC's commercial general liability (CGL) policy

B. ABC's employers liability insurance

C. Coverage BPersonal and Advertising Injury Liability of ABC's commercial general liability (CGL) policy

D. ABC's premises liability insurance

Question 22 B
Under a Commercial General Liability (CGL) Coverage Form, medical payments coverage pays

A. Only if the insured is not legally liable.

B. Regardless of whether the insured is legally liable.

C. Only if the insured is legally liable.

D. Regardless of whether the insured incurs any medical expenses.

Question 23 D
An architect makes false statements about one of his competitors, who sues him for slander. Which one of the follow-
ing would cover the architect for a loss arising out of this situation?

A. Coverage A Bodily Injury and Property Damage Liability of the Commercial General Liability (CGL) Coverage Form.

B. Employers liability insurance

C. Operations liability insurance

D. Coverage B Personal and Advertising Injury Liability of the Commercial General Liability (CGL) Coverage Form

220
Question 24 B
Damage to property caused by rust is uninsurable because it is

A. Difficult to value.

B. Certain to occur over time.

C. Usually due to latent defects.

D. Catastrophic in nature.

Question 25 B
Under a Commercial General Liability (CGL) Coverage Form, the purpose of Coverage CMedical Payments is to pro-
vide

A. Unlimited medical expense coverage.

B. A modest amount of medical expense coverage for settling minor injury cases.

C. Medical expense coverage for injury arising from products.

D. Medical expense coverage for injury arising from personal injury.

Question 1 B
Damage to property caused by rust is uninsurable because it is

A. Difficult to value.

B. Certain to occur over time.

C. Usually due to latent defects.

D. Catastrophic in nature.

Question 2 D
Virtually all employers purchase workers compensation and employers liability insurance to cover workers compensa-
tion benefits and liability

A. To employees for advertising injury.

B. To employees for property damage.

C. For employee injury in which the employee subsequently dies.

221
D. For employee injury not covered by workers compensation statutes.

Question 3 B
For which of the following does the Building and Personal Property Coverage Form (BPP) usually provide very limited
coverage?

A. Building

B. Property away from the insured's premises

C. Personal property

D. Personal property of others

Question 4 D
Under a Commercial General Liability (CGL) Coverage Form, property damage means physical injury to property and
the

A. Cost of repairing the property.

B. Related premises liability.

C. Cost of defending claims for property damage.

D. Resulting loss of use.

Question 5 D
Which one of the following is likely to be covered by inland marine insurance rather than by the Building and Personal
Property Coverage Form (BPP)?

A. Completed additions to covered buildings

B. Permanently installed production machinery

C. Personal property used to service a covered building

D. Personal property situated at the premises of others

Question 6 D
An architect makes false statements about one of his competitors, who sues him for slander. Which one of the follow-
ing would cover the architect for a loss arising out of this situation?

A. Coverage A Bodily Injury and Property Damage Liability of the Commercial General Liability (CGL) Coverage Form.

B. Employers liability insurance

C. Operations liability insurance

D. Coverage B Personal and Advertising Injury Liability of the Commercial General Liability (CGL) Coverage Form

222
Question 7 A
Under Coverage A of the Commercial General Liability (CGL) Coverage Form, the insurer agrees to pay those sums that
the insured becomes legally obligated to pay as damages because of property damage or

A. Bodily injury.

B. Personal injury.

C. Advertising injury.

D. Medical injury.

Question 8 C
Coverage ABodily Injury and Property Damage Liability of a Commercial General Liability (CGL) Coverage Form in-
sures two general categories of loss exposures. One is premises and operations liability and the other is

A. Medical payments.

B. Personal injury liability.

C. Products and completed operations liability.

D. Advertising liability.

Question 9 D
Which one of the following can be insured by the Building and Personal Property Coverage Form (BPP)?

A. A bridge

B. Money

C. Securities

D. Merchandise

Question 10 A
The main purpose of umbrella liability insurance is to provide

A. Limits of insurance that apply in addition to those of the insured's primary policies.

B. Unlimited medical expense coverage.

C. Adequate limits of insurance for workers compensation benefits.

D. Disability income benefits.

223
Question 11 D
Under the Building and Personal Property Coverage Form (BPP), "your business personal property" is covered under all
of the following conditions, EXCEPT:

A. In the described building

B. In the open within 100 feet of the premises

C. On the described building

D. Situated at the premises of others on a permanent basis

Question 12 A
ABC Company (ABC) manufactures welding torches and sells them to contractors. One of the torches used by a con-
tractor malfunctions and causes a fire that destroys a building under construction. Which one of the following would
cover ABC against a subsequent products liability claim for the fire loss?

A. Coverage ABodily Injury and Property Damage Liability of ABC's commercial general liability (CGL) policy

B. ABC's employers liability insurance

C. Coverage BPersonal and Advertising Injury Liability of ABC's commercial general liability (CGL) policy

D. ABC's premises liability insurance

Question 13 C
All of the following conditions must exist in order for business income insurance to cover a loss, EXCEPT:

A. There must be a reduction in the insured's net income.

B. There must be a suspension of the insured's operations.

C. There must be a reduction in extra expenses.

D. There must be damage by a covered cause of loss at the insured's location.

Question 14 B
Umbrella liability insurance is often referred to as "excess" insurance because it

A. Is not needed by most insureds.

B. Applies in excess of the primary policies.

C. Provides unlimited coverage.

D. Often provides broader coverage than the primary policies.

224
Question 15 B
Which one of the following causes of loss forms is the lowest-cost version that provides coverage for approximately
one dozen named perils?

A. Special form

B. Basic form

C. Intermediate form

D. Broad form

Question 16 B
Most policies that provide coverage on buildings and personal property at fixed locations exclude coverage for

A. Fire and lightning losses.

B. Earthquake and flood losses.

C. Riot or civil commotion losses.

D. Sinkhole collapse and volcanic action losses.

Question 17 B
Under a Commercial General Liability (CGL) Coverage Form, medical payments coverage pays

A. Only if the insured is not legally liable.

B. Regardless of whether the insured is legally liable.

C. Only if the insured is legally liable.

D. Regardless of whether the insured incurs any medical expenses.

Question 18 C
Which one of the following is considered to be a "maintenance peril"?

A. Nuclear reaction

B. Sprinkler leakage

C. Damage by insects

D. Falling objects

225
Question 19 C
Building and contents insurance policies usually provide either a nominal amount of coverage or no coverage for all of
the following, EXCEPT:

A. Merchandise shipped by railcar

B. Property that moves from site-to-site

C. Completed products in inventory

D. Property in the possession of others

Question 20 B
Under a Commercial General Liability (CGL) Coverage Form, the purpose of Coverage CMedical Payments is to pro-
vide

A. Unlimited medical expense coverage.

B. A modest amount of medical expense coverage for settling minor injury cases.

C. Medical expense coverage for injury arising from products.

D. Medical expense coverage for injury arising from personal injury.

Question 21 C
Which one of the following is true with regard to auto liability coverage under business auto insurance?

A. It does not cover liability for injury resulting from faulty maintenance of an auto.

B. It covers bodily injury and property damage regardless of whether the insured is legally liable.

C. It can be arranged to cover liability arising from autos the insured does not own, lease, or hire but that are used on
the insured's behalf.

D. Commercial general liability (CGL) insurance and business auto insurance overlap and cover the same auto liability
losses.

Question 22 A
The benefits that workers compensation laws require employers to pay include all of the following, EXCEPT:

A. Meal expenses

B. Medical expenses

C. Disability income

D. Rehabilitation benefits

226
Question 23 B
Under the Building and Personal Property Coverage Form (BPP), personal property of others is covered

A. Only if the insured is legally liable for it.

B. While in the insured's care, custody, or control.

C. While in transit.

D. Up to 500 feet from the described premises.

Question 24 C
Which one of the following is excluded under the Building and Personal Property Coverage Form (BPP)?

A. Completed additions to covered buildings

B. Business personal property located within 100 feet of the premises

C. Property in transit

D. Personal property used to maintain the building

Question 25 A
Extra expense insurance covers additional expenses incurred to

A. Reduce the length of a business interruption.

B. Suspend the insured's operations.

C. Cover "floating property."

D. Cover the "maintenance" perils.

END

227

Potrebbero piacerti anche